Tải bản đầy đủ (.pdf) (72 trang)

Bộ đề thi vào lớp 10 chuyên Toán Thái Bình

Bạn đang xem bản rút gọn của tài liệu. Xem và tải ngay bản đầy đủ của tài liệu tại đây (1.97 MB, 72 trang )

<span class='text_page_counter'>(1)</span><div class='page_container' data-page=1>





<b> T</b>

<b>rịnh Bình sưu tầm tổng hợp </b>



<b>BỘ ĐỀ THI VÀO LỚP 10 </b>



<b>CHUN MƠN TỐN THÁI BÌNH </b>



</div>
<span class='text_page_counter'>(2)</span><div class='page_container' data-page=2>

<b>SỞ GIÁO DỤC VÀ ĐÀO TẠO </b>


<b>THÁI BÌNH </b> <b>ĐỀTHI TUYỂN SINH LỚP 10 THPT CHUYÊN THÁI BÌNH<sub>NĂM HỌ</sub><sub>C 2019 – 2020 </sub></b>
<b>MƠNTHI:TỐN</b>


<b>(Dành cho tất cả các thí sinh) </b>


<i>Thời gian làm bài:<b> 120 phút</b> (Không kể thời gian giao đề) </i>
<b>Đề số 1 </b>


<b>Câu 1.</b><i>(2,0 điểm)</i> Cho biểu thức: =<sub></sub> 2 + +1 1<sub></sub>⋅

(

+

)


+


 


<i>xy x</i> <i>y</i> <i>xy</i>


<i>P</i>


<i>x</i> <i>y</i>


<i>xy</i> <i>x x</i> <i>y y</i> (với <i>x</i>>0;<i>y</i>>0).



1. Rút gọn biểu thức <i>P</i>.


2. Biết <i>xy</i> =16. Tìm giá trị nhỏ nhất của <i>P</i>.


<b>Câu 2.</b> <i>(1,0 điểm)</i> Hai lớp 9A và 9B của một trường qun góp sách ủng hộ. Trung bình
mỗi bạn lớp 9A ủng hộ 5 quyển, mỗi bạn lớp 9B ủng hộ 6 quyển nên cả hai lớp ủng hộ 493
quyển. Tính số học sinh mỗi lớp biết tổng số học sinh của hai lớp là 90.


<b>Câu 3.</b><i>(2,0 điểm)</i> Trên mặt phẳng tọa độOxy, cho hai đường thẳng


2
1


( ) :<i>d</i> <i>y</i>=(<i>m</i> +1)<i>x</i>−2<i>m</i> và (<i>d</i><sub>2</sub>) :<i>y</i>=(<i>m</i>+3)<i>x</i>− −<i>m</i> 2 (<i>m</i> là tham số).
1. Tìm <i>m</i>để ( )<i>d</i>1 song song với (<i>d</i>2).


2. Chứng minh: với mọi <i>m</i>đường thẳng (<i>d</i><sub>2</sub>) luôn đi qua một điểm cốđịnh.


3. Tìm m để ( ), (<i>d</i>1 <i>d</i>2) cắt nhau tại <i>M x</i>( <i>M</i>;<i>yM</i>) thỏa mãn <i>A</i>=2020<i>xM</i>(<i>yM</i> +2) đạt giá


trị nhỏ nhất.


<b>Câu 4.</b><i>(1,0 điểm)</i> Giải hệphương trình:


3 3 2 2


2


( 1) ( 1) 0



4 4 2 7


 − + − − + =





+ + = + +





<i>x</i> <i>y</i> <i>x</i> <i>y</i> <i>y</i> <i>x</i>


<i>x</i> <i>y</i> <i>x</i> <i>y</i>


<b>Câu 5.</b><i>(3,5 điểm)</i> Cho tam giác <i>ABC</i> nhọn có <i>AB</i> < <i>AC</i> nội tiếp đường trịn tâm <i>O</i> bán kính
<i>R</i>, vẽ<i>AH</i> vng góc với <i>BC</i> tại <i>H</i>, vẽđường kính <i>AD</i> cắt <i>BC</i> tại <i>I</i>, trên cạnh <i>AC</i> lấy điểm <i>M</i>
sao cho <i>IM</i> song song với <i>CD</i>.


1. Chứng minh: Tứ giác <i>AHIM</i> nội tiếp một đường tròn.
2. Chứng minh:

<i>AB AC</i>

.

=

<i>AH AD</i>

.

.


3. Chứng minh: <i>HM</i> là tiếp tuyến của đường tròn ngoại tiếp tam giác <i>ABH</i>.


4. Chứng minh: 2


.

+

.

<

4


<i>AB CD</i>

<i>AC BD</i>

<i>R</i>

.


<b>Câu 6.</b> <i>(0,5 điểm)</i> Xét các số thực

<i>a b c a</i>

; ; (

0)

sao cho phương trình bậc hai


2


0



+

+ =



<i>ax</i>

<i>bx</i>

<i>c</i>

có hai nghiệm

<i>m n</i>

;

thỏa mãn: 0≤ ≤<i>m</i> 1;0≤ ≤<i>n</i> 1. Tìm giá trị nhỏ nhất của
biểu thức: = 2 2−<sub>2</sub> −2 +


− +


<i>a</i> <i>ac</i> <i>ab bc</i>


<i>Q</i>


<i>a</i> <i>ab</i> <i>ac</i>


<b>---Hết--- </b>


</div>
<span class='text_page_counter'>(3)</span><div class='page_container' data-page=3>

<b>SỞ GIÁO DỤC VÀ ĐÀO TẠO </b>


<b>THÁI BÌNH </b> <b>ĐỀTHI TUYỂN SINH LỚP 10 THPT CHUYÊN THÁI BÌNH<sub>NĂM HỌ</sub><sub>C 2019 – 2020 </sub></b>
<b>MƠNTHI:TỐN</b>


<b>(Dành cho học sinh chuyên toán tin) </b>
<i>Thời gian làm bài:<b> 150 phút</b> (Không kể thời gian giao đề) </i>
<b>Đề số 2 </b>


<b>Câu 1. (2,0 điểm) </b>Cho các số thực a, b khác 0 thỏa mãn: 1 1 1.



<i>a</i> + =<i>b</i>


1. Tính giá trị biểu thức

(

)



2


2 2


4 4


4
.


<i>a</i> <i>b</i>


<i>A</i>


<i>ab</i>
<i>a b</i>




= +


2. Chứng minh rằng:

(

) (

3

) (

3

)

3

(

)



2 1 1 3 6 0.


<i>a</i>+ −<i>b</i> − <i>a</i>− − <i>b</i>− − <i>a</i>+<i>b</i> + =



<b>Câu 2. (2,0 điểm) </b>


1. Giải phương trình: <i>x</i>+ −2 2 <i>x</i>− =1 3<i>x</i>−3

(

<i>x</i>+2

)(

<i>x</i>−1

)


<b>2.</b> Giải hệphương trình:


(

)

(

)



2


2 4 3


3 4 4 1 2 0


<i>y</i> <i>x</i> <i>y</i> <i>x</i>


<i>x</i> <i>y</i> <i>y</i> <i>x</i>


 <sub>+</sub> <sub>+ =</sub> <sub>+</sub>





− + + − − + =





<b>Câu 3. (3,5 điểm) </b>Cho hình vng <i>ABCD</i> nội tiếp đường trịn tâm <i>O</i>, bán kính <i>R</i>. Trên
cung nhỏ <i>AD </i>lấy điểm<i> E</i> bất kì (<i>E</i> khơng trùng với <i>A</i> và <i>D</i>). Tia<i> EB</i> cắt các đường thẳng
<i>AD, AC</i> lần lượt tại<i> I </i>và <i>K.</i> Tia <i>EC</i> cắt các đường thẳng <i>DA, DB</i> lần lượt tại<i> M</i> và <i>N</i>. Hai


đường thẳng <i>AN, DK</i> cắt nhau tại<i> P. </i>


1. Chứng minh: Tứ giác <i>EPND </i>nội tiếp một đường tròn
2. Chứ<sub>ng minh: </sub>∠<i>EKM</i> = ∠<i>DKM</i>.


<i>3.</i> Khi<i> M</i>là trung điểm của <i>AD, tính độ</i>dài đoạn thẳng <i>AE</i> theo <i>R. </i>
<b>Câu 4. (1,0 điểm) </b>


Tìm các nghiệm nguyên <i>(x, y)</i> của phương trình <i>x</i>+ <i>y</i> = 2020.
<b>Câu 5. </b><i>(1,5 điểm) </i>


1. Cho các số thực a, b, c thỏa mãn


1
0 , b, c


2


2 3 4 3


<i>a</i>


<i>a</i> <i>b</i> <i>c</i>


 < <




 <sub>+</sub> <sub>+</sub> <sub>=</sub>





. Tìm giá trị nhỏ nhất của


biểu thức:


(

3 24 2

)

(

4 98 3

)

(

2 83 1

)



<i>P</i>


<i>a</i> <i>b</i> <i>c</i> <i>b</i> <i>a</i> <i>c</i> <i>c</i> <i>a</i> <i>b</i>


= + +


+ − + − + −


2. Trong mặt phẳng tọa độ Oxy, điểm <i>M a b</i>

( )

; được gọi là điểm nguyên nếu cả<i>a </i> và <i>b</i>
đều là số nguyên. Chứng minh rằng tồn tại điểm <i>I</i> trong mặt phẳng tọa độ và 2019
số thực dương <i>R R</i>1; 2;....<i>R</i>2019sao cho có đúng kđiểm nguyên nằm trong đường tròn


(

<i>I R</i>; <i><sub>k</sub></i>

)

với mọi <i>k</i> là sốnguyên dương không vượt quá 2019.


<b>---Hết--- </b>


</div>
<span class='text_page_counter'>(4)</span><div class='page_container' data-page=4>

<b>SỞ GIÁO DỤC VÀ ĐÀO TẠO </b>


<b>THÁI BÌNH </b> <b>ĐỀTHI TUYỂN SINH LỚP 10 THPT CHUYÊN THÁI BÌNH<sub>NĂM HỌ</sub><sub>C 2018 – 2019 </sub></b>
<b>MƠNTHI:TỐN</b>



<b>(Dành cho tất cả các thí sinh) </b>


<i>Thời gian làm bài:<b> 120 phút</b> (Không kể thời gian giao đề) </i>
<b>Đề số 3 </b>


<b>Câu 1. </b>Cho biểu thức: 4 1 : 1

(

0; 1; 4

)



3 2 2 3 1




 


=<sub></sub> + <sub></sub> ≥ ≠ ≠


− + − +


 


<i>x</i>


<i>P</i> <i>x</i> <i>x</i> <i>x</i>


<i>x</i> <i>x</i> <i>x</i> <i>x</i>


1) Rút gọn biểu thức P.
2) Tìm x sao cho P = 2019.


3) Với x ≥ 5, tìm giá trị nhỏ nhất của <i>T</i> = +<i>P</i> 10.
<i>x</i>



<b>Câu 2. Cho hai đườ</b>ng thẳng (d1): y = mx + n và (d2): <i>y</i>= − 1 <i>x</i>+ 1


<i>m</i> <i>m</i> ( với m là tham số, m ≠


0). Gọi là tọa độgiao điểm của hai đường thẳng d1 và d2. Tính <i>T</i> =<i>x</i><sub>0</sub>2+ <i>y</i><sub>0</sub>2.


<b>Câu 3. </b>


Gọi x1, x2 là hai nghiệm phương trình: <i>x</i>2+ −

(

2 <i>m x</i>

)

− − =1 <i>m</i> 0

( )

1 (với m là tham số)
a) Tìm m để : <i>x</i>1−<i>x</i>2 =2 2


b) Tìm m để :


(

) (

2

)

2


1 2


1 1


1 1


= +


+ +


<i>T</i>


<i>x</i> <i>x</i> đạt giá trị nhỏ nhất



<b>Câu 4. </b>


a) Giải phương trình :

4

<i>x</i>

+

8072

+

9

<i>x</i>

+

18162

=

5

<sub>. </sub>
b) Giải hệphương trình


3 3 2


2 2


3

6

3

4

0



3

1



 −

+

+

+ =






+

=







<i>x</i>

<i>y</i>

<i>x</i>

<i>x</i>

<i>y</i>


<i>x</i>

<i>y</i>

<i>x</i>



<b>Câu 5: </b>Cho đường tròn O bán kính a và điểm J có JO = 2a. Các đường thẳng JM, JN theo
thứ tự là các tiếp tuyến tại M, tại N của đường tròn (O). Gọi K là trực tâm của tam giác
JMN, H là giao điểm của MN và JO.


a) Chứng minh rằng: H là trung điểm của OK.



b) Chứng minh rằng: K thuộc đường trịn tâm O bán kính a.
c) JO là tiếp tuyến của đường tròn tâm M bán kính r. Tính r.


d) Tìm tập hợp điểm I sao cho từ I kẻđược hai tiếp tuyến với đường trịn (O) và hai
tiếp tuyến đó vng góc với nhau.


<b>Câu 5:</b>Cho

<i>x y z</i>

, ,

là ba số thực khơng âm thỏa mãn :

12

<i>x</i>

+

10

<i>y</i>

+

15

<i>z</i>

60

.Tìm giá trị


lớn nhất của .


<b>---Hết--- </b>


Họ và tên ...Số báo danh ...


2 2 2


4

4



<i>T</i>

=

<i>x</i>

+

<i>y</i>

+

<i>z</i>

<i>x</i>

<i>y</i>

<i>z</i>



</div>
<span class='text_page_counter'>(5)</span><div class='page_container' data-page=5>

<b>SỞ GIÁO DỤC VÀ ĐÀO TẠO </b>


<b>THÁI BÌNH </b> <b>ĐỀTHI TUYỂN SINH LỚP 10 THPT CHUYÊN THÁI BÌNH<sub>NĂM HỌ</sub><sub>C 2018 – 2019 </sub></b>
<b>MƠNTHI:TỐN</b>


<b>(Dành cho học sinh chun tốn tin) </b>
<i>Thời gian làm bài:<b> 150 phút</b> (Không kể thời gian giao đề) </i>
<b>Đề số 4 </b>



<b>Câu 1. (2,0 diểm) </b>


1) Cho phương trình 2 2


2 2 4 0 (1)


<i>x</i> − <i>mx</i>+<i>m</i> − <i>m</i>+ = (với m là tham số). Tìm <i>m</i>để
phương trình (1) có hai nghiệm khơng âm

<i>x x</i>

<sub>1</sub>

;

<sub>2</sub>. Tính theo m giá trị của biểu thức


1 2


<i>P</i>= <i>x</i> + <i>x</i> và tìm giá trị nhỏ nhất của P


2) Cho hàm số 2 2.
2


<i>x</i>
<i>y</i>


<i>x</i>


+
=


+ Tìm tất cả các giá trị <i>x</i>nguyên.
<b>Câu 2 (2 điểm) </b>


1) Cho các số <i>a b c</i>; ; thỏa mãn điều kiện <i>a</i>+2<i>b</i>+5<i>c</i>=0.Chứng minh phương trình


2



0


<i>ax</i> +<i>bx</i>+ =<i>c</i> có nghiệm


2) Giải phương trình:

(

3

)

3 3 3


4 3 :


2


<i>x</i> − +<i>x</i> = <i>x</i>


<b>Câu 3. (1 điểm) </b>Hai cây nến cùng chiều dài và làm bằng các chất liệu khác nhau, cây nến
thứ nhất cháy hết với tốc độđều trong 3 giờ, cây nến thứ hai cháy hết với tốc độđều trong
4 giờ. Hỏi phải cùng bắt đầu đốt lúc mấy giờ chiều để 4 giờ chiều phần còn lại của cây nến
thứ hai dài gấp đơi phần cịn lại của cây nến thứ nhất?


<b>Câu 4. (1,0 điểm) </b> Cho biểu thức

(

2

)(

2

)



1 1 2018.


<i>x</i>+ +<i>x</i> <i>y</i>+ +<i>y</i> = Tìm giá trị nhỏ nhất


của biểu thức <i>P</i>= +<i>x</i> <i>y</i>
<b>Câu 5 (3,5 điểm) </b>


1) Cho tam giác <i>ABC</i>có <i>AB</i>=4,<i>AC</i> =3,<i>BC</i> =5, đường cao AH. Trên nửa mặt phẳng
bờ BC chứa điểm A vẽ hai nửa đường trịn đường kính BH và HC. Hai nửa đường
tròn này cắt AB, AC lần lượt tại E, F.



a) Tính diện tích nửa đường trịn đườn kính BH


b) Chứng minh tứ giác BEFC nội tiếp và đường thẳng EF là tiếp tuyến chung của
hai đường trịn đường kính BH và CH


2) Cho nửa đường trịn đường kính <i>AB</i>=2 .<i>R</i> Tìm kích thước hình chữ nhật MNPQ có


hai đỉnh M, N thuộc đường trịn , hai đỉnh P, Q thuộc đường kính AB sao cho điện
tích MNPQ lớn nhất


<b>Câu 6 (0,5 điểm) </b>Cho a,b,c là ba số thức dương thỏa mãn điều kiện : 1<sub>2</sub> 1<sub>2</sub> 1<sub>2</sub> 1


<i>a</i> +<i>b</i> +<i>c</i> =


Tìm giá trị lớn nhất của biểu thức


2 2 2 2 2 2


1 1 1


5 2 2 5 2 2 5 2 2


<i>P</i>


<i>a</i> <i>ab</i> <i>b</i> <i>b</i> <i>bc</i> <i>c</i> <i>c</i> <i>ca</i> <i>a</i>


= + +


+ + + + + +



<b>---Hết--- </b>


</div>
<span class='text_page_counter'>(6)</span><div class='page_container' data-page=6>

<b>SỞ GIÁO DỤC VÀ ĐÀO TẠO </b>


<b>THÁI BÌNH </b> <b>ĐỀTHI TUYỂN SINH LỚP 10 THPT CHUYÊN THÁI BÌNH<sub>NĂM HỌ</sub><sub>C 2017 – 2018 </sub></b>
<b>MƠNTHI:TỐN</b>


<b>(Dành cho mọi thí sinh) </b>


<i>Thời gian làm bài:<b> 150 phút</b> (Không kể thời gian giao đề) </i>
<b>Đề số 5 </b>


<b>Câu 1. (2.0 điểm). Cho </b>



2
1
1 3 5


1
1 <sub>1</sub> <sub>4</sub>


<i>x</i>
<i>x</i>


<i>A</i>


<i>x</i> <i><sub>x x</sub></i> <i><sub>x</sub></i> <i><sub>x</sub></i> <i><sub>x</sub></i>


 



 


 <sub></sub> <sub></sub> 


 <sub></sub> 




<sub></sub>  <sub> </sub><sub></sub>  <sub></sub>


  


     


 <sub> </sub> <sub></sub>


 


với <i>x</i> 0;<i>x</i> 1.
a) Rút gọn biểu thức A.


b) Đặt <i>B</i> 

<i>x</i>  <i>x</i> 1

<i>A</i>. Chứng minh rằng <i>B</i>1 với <i>x</i> 0;<i>x</i> 1.
<b>Câu 2. (2.0 điểm) </b>


Trong mặt phẳng tọa độ Oxy, cho Parabol (P): <i><sub>y</sub></i> <sub></sub><i><sub>x</sub></i>2 <sub>và đường thẳng (d): </sub>


2 2 8


<i>y</i>  <i>x</i>  <i>m</i> (với m là tham số).



a) Khi m = – 4, tìm tọa độ giao điểm của đường thẳng (d) và Parabol (P) .


b) Chứng minh rằng đường thẳng (d) và Parabol (P) luôn cắt nhau tại hai điểm phân
biệt có hồnh độ <i>x x</i><sub>1</sub>; <sub>2</sub>. Tìm các giá trị m thỏa mãn điều kiện x<sub>1</sub> 2x<sub>2</sub> 2.


<b>Câu 3. (1.0 điểm) </b>Giải hệ phương trình


2 2 <sub>2</sub> 2 <sub>3</sub>


4 1 0


<i>xy</i> <i>y</i> <i>x</i> <i>x</i>


<i>x</i> <i>y</i> <i>y</i>


 <sub></sub> <sub> </sub> <sub></sub>



    



<b>Câu 4. (1.0 điểm) </b>Cho quãng đường AB dài 300km. Cùng một lúc xe ô tô thứ nhất xuất
phát từ A đến B, xe ô tô thứ hai đi từ B về A. Sau khi xuất phát được 3 giờ thì hai xe gặp
nhau. Tính vận tốc của mỗi xe, biết thời gian đi cả quãng đường AB của xe thứ nhất nhiều
hơn xe thứ hai là 2 giờ 30 phút.


<b>Câu 5. (3.5 điểm) </b>Cho đường tròn

 

<i>O R</i>; có đường kính AB. Điểm C là điểm bất kỳ trên


 

<i>O</i> , C không trùng với A, B. Tiếp tuyến tại C của

 

<i>O R</i>; cắt tiếp tuyến tại A, B của

 

<i>O R</i>;

lần lượt tại P, Q. Gọi M là giao điểm của OP với AC, N là giao điểm của OQ với BC.


a) Chứng minh rằng tứ giác CMON là hình chữ nhật và <sub>AP.BQ</sub><sub></sub> <sub>MN</sub>2<sub>. </sub>
b) Chứng minh rằng AB là tiếp tuyến của đường trịn đường kính PQ.


c) Chứng minh rằng PMNQ là tứ giác nội tiếp. Xác định vị trí điểm C để đường trịn
ngoại tiếp tứ giác PMNQ có bán kính nhỏ nhất.


<b>Câu 6. (0.5 điểm) Cho ba số thực dương x, y, z thỏa mãn </b><sub>8</sub><i><sub>t</sub></i>2 <sub></sub><i><sub>mt</sub></i><sub> </sub><sub>1</sub> <sub>0</sub><sub>. Tìm giá trị nhỏ </sub>
nhất của biểu thức:


 

 



2 2 2 2 2 2


2 2 2 2 2 2


<i>y z</i> <i>z x</i> <i>x y</i>


<i>P</i>


<i>x y</i> <i>z</i> <i>y z</i> <i>x</i> <i>z x</i> <i>y</i>


  


   .


<b>---Hết--- </b>


</div>
<span class='text_page_counter'>(7)</span><div class='page_container' data-page=7>

<b>SỞ GIÁO DỤC VÀ ĐÀO TẠO </b>



<b>THÁI BÌNH </b> <b>ĐỀTHI TUYỂN SINH LỚP 10 THPT CHUN THÁI BÌNH<sub>NĂM HỌ</sub><sub>C 2017 – 2018 </sub></b>
<b>MƠNTHI:TỐN</b>


<b>(Dành cho học sinh chuyên toán tin) </b>
<i>Thời gian làm bài:<b> 150 phút</b> (Không kể thời gian giao đề) </i>
<b>Đề số 6 </b>


<b>Câu 1(2.0 điểm).</b>


1) Cho a, b là hai số thực bất kỳ. Chứng minh có ít nhất một trong hai phương trình
ẩn x sau vơ nghiệm:


2 2 2


2 2


2 2 1 0
2 3 0


<i>x</i> <i>ax</i> <i>a</i> <i>b</i>


<i>x</i> <i>bx</i> <i>b</i> <i>ab</i>


    


   


2) Cho các số thực x, y, z thỏa mãn các điều kiện <i>x</i>   <i>y</i> <i>z</i> 0và <sub>x</sub>2
Tính giá trị biểu thức: <i>P</i> <sub>2</sub> <i>x</i><sub>2</sub>2 <sub>2</sub> <sub>2</sub> <i>y</i>2<sub>2</sub> <sub>2</sub> <sub>2</sub> <i>z</i>2<sub>2</sub> <sub>2</sub>



<i>y</i> <i>z</i> <i>x</i> <i>z</i> <i>x</i> <i>y</i> <i>x</i> <i>y</i> <i>z</i>


  


     


<b>Câu 2(2.5 điểm). </b>


1) Giải phương trình 2
2


1 1


4x 2 2x 20 0
x


x


 <sub></sub>  <sub></sub>


 <sub></sub> <sub></sub> <sub></sub>  <sub></sub> <sub></sub><sub></sub> <sub></sub>


 <sub></sub>  <sub></sub>


 <sub></sub>  <sub></sub>


 


   



2) Giải hệ phương trình



2 2


2 2


2


4 1 4 4


3 2 1 2 3


<i>x</i> <i>y</i> <i>xy</i> <i>xy</i>


<i>x</i> <i>y</i>


<i>x</i> <i>y</i> <i>y</i> <i>y</i> <i>y</i> <i>x</i>


  


 <sub></sub>


 <sub></sub> <sub></sub>  <sub> </sub><sub></sub> <sub></sub>


 <sub></sub> <sub></sub>


 <sub> </sub><sub></sub> <sub></sub>






 <sub> </sub> <sub>  </sub> <sub> </sub>






<b>Câu 3(1.0 điểm). </b>


Tìm tất cả các cặp số nguyên

 

<i>x y</i>; thỏa mãn phương trình <i><sub>x</sub></i>3 <sub></sub><i><sub>y</sub></i>3 <sub></sub><sub>6</sub><i><sub>xy</sub></i> <sub></sub><sub>3</sub>


<b>Câu 4(3.0 điểm). Cho tứ giác ABCD nội tiếp đường trịn tâm O và có hai tia BA và CD cắt </b>
nhau tại E, hai tia AD và BC cắt nhau tại F. Gọi M, N lần lượt là trung điểm của AC và BD.
Các đường phân giác trong của các góc<i>BEC</i>và góc<i>BFA</i>cắt nhau tại K.


a) Chứng minh rằng DEF<i>DFE</i> <i>ABC</i> và tam giác EKF là tam giác vuông
b) Chứng minh rằng EM.BDEN.AC


c) Chứng minh rằng ba điểm K, M, N thẳng hàng
<b>Câu 5(1.5 điểm).</b>


1) Cho các số thực dương a, b, c bất kì. Chứng minh rằng:
1 1 1 3
3 2 3 2 3 2 5


<i>a</i> <i>a</i>  <i>b</i> <i>b b</i> <i>c</i> <i>c c</i>  <i>a</i>  <i>abc</i>


2) Cho 5 số tự nhiên phân biệt sao cho tổng của ba số bất kỳ trong chúng lớn hơn tổng
của hai số còn lại. Chứng minh rằng tất cả 5 số đã cho đều không nhỏ hơn 5


<b>---Hết--- </b>



</div>
<span class='text_page_counter'>(8)</span><div class='page_container' data-page=8>

<b>SỞ GIÁO DỤC VÀ ĐÀO TẠO </b>


<b>THÁI BÌNH </b> <b>ĐỀTHI TUYỂN SINH LỚP 10 THPT CHUYÊN THÁI BÌNH<sub>NĂM HỌ</sub><sub>C 2016 – 2017 </sub></b>
<b>MƠNTHI:TỐN</b>


<b>(Dành cho học sinh chun tốn tin) </b>
<i>Thời gian làm bài:<b> 150 phút</b> (Không kể thời gian giao đề) </i>
<b>Đề số 7 </b>


<b>Câu 1 (2,0 điểm) </b>


1) Giải phương trình 5<i>x</i>−

(

<i>x</i>+3

)

2<i>x</i>− − =1 1 0


2) Cho hai số thực a, b bất kì. Chứng minh rằng ít nhất một trong hai phương


trình sau có nghiệm: 2

( )

2

( )



2 3 0 1 2 8 0 2


<i>x</i> + <i>ax</i>+ <i>ab</i>= <i>x</i> + <i>bx</i>− <i>ab</i>=


<b>Câu 2 (2,5 điểm) </b>


1) Tìm các số nguyên x, y thỏa mãn: 2 2


9<i>x</i> +3<i>y</i> +6<i>xy</i>−6<i>x</i>+2<i>y</i>−35=0


2) Cho P(x) là đa thức bậc ba có hệ số bậc cao nhất nhất bằng 1 và thỏa mãn:
P(2016) = 2017, P(2017) = 2018. Tính giá trị của -3P(2018) + P(2019)



<b>Câu 3 (1,5 điểm) </b>


Giải hệphương trình:

(

)



3 4 4


2 2


8 2 2 2 3


2 2 2 9 2 19


<i>y</i> <i>x</i> <i>y</i> <i>x</i>


<i>x</i> <i>x</i> <i>y</i> <i>x</i> <i>y</i> <i>x</i> <i>x</i> <i>y</i>


 <sub>+</sub> <sub>−</sub> <sub>=</sub> <sub>+</sub>





 + + + + = − +



<b>Câu 4 (3,0 điểm) </b>


Từ một điểm I nằm bên ngoài đường tròn (O), vẽ các tiếp tuyến IA, IB (A, B
là các tiếp điểm) và vẽ tiếp tuyến ICD (khơng qua tâm O) với đường trịn (C nằm
giữa I và D).



1. Chứng minh rằng: AC.BD = AD.BC


2. Gọi K là giao điểm của CD và AB, E là trung điểm của OI.
Chứng minh rằng KA.KB = OE2 – EK2.


3. Gọi H là trung điểm của AB. Chứng minh: ∠<i>ADH</i> = ∠<i>IDB</i>


<b>Câu 5 (1,0 điểm) </b>


Cho các số thực <i>x</i>≥1,<i>y</i>≥1,<i>z</i>≥1và thỏa mãn 3<i>x</i>2 +4<i>y</i>2 +5<i>z</i>2 =52 . Tìm giá
trị nhỏ nhất của biểu thức F = x + y + z.


<b>---Hết--- </b>


</div>
<span class='text_page_counter'>(9)</span><div class='page_container' data-page=9>

<b>SỞ GIÁO DỤC VÀ ĐÀO TẠO </b>


<b>THÁI BÌNH </b> <b>ĐỀTHI TUYỂN SINH LỚP 10 THPT CHUYÊN THÁI BÌNH<sub>NĂM HỌ</sub><sub>C 2015 – 2016 </sub></b>
<b>MƠNTHI:TỐN</b>


<b>(Dành cho tất cả các thí sinh) </b>


<i>Thời gian làm bài:<b> 120 phút</b> (Không kể thời gian giao đề) </i>
<b>Đề số 8 </b>


<b>Bài 1 </b><i>(3,0 điểm).</i>


Cho biểu thức:


<i>x</i>



<i>x</i>


<i>x</i>



<i>x</i>


<i>x</i>



<i>x</i>


<i>x</i>



<i>x</i>


<i>x</i>


<i>x</i>


<i>x</i>


<i>P</i>



+


+






+



+



=

2

2

1

2

(

<i>x</i>

>

0

;

<i>x</i>

1

)

.


a) Rút gọn biểu thức <i>P</i>.


b) Tính giá trị của thức <i>P</i> khi

<i>x</i>

=

3

2

2




c) Chứng minh rằng: với mọi giá trị của <i>x</i>để biểu thức <i>P</i>có nghĩa thì biểu thức
<i>P</i>
7
chỉ nhận một giá trị nguyên.


<b>Bài 2 </b><i>(2,0 điểm).</i>


Cho phương trình x<i>2 – 2mx + (m – 1)3</i> = 0 (m là tham số).


a) Giải phương trình khi m = –1.


b) Tìm m đểphương trình có hai nghiệm phân biệt trong đó có một nghiệm bằng
bình phương nghiệm cịn lại.


<b>Bài 3 </b><i>(1,0 điểm).</i>


Giải phương trình:

1

0

.


9



2


2


9



2


2

=



+


+




<i>x</i>


<i>x</i>


<i>x</i>



<b>Bài 4 </b><i>(3,5 điểm).</i> Cho tam giác ABC vuông tại A, đường cao AH. Đường trịn đường kính
AH, tâm O, cắt các cạnh AB và AC lần lượt tại E và F. Gọi M là trung điểm của cạnh HC.


a) Chứng minh AE.AB = AF.AC.


b) Chứng minh rằng MF là tiếp tuyến của đường tròn đường kính AH.
c) Chứng minh  <i>HAM</i> =<i>HBO</i>


d) Xác định điểm trực tâm của tam giác ABM.


<b>Bài 5 </b><i>(0,5 điểm). </i>Cho các sốdương a, b, c thỏa mãn <i>ab + bc + ca</i> = 3. Chứng minh rằng:


2


3


1


1


1


1


1


1



2
2


2

+

+

<i><sub>b</sub></i>

+

+

<i><sub>c</sub></i>

+



<i>a</i>



<b>---Hết--- </b>


</div>
<span class='text_page_counter'>(10)</span><div class='page_container' data-page=10>

<b>SỞ GIÁO DỤC VÀ ĐÀO TẠO </b>


<b>THÁI BÌNH </b> <b>ĐỀTHI TUYỂN SINH LỚP 10 THPT CHUYÊN THÁI BÌNH<sub>NĂM HỌ</sub><sub>C 2015 – 2016 </sub></b>
<b>MƠNTHI:TỐN</b>


<b>(Dành cho học sinh chun tốn, tin) </b>
<i>Thời gian làm bài:<b> 120 phút</b> (Không kể thời gian giao đề) </i>
<b>Đề số 9 </b>


<b>Bài 1 (1,5 điểm) Cho phương trình: </b> 2


2<i>x</i> −<i>mx</i>− =1 0 (với <i>m</i> là tham số).
a) Tìm m sao cho phương trình có hai nghiệm x1, x2 thỏa mãn <i>x</i><sub>1</sub>2 −4<i>x</i><sub>2</sub>2 =0
b) Chứng minh rằng với mọi <i>m</i>phương trình trên có nghiệm x thỏa mãn <i>x</i> <1
<b>Bài 2 </b><i>(2,0 điểm) </i>


a) Giải phương trình: 2 17 1


18 2 9 0.


3 3


<i>x</i> − <i>x</i>− + <i>x</i>− =


b) Tìm các số nguyên x, y với <i>x y</i>, ≥0thỏa mãn:



2 2


3 4 4 10 12 0.


<i>x</i> + <i>y</i> + <i>xy</i>+ <i>x</i>+ <i>y</i>− =


<b>Bài 3 </b><i>(1,0 điểm) </i>Giải hệphương trình:

(

)

(

)



2
2


1 1 4 3


4 2 1


<i>x</i> <i>y</i> <i>x</i> <i>y</i> <i>x</i> <i>y</i>


<i>x</i> <i>xy</i>


 <sub>− + + =</sub> <sub>−</sub> <sub>+</sub> <sub>−</sub>





+ =



<b>Bài 4. (</b><i>1,0 điểm)</i> Cho x, y thỏa mãn 2 2


4 2 0



<i>x</i> +<i>y</i> − <i>x</i>− = . Chứng minh:




2 2


10 4 6− ≤<i>x</i> + <i>y</i> ≤4 6 10.+


<b>Bài 5 </b><i>(2,5 điểm) </i>Cho tam giác ABC nhọn, nội tiếp đường tròn (O). Đường thẳng AO
cắt đường tròn (O) tại M (M khác A). Đường thẳng qua C vng góc với AB cắt
đường trịn (O) tại N (N khác C). Gọi K là giao điểm MN với BC.


a) Chứng minh tam giác KCN cân.
b) Chứng minh OK vng góc BM.


c) Khi tam giác ABC cân tại A, hai tiếp tuyến của đường tròn (O) tại M và N cắt
nhau tại P. Chứng minh ba điểm P, B, O thẳng hàng.


<b>Bài 6. </b><i>(1,0 điểm) </i>Cho tam giác ABC có độ dài AB = 3a, AC = 4a và góc ∠<i>BAC</i> =60<i>o</i> .
Qua A kẻ AH vng góc với BC tại H. Tính độdài đoạn AH theo a.


<b>Bài 7. </b><i>(1,0 điểm) </i> Cho ba sốdương a, b, c thỏa mãn abc = 1. Chứng minh:


(

)



2 2 2


9 9



.


2 2


<i>b</i> <i>c</i> <i>a</i>


<i>a</i> + <i>b</i> + <i>c</i> + <i>ab bc</i>+ +<i>ca</i> ≥


<b>---Hết--- </b>


</div>
<span class='text_page_counter'>(11)</span><div class='page_container' data-page=11>

<b>SỞ GIÁO DỤC VÀ ĐÀO TẠO </b>


<b>THÁI BÌNH </b> <b>ĐỀTHI TUYỂN SINH LỚP 10 THPT CHUYÊN THÁI BÌNH<sub>NĂM HỌ</sub><sub>C 2014 – 2015 </sub></b>
<b>MƠNTHI:TỐN</b>


<b>(Dành cho tất cả các thí sinh) </b>


<i>Thời gian làm bài:<b> 120 phút</b> (Khơng kể thời gian giao đề) </i>
<b>Đề số 10 </b>


<b>Bài 1. </b><i>(2,0 điểm) </i>Cho biểu thức 2 3 5 7 : 2 3
2 2 1 2 3 2 5 10


<i>x</i> <i>x</i>


<i>A</i>


<i>x</i> <i>x</i> <i>x</i> <i>x</i> <i>x</i> <i>x</i>


 <sub>−</sub>  <sub>+</sub>



=<sub></sub><sub></sub> + − <sub></sub><sub></sub>


− + − − −


  (x > 0, x ≠ 4)


1, Rút gọn biểu thức A.


2, Tìm x sao cho A nhận giá trị là một số nguyên.


<b>Bài 2. </b><i>(2, 5 điểm) </i>Cho parabol (P): y = x2và đường thẳng (d) : y = 2(m + 3)x – 2m + 2 ( m là
tham số, m ∈ℝ).


1, Với m = –5 tìm tọa độgiao điểm của parabol (P) và đường thẳng (d).


2, Chứng minh rằng: với mọi m parabol (P) và đường thẳng (d) cắt nhau tại hai điểm phân
biệt.


Tìm m sao cho hai giao điểm đó có hồnh độdương.


3, Tìm điểm cốđịnh mà đường thẳng (d) luôn đi qua với mọi m
<b>Bài 3. </b><i>(1,5 điểm) </i>Giải hệphương trình: 2<sub>2</sub>2 3 2<sub>2</sub>2 5(2 ) 0


2 3 15 0


<i>x</i> <i>xy</i> <i>y</i> <i>x</i> <i>y</i>


<i>x</i> <i>xy</i> <i>y</i>



 + − − − =





− − + =





<b>Bài 4. </b><i>(3,5 điểm) </i>Cho tam giác nhọn ABC nội tiếp đường tròn (O; R). Tiếp tuyến tại B và C
của đường tròn (O; R) cắt nhau tại T, đường thẳng AT cắt đường tròn tại điểm thứhai là D
khác A.


1, Chứng minh rằng tam giác ABT đồng dạng với tam giác BDT.
2, Chứng minh rằng: AB.CD = BD.AC


3, Chứng minh rằng hai đường phân giác góc BAC , góc BDC và đường thẳng BC đồng
quy tai một điểm.


4, Gọi M là trung điểm của BC, chứng minh rằng góc BAD bằng góc MAC.


<b>Bài 5. </b>(0,5 điểm) Cho các sốdương x, y, z thay đổi thỏa mãn: x( x + 1) + y( y + 1) + z( z + 1)
≤ 18. Tìm giá trị nhỏ nhất của biểu thức: 1 1 1


1 1 1


<i>B</i>


<i>x</i> <i>y</i> <i>y</i> <i>z</i> <i>z</i> <i>x</i>



= + +


+ + + + + +


<b>---Hết--- </b>


</div>
<span class='text_page_counter'>(12)</span><div class='page_container' data-page=12>

<b>SỞ GIÁO DỤC VÀ ĐÀO TẠO </b>


<b>THÁI BÌNH </b> <b>ĐỀTHI TUYỂN SINH LỚP 10 THPT CHUYÊN THÁI BÌNH<sub>NĂM HỌ</sub><sub>C 2014 – 2015 </sub></b>
<b>MƠNTHI:TỐN</b>


<b>(Dành cho chun tốn, tin) </b>


<i>Thời gian làm bài:<b> 150 phút</b> (Không kể thời gian giao đề) </i>
<b>Đề số 11 </b>


<b>Bài 1. </b><i>(3,0 điểm) </i>


1) Giải phương trình: 2


5<i>x</i>− +6 10 3− <i>x</i> =2<i>x</i> − −<i>x</i> 2
2) Giải hệphương trình:


3 2


2 2


8 96
32 48



<i>x</i> <i>xy</i> <i>y</i>


<i>x</i> <i>y</i>


 + =





+ =



<b>Bài 2. </b><i>(2,0 điểm) </i>


1) Cho phương trình x2 – 2x – 4 = 0 có hai nghiệm x<sub>1</sub>; x<sub>2</sub>. Tính 7 7


1 2


<i>S</i> =<i>x</i> +<i>x</i>


2) Cho a, b, c, d là các sốnguyên dương thỏa mãn: a2<sub> + ab + b</sub>2 <sub>= c</sub>2<sub> + cd + d</sub>2<sub>. Ch</sub><sub>ứ</sub><sub>ng </sub>
minh a + b + c + d là hợp số.


<b>Bài 3. </b><i>(1,0 điểm) </i>


Cho a, b, c là ba số thực dương và có tổng bằng 1.


Chứng minh: 3


2



<i>a bc</i> <i>b ca</i> <i>c</i> <i>ab</i>


<i>a</i> <i>bc</i> <i>b</i> <i>ca</i> <i>c</i> <i>ab</i>


− <sub>+</sub> − <sub>+</sub> − <sub>≤</sub>


+ + +


<b>Bài 4. </b><i>(3,0 điểm) </i>


Cho hình bình hành ABCD với A, C cốđịnhvà B, D di động. Đường phân giác của góc
BCD cắt AB và AD theo thứ tự tại I và J (J nằm giữa A và D). Gọi M là giao điểm khác A
của hai đường tròn ngoại tiếp tam giác ABD và AIJ, O là tâm đường tròn ngoại tiếp tam
giác AIJ.


1) Chứng minh AO là phân giác góc IAJ.


2) Chứng minh bốn điểm A, B, D, O cùng thuộc một đường trịn.
3) Tìm đường trịn cốđịnh ln đi qua M khi B, D di động.
<b>Bài 5. </b><i>(1,0 điểm) </i>


Chứng minh rằng trong 39 số tự nhiên liên tiếp bất kỳ ln tồn tại ít nhất một số có tổng
các chữ số chi hết cho 11


<b>---Hết--- </b>


</div>
<span class='text_page_counter'>(13)</span><div class='page_container' data-page=13>

<b>SỞ GIÁO DỤC VÀ ĐÀO TẠO </b>


<b>THÁI BÌNH </b> <b>ĐỀTHI TUYỂN SINH LỚP 10 THPT CHUYÊN THÁI BÌNH<sub>NĂM HỌ</sub><sub>C 2013 – 2014 </sub></b>


<b>MƠNTHI:TỐN</b>


<b>(Dành cho chun tốn, tin) </b>


<i>Thời gian làm bài:<b> 150 phút</b> (Không kể thời gian giao đề) </i>
<b>Đề số 12 </b>


<b>Bài 1</b>.(2 điểm) Cho biểu thức 1 1 ( 4)
4


2 2


<i>x</i>


<i>P</i> <i>x</i>


<i>x</i>


<i>x</i> <i>x</i>


 


=<sub></sub> + + <sub></sub> −




+ −


  với <i>x</i>≥0;<i>x</i>≠4



1) Rút gọn biểu thưc P


2) Tìm giá trị nhỏ nhất của P.


<b>Bài 2.</b>(2 điểm) Cho hệphương trình : 1(1)


6(2)
<i>mx</i> <i>y</i>


<i>x</i> <i>my</i> <i>m</i>
− =


 + = +


 (với m là tham số)
1) giải hệphương trình với m=1


2) Tìm m để hệ có nghiệm (x;y) thoả mãn 3<i>x</i>− =<i>y</i> 1


<b>Bài 3 </b>(2 điểm)


1) Cho phương trình bậc hai : 2 2


(2 1) 6 0


<i>x</i> − <i>m</i>− <i>x</i>+<i>m</i> − − =<i>m</i> (với m là tham số). Chứng


minh phương trình ln có 2 nghiệm phân biệt <i>x x</i>1; 2với mọi giá trị của m. Tìm m để :



1 2


5 <i>x</i> <i>x</i> 5
− < < <


2) Giải phương trình: 2 2


(<i>x</i>+2)(<i>x</i>−3)(<i>x</i> +2<i>x</i>−24) 16= <i>x</i>


<b>Bài 4.</b>(3.5 điểm) Cho tam giác đều ABC có đường cao AH. Trên đường thẳng BC lấy điểm M nằm


ngoài đoạn BC sao cho MB >MC và hình chiếu vng góc của M trên AB là điểm P ( P giữa A và
B). Kẻ MQ vuông góc với đường thẳng AC tại Q.


1.Chứng minh bốn điểm A,P,Q,M cùng nằm trên một đường tròn. Xác định tâm O của


đường trịn đó.


2.Chứng minh BA.BP = BM.BH
3. Chứng minh OH vng góc với PQ
4. Chứng minh PQ >AH


<b>Bài 5</b>. (0.5 điểm) Giải phương trình: 3


3


2 2


2013 1 2013 1



2 2014 2013 1


2 2


<i>x</i> <i>x</i>


<i>x</i> <i>x</i> <i>x</i>


<i>x</i> <i>x</i>


− −


+ − − = + − +


− −


<b>---Hết--- </b>


</div>
<span class='text_page_counter'>(14)</span><div class='page_container' data-page=14>

<b>SỞ GIÁO DỤC VÀ ĐÀO TẠO </b>


<b>THÁI BÌNH </b> <b>ĐỀTHI TUYỂN SINH LỚ<sub>NĂM HỌ</sub>P 10 THPT CHUN THÁI BÌNH<sub>C 2012 – 2013 </sub></b>
<b>MƠNTHI:TỐN</b>


<b>(Dành cho chuyên toán, tin) </b>


<i>Thời gian làm bài:<b> 150 phút</b> (Không kể thời gian giao đề) </i>
<b>Đề số 13 </b>


<b>Câu 1. (2,5 điểm) </b>



a) Tính <i>A</i>=

(

4+ 15

)(

10− 6

)

4− 15
b) Giải hệphương trình

<sub>(</sub>

<sub>)</sub>

(

)



2 2


1 2


2 2 1


<i>x</i> <i>y</i> <i>x</i> <i>y</i>


<i>y</i> <i>x</i> <i>y</i> <i>y</i>


 + + = +




 <sub>−</sub> <sub>=</sub> <sub>+</sub>



<b>Câu 2. (1,5 điểm) </b>


Cho a, b, c là các sốdương thỏa mãn a + b + c = 3. Tìm giá trị nhỏ nhất của
biểu thức:


3 3 3


<i>a</i> <i>b</i> <i>c</i>


<i>P</i>



<i>b</i> <i>a</i> <i>c</i> <i>b</i> <i>a</i> <i>c</i>


= + +


+ − + − + −


<b>Câu 3. (2,0 điểm) </b>Cho <i>m, n</i> là hai sốnguyên dương thỏa mãn <i>m + n – 1 </i> là số nguyên
tố và <i> m + n – 1 </i>là ước của

(

2 2

)



2 <i>m</i> +<i>n</i> −1


Chứng minh <i> m = n. </i>


<b>Câu 4. (3,0 điểm) </b> Cho tam giác ABC có ba góc nhọn nội tiếp (O). Đường trịn tâm J đường
kính BC cắt AB, AC lần lượt tại E, F. Gọi H, K lần lượt là trực tâm của tam giác ABC và
AEF. Gọi I là tâm đường tròn ngoại tiếp tam giác AEF.


a) Tiếp tuyến tại A của (O) song song với EF
b) Ba điểm A, I, H thẳng hàng.


c) KH,EF, IJ đồng quy


<b>Câu 5: </b><i>(1,0 điểm) </i>Cho nửa đường trịn tâm O đường kính AB, CD là một dây cung của
đường tròn (A, B, C, D là bốn điểm phân biệt). M là điểm bất kì di động trên công nhỏCD,
gọi I, J lần lượt là giao điểm của MA, MB với dây cung CD.


Xác định vị trí của điểm M đểđoạn IJ có độ dài lớn nhất


<b>---Hết--- </b>



</div>
<span class='text_page_counter'>(15)</span><div class='page_container' data-page=15>

<b>HƯỚNG DẪN GIẢ</b>

<b>I </b>



<b>Đề</b>

<b> s</b>

<b>ố</b>

<b> 1 </b>



<b>Câu 1:</b>


<b>a)</b> Ta có:


(

)



(

)(

)



(

)

2


2 2 1


. .


1
.


<i>xy x</i> <i>y</i> <i>xy</i>


<i>xy</i> <i>x</i> <i>y</i> <i>xy</i> <i>x</i> <i>y</i>


<i>P</i>


<i>xy</i> <i>x</i> <i>y</i> <i>x</i> <i>y</i> <i>xy</i> <i>xy</i> <i>x</i> <i>y</i>



<i>x</i> <i>y</i> <i><sub>x</sub></i> <i><sub>y</sub></i>


<i>xy</i> <i>x</i> <i>y</i> <i>xy</i>


+ −


+ + + +


= =


+


+ + −


+ <sub>+</sub>


= =


+


<b>b)</b> Với x > 0; y > 0 xy = 16 ta có:


2


1


4 4 2


<i>x</i> <i>y</i> <i>xy</i>



<i>x</i> <i>y</i>


<i>P</i>= + ≥ = =


Vậy giá trị nhỏ nhất của P là 1 khi x = y = 4.
<b>Câu 2:</b>


Gọi số học sinh lớp 9A là x (0 < x < 90)
Suy ra số học sinh lớp 9B là 90 – x


Theo bài ra ta có: 5x + 6.(90 – x) = 493 nên x = 47


Kết luận: số hoc sinh lớp 9A là 47, số học sinh lớp 9B là 43.
<b>Câu 3. </b>


<b>1)</b> Điều kiện (d1) //(d2) là


2 2 2


1 3 2 0


1
1


2 2 2


2
<i>m</i>


<i>m</i> <i>m</i> <i>m</i> <i>m</i>



<i>m</i>
<i>m</i>


<i>m</i> <i>m</i> <i>m</i>


<i>m</i>


 =


 + = +  − − = <sub></sub>


⇔ ⇔ = − ⇔ = −


<sub>−</sub> <sub>≠ − −</sub>  <sub>≠</sub> 


   <sub>≠</sub>




<b>2)</b> Gọi <i>M x y</i>

(

0; 0

)

là điểm cốđịnh mà đường thẳng (d2) luôn đi qua


(

)



0 3 0 2


<i>m</i> <i>R</i> <i>y</i> <i>m</i> <i>x</i> <i>m</i>


∀ ∈ ⇒ = + − − đúng ∀ ∈<i>m</i> <i>R</i>



(

0 1

)

3 0 2 0 0


<i>m x</i> <i>x</i> <i>y</i>


⇔ − + − − = đúng ∀ ∈<i>m</i> <i>R</i>


0 0


0 0 0


1 0 1


3 2 0 1


<i>x</i> <i>x</i>


<i>x</i> <i>y</i> <i>y</i>


 − =  =




⇔<sub></sub> ⇔<sub></sub>


− − = =


 





Vậy với mọi <i>m</i>đường thẳng (d2) luôn đi qua M(1;1) cốđịnh.
<b>3)</b> Phương trình hồnh độgiao điểm (d1) và (d2):


(

2

)

(

)

(

)(

)



1 2 3 2 1 2 2


<i>m</i> + <i>x</i>− <i>m</i>= <i>m</i>+ <i>x</i>− − ⇔<i>m</i> <i>m</i>+ <i>m</i>− <i>x</i>= −<i>m</i>


Để (d1), (d2) cắt nhau tại M(xM, yM) thì 1


2
<i>m</i>


<i>m</i>
≠ −


⇔  <sub>≠</sub>


</div>
<span class='text_page_counter'>(16)</span><div class='page_container' data-page=16>

Khi đó: 1

(

3

)

1 2 2 2 1


1 1 1


<i>M</i> <i>M</i>


<i>m</i> <i>m</i>


<i>x</i> <i>y</i> <i>m</i> <i>m</i>



<i>m</i> <i>m</i> <i>m</i>


− − +


= ⇒ = + − − =


+ + +


(

)



(

)

(

)



(

)

(

)



(

)

(

)



2 2


2 2


2 2


2 2


3 2 6


2020 2 2020 1010.


1 1



2 1 4 1 4 4 4 2


1010. 1010. 2 1010 1 3 3030


1 1


1 1


<i>M</i> <i>M</i>


<i>m</i> <i>m</i>


<i>A</i> <i>x</i> <i>y</i>


<i>m</i> <i>m</i>


<i>m</i> <i>m</i>


<i>m</i> <i>m</i>


<i>m</i> <i>m</i>


− − +


= + = =


+ +


   



− + + + + <sub></sub> <sub></sub>


= = − + + = <sub></sub> + <sub></sub> − ≥ −


+  + 


+ <sub></sub> + <sub></sub> <sub></sub> <sub></sub>


Vậy A đạt giá trị nhỏ nhất là -3030 khi m = -3
<b>Câu 4. </b>


Điều kiện: <i>y</i>≥ −4


Từ

( )

(

2 2

)

(

)

2 2 0


1 1 0


1 0


<i>x</i> <i>y</i>


<i>x</i> <i>y</i> <i>x</i> <i>y</i>


<i>x</i> <i>y</i>


 + =


⇒ + <sub>− − = ⇔ </sub>


− − =




Trường hợp 1: 2 2


0 0


<i>x</i> +<i>y</i> = ⇔ = =<i>x</i> <i>y</i>


Trường hợp 2: <i>x</i>− − = ⇒ = −<i>y</i> 1 0 <i>y</i> <i>x</i> 1


Thay vào (2) ta có: 2

(

)



4 3 3 6 3


<i>x</i> + <i>x</i>+ = <i>x</i>+ <i>x</i>≥ −


(

)

(

)



2


2
2


2 1 3 4 3 4


1 3 2


<i>x</i> <i>x</i> <i>x</i> <i>x</i>


<i>x</i> <i>x</i>



⇔ − + = + − + +


⇔ − = + −


(

)

(

)



(

)

(

)



(

)

( )



2
2


2


2


1


1 3 2 0 3


3 2


1


3 2 1 3 2 2


<i>x</i>



<i>x</i> <i>x</i> <i>x</i>


<i>x</i>
<i>x</i>


<i>x</i> <i>L do x</i>




⇔ − = + + > ∀ ≥ −


+ +
=





 + + = + + ≥




</div>
<span class='text_page_counter'>(17)</span><div class='page_container' data-page=17>

<b>1)</b> Ta có 0
90
<i>ACD</i>


∠ = (góc nội tiếp chắn nửđường trịn)
Vì <i>IM //CD</i> nên ∠<i>AMI</i> = ∠<i>ACD</i>=90<i>o</i>


Nên ∠<i>AMI</i>+ ∠<i>AHI</i> =180<i>o</i> ⇒tứ giác <i>AHIM </i> nội tiếp


2) Xét hai tam giác <i>AHB</i> và <i>ACD</i> có


90<i>o</i>


<i>AHB</i> <i>ACD</i>


∠ = ∠ =


<i>ABH</i> <i>ADC</i>


∠ = ∠ (2 góc nội tiếp cùng chắn một cung)
Suy ra hai tam giác <i>AHB</i> và <i>ACD </i>đồng dạng.


. .


<i>AB</i> <i>HA</i>


<i>AB AC</i> <i>AH AD</i>


<i>AD</i> <i>CA</i>


⇒ = ⇒ =


3) Gọi đường tròn <i>O’</i> ngoại tiếp tam giác <i>ABH</i>


Vì tam giác <i>ABH vng nên O’ là trung điể</i>m của <i>AB </i>
Tam giác <i>AO’H</i> cân tại <i>O’</i> nên ∠<i>O HA</i>' = ∠<i>O AH</i>'

( )

1


<i>AHM</i> <i>AIM</i>



∠ = ∠ (2 góc nội tiếp cùng chắn một cung)


<i>AIM</i> <i>ADC</i>


∠ = ∠ (đồng vị)


<i>ADC</i> <i>ABH</i>


∠ = ∠ (2 góc nội tiếp cùng chắn một cung)
Nên∠<i>AHM</i> = ∠<i>ABH</i>

( )

2


Từ (1) và (2) ⇒ ∠<i>O HA</i>' + ∠<i>AHM</i> = ∠<i>O AH</i>' + ∠<i>ABH</i> =90<i>o</i>


'


<i>MH</i> <i>O H</i> <i>HM</i>


⇒ ⊥ ⇒ Là tiếp tuyến của đường tròn ngoại tiếp tam giác <i>ABH.</i>
<b>4)</b> Ta có hai tam giác <i>AHB</i> và <i>ACD</i>đồng dạng


. .


<i>AB CD</i> <i>BH AD</i>


⇒ =


Chứng minh tương tựnhư trên ta có hai tam giác AHC và <i>ABD đồ</i>ng dạng


. .



<i>AC BD</i> <i>AD HC</i>


⇒ =


(

)



. . . .


<i>AC BD</i> <i>AB CD</i> <i>AD HC</i> <i>BH AD</i> <i>AD HC</i> <i>HB</i> <i>AD BC</i>


⇒ + = + = + =


2


2 ; 2 . . 4


</div>
<span class='text_page_counter'>(18)</span><div class='page_container' data-page=18>

<b>Câu 6. </b>


2


0


<i>ax</i> +<i>bx</i>+ =<i>c</i> Có nghiệm <i>m, n </i> nên


<i>b</i>
<i>m</i> <i>n</i>
<i>a</i>
<i>c</i>
<i>mn</i>
<i>a</i>



 + =


 <sub>=</sub>


(

)(

)

(

)(

)


2
2 2
1 2


2 1 2


2 2


1
1


<i>b</i> <i>c</i>


<i>a</i> <i>b</i> <i>a</i> <i>c</i> <i>m</i> <i>n</i> <i>mn</i>


<i>a</i> <i>ac</i> <i>ab</i> <i>bc</i> <i>a</i> <i>a</i>


<i>Q</i>


<i>b</i> <i>c</i> <i>m</i> <i>n</i> <i>mn</i>


<i>a</i> <i>ab</i> <i>ac</i> <i>a</i> <i>ab</i> <i>ac</i>



<i>a</i> <i>a</i>
 <sub>−</sub>  <sub>−</sub> 
  
− − + + −
− − +   
= = = =
+ + +
− + − + <sub>− +</sub>

(

) (

) (

)


(

)

<sub>(</sub>

<sub>)</sub>


1
0 1; 0 1


1 1 1 0


1
1 3
1
1 4
1
1 1
3
3
<i>mn</i>


<i>Do</i> <i>m</i> <i>n</i>


<i>m n</i> <i>n m</i> <i>mn</i>


<i>mn</i>



<i>m</i> <i>n</i>


<i>Q</i> <i>Q</i>


<i>mn</i> <i>m</i> <i>n</i>


<i>m</i> <i>n</i> <i>m</i> <i>n</i>




≤ ≤ <sub>≤ ≤ ⇒ </sub>
− + − + − ≤

 <sub>≤</sub>
 <sub>+ +</sub>

⇒<sub></sub> ⇒ ≥ ⇔ ≥
≤ + +
 <sub>+ + +</sub> <sub>+ +</sub>



Vậy giá trị nhỏ nhất của Q là 3
4khi


0
<i>a</i> <i>b</i> <i>c</i>


<i>a</i> <i>c</i>
+ + =




 <sub>=</sub>




<b>Đề</b>

<b> s</b>

<b>ố</b>

<b> 2 </b>



<b>Câu 1. </b>
<b>1)</b> Ta có:


2 2


2 2


2 2 2 2


2


2


2


4 1 1 4


1 1 1 1 4


1 1 4


1 1


1


<i>a</i> <i>b</i>


<i>A</i>


<i>ab</i> <i>ab</i>


<i>a b</i> <i>a</i> <i>b</i>


<i>a</i> <i>b</i> <i>a</i> <i>b</i> <i>ab</i>


<i>a</i> <i>b</i> <i>ab</i>


<i>a</i> <i>b</i>
 −   
=<sub></sub> <sub></sub> + =<sub></sub> − <sub></sub> +
 
 
  
=<sub></sub><sub></sub> − <sub></sub> + <sub></sub><sub></sub> +
  
 
 
=<sub></sub> − <sub></sub> +
 
 
=<sub></sub> + <sub></sub>
 
=



2) Từ giả thiết:


(

)(

)



1 1


1 <i>a</i> <i>b</i> <i>ab</i> <i>ab</i> <i>a</i> <i>b</i> 1 1 <i>a</i> 1 <i>b</i> 1 1


<i>a</i>+ = ⇒ + =<i>b</i> ⇒ − − + = ⇔ − − =


Áp dụng hằng đẳng thức:


(

)

3 3 3

(

)



3


</div>
<span class='text_page_counter'>(19)</span><div class='page_container' data-page=19>

(

) (

)

(

) (

)

(

)(

) (

) (

)



(

) (

) (

)

(

)



(

) (

) (

)

(

)



3 3 3


2 3 3


3 3 3


1 1 1 1 3 1 1 1 1



2 1 1 3 2


2 1 1 3 6 0


<i>a</i> <i>b</i> <i>a</i> <i>b</i> <i>a</i> <i>b</i> <i>a</i> <i>b</i>


<i>a</i> <i>b</i> <i>a</i> <i>b</i> <i>a</i> <i>b</i>


<i>a</i> <i>b</i> <i>a</i> <i>b</i> <i>a</i> <i>b</i>


⇒<sub></sub> − + − <sub></sub> = − + − + − − <sub></sub> − + − <sub></sub>


⇒ + − = − + − + + −


⇔ + − − − − − − + + =


<b>Câu 2. </b>


<b>1)</b> Điều kiện <i>x</i>≥1


(

)(

)



(

) (

)

(

)(

)



2 2 1 3 3 2 1


2 2 1 2 2 1 3 2 1


<i>x</i> <i>x</i> <i>x</i> <i>x</i> <i>x</i>



<i>x</i> <i>x</i> <i>x</i> <i>x</i> <i>x</i> <i>x</i>


+ − − = − + −


⇔ + − − = + + − − + −


Đặt <i>a</i>= <i>x</i>+2;<i>b</i>= <i>x</i>−1

(

<i>a b</i>; ≥0

)



Ta được phương trình:


(

)(

)



2 2 2


2 2 3 2 1 0


1


<i>a</i> <i>b</i>


<i>a</i> <i>b</i> <i>a</i> <i>b</i> <i>ab</i> <i>a</i> <i>b</i> <i>a</i> <i>b</i>


<i>a</i> <i>b</i>
=

− = + − ⇔ − <sub>− − = ⇔ </sub>
= +



Với <i>a</i>=2<i>b</i>⇒ <i>x</i>+ =2 2 <i>x</i>− ⇔ =1 <i>x</i> 2


Với <i>a</i>= + ⇒<i>b</i> 1 <i>x</i>+ =2 <i>x</i>− + ⇔ + = +1 1 <i>x</i> 2 <i>x</i> 2 <i>x</i>− ⇔ =1 <i>x</i> 2


Vậy phương trình có nghiệm là x = 2.
<b>2)</b> Điều kiện 2


1; 4; 0


<i>x</i>≥ <i>y</i>≥ − <i>x</i> + ≥<i>y</i>


Biến đổi phương trình (1):

( )



(

)

(

)



( )



2 2 2


2
2


2
2


2


1 2 1 4 4



1


1 2


3 1


<i>x</i> <i>y</i> <i>x</i> <i>y</i> <i>x</i> <i>x</i>


<i>x</i> <i>y</i> <i>x</i>


<i>x</i> <i>y</i> <i>x</i>


<i>x</i> <i>y</i> <i>x</i> <i>L</i> <i>x</i>


⇔ + + + + = + +
 <sub>+ = +</sub>

⇔ + + = + ⇔
 <sub>+ = − −</sub> <sub>≥</sub>


Với 2


1 2 1


<i>x</i> + = + ⇔ =<i>y</i> <i>x</i> <i>y</i> <i>x</i>+ thay vào (2) ta được:


(

)

(

)



(

)

(

)

(

)

(

)




(

)(

) (

)(

)

<sub>(</sub>

<sub>)</sub>



(

)



3 2 5 2 3 1 2 0


3 2 5 3 2 3 1 1 5 10 0


3 2 4 2 3 2


5 2 0


2 5 3 1 1


2 6 2 3


2 5 0


2 5 3 1 1


<i>x</i> <i>x</i> <i>x</i> <i>x</i>


<i>x</i> <i>x</i> <i>x</i> <i>x</i> <i>x</i>


<i>x</i> <i>x</i> <i>x</i> <i>x</i>


<i>x</i>
<i>x</i> <i>x</i>
<i>x</i> <i>x</i>


<i>x</i>
<i>x</i> <i>x</i>
− + + − − + =
⇔ − − − + − − − + − =
− − − −
⇔ + + − =
+ + − +
 − − 
⇔ − <sub></sub> + + <sub></sub>=
+ + − +
 

(

)


(

)



2 6 2 3


2 2 3 0


2 5 3 1 1


2 2 2 5 2 3 1


2 0


2 5 3 1 1


2


<i>x</i> <i>x</i>



<i>x</i>


<i>x</i> <i>x</i>


<i>x</i> <i>x</i> <i>x</i> <i>x</i>


</div>
<span class='text_page_counter'>(20)</span><div class='page_container' data-page=20>

2 2 2 5 2 3 1


0 1


2 5 3 1 1


<i>x</i> <i>x</i> <i>x</i> <i>x</i>


<i>Do</i> <i>x</i>


<i>x</i> <i>x</i>


+ + + −


+ > ∀ ≥


+ + − +


Vậy hệphương trình có nghiệm (x, y) = (2; 5)
<b>Câu 3. </b>


<b>1)</b> Ta có <i>PNE</i>  <i>NAC</i>  <i>NCA</i> 2 <i>NCA</i><i>sdEA</i>


Chứng minh∠<i>PAD</i>= ∠<i>ABE</i>



Suy ra ∠<i>PDE</i> = ∠<i>PAD</i>+ ∠<i>ADE</i> = ∠<i>ABE</i>+ ∠<i>ADE</i> = ∠2 <i>ABE</i> =<i>sd EA</i>


Xét tứ giác <i>EPND</i> có <i>PNE</i> <i>PDE</i>và hai đỉnh <i>N; D</i>là hai đỉnh liên tiếp nên
<i>EPND </i>là tứ giác nội tiếp đường tròn.


2) Ta thấy tứ giác AKME là tứ giác nội tiếp do <i><sub>MEK</sub></i> <sub></sub><i><sub>MAK</sub></i> <sub></sub>45<i>o</i>
Suy ra <i><sub>MEA</sub></i> <sub></sub><i><sub>MKA</sub></i> <sub></sub>90<i>o</i>


Do đó <i>MK</i> / /<i>BD</i> <i>MKD</i> <i>KDB</i> <i>KBD</i> <i>EKM</i>


<b>3)</b> Chứng minh <i>MDC</i>đồng dạng <i>MEA g g</i>

 

.


Suy ra <sub>.</sub> <sub>.</sub> 2 2


4


<i>MD</i> <i>ME</i> <i><sub>MC ME</sub></i> <i><sub>MD MA</sub></i> <i><sub>MD</sub></i> <i>CD</i>


<i>MC</i>  <i>MA</i>    


Mặt khác ta có 2 2 2 2 2 5 2


4 4


<i>CD</i> <i>CD</i>


<i>MC</i> <i>CD</i> <i>MD</i> <i>CD</i>  


Suy ra: 5 5



2 10


<i>CD</i> <i>CD</i>


<i>MC</i>  <i>ME</i> 


Mà . 5 10


5 5


<i>EA</i> <i>AM</i> <i>AM CD</i>


<i>EA</i> <i>CD</i> <i>R</i>


<i>CD</i>  <i>MC</i>   <i>MC</i>  


<b>Câu 4. </b>


1) Điều kiện: <i>x</i>≥0,<i>y</i>≥0


</div>
<span class='text_page_counter'>(21)</span><div class='page_container' data-page=21>

( )

1 2020 2020 2 2020


2020 4 5.101


<i>x</i> <i>y</i> <i>x</i> <i>y</i> <i>y</i>


<i>x</i> <i>y</i> <i>y</i>


⇔ = − ⇔ = + −



⇔ = + −


Do x, y nguyên nên 5.101<i>y</i>nguyên hay 5.101y là sốchính phương.


Suy ra 2 2 2


5.101<i>y</i>=<i>k</i> ⇒ =<i>y</i> 5.101.<i>a</i> =505<i>a</i> (<i>a</i> là số nguyên)


Tương tự 2 2


5.101 505


<i>x</i>= <i>b</i> = <i>b</i> (b là số nguyên)


Thay <i>x; y</i> theo <i>a; b</i>vào (1) ta được:


505 505 2 505 2


<i>a</i> + <i>b</i> = ⇔ <i>a</i> + <i>b</i> =


<i>a</i> <i>b</i> 2


505


<i>x</i>= <i>b</i> <i>y</i>=505<i>a</i>2


0 2 2020 0


1 1 505 505



2 0 0 2020


Vậy phương trình có các nghiệm là: (2020;0);(505;505);(0;2020)
<b>Câu 5. </b>


2) Ta có:


(

1 22

)

(

1 23

)

(

1 24

)



<i>P</i>


<i>a</i> <i>a</i> <i>b</i> <i>b</i> <i>c</i> <i>c</i>


= + +


− − −


(

)

(

)

(

)



2 2 2


2 3 4


1 2 1 2 1 2


<i>a</i> <i>b</i> <i>c</i>


<i>a</i> <i>a</i> <i>b</i> <i>b</i> <i>c</i> <i>c</i>



= + +


− − −


Áp dụng bất đẳng thức AG – GM ta có: 2

(

)

1 2 3 1


1 2


3 27


<i>a</i> <i>a</i> <i>a</i>


<i>a</i> − <i>a</i> ≤<sub></sub> + + − <sub></sub> =


 


Tương tự : 2

(

)

1 2

(

)

1


1 2 ; 1 2


27 27


<i>b</i> − <i>b</i> ≤ <i>c</i> − <i>c</i> ≤


Suy ra: <i>P</i>≥27 2

(

<i>a</i>+3<i>b</i>+4<i>c</i>

)

=81


Dấu “=” xảy ra khi 1


3
<i>a</i>= = =<i>b</i> <i>c</i>



Vậy giá trị nhỏ nhất của P là 81.


3) Xét điểm <i>I</i>

(

2; 3

)

. Ta chứng minh khoảng cách từ<i>I</i>đến hai điểm nguyên


khác nhau là khác nhau.


Xét hai điểm nguyên <i>M a b</i>

( )

; ;<i>M</i>'

(

<i>a</i>'; b'

)



(

) (

) (

) (

)



(

)

(

)



2 2


2 2 2 2


2 2 2 2


' IM'


2 3 ' 2 ' 3


' ' 2 ' 2 2 ' 3 0


<i>IM</i> <i>IM</i> <i>IM</i>


<i>a</i> <i>b</i> <i>a</i> <i>b</i>


<i>a</i> <i>b</i> <i>a</i> <i>b</i> <i>a</i> <i>a</i> <i>b</i> <i>b</i>



= ⇔ =


⇔ − + − = − + −


</div>
<span class='text_page_counter'>(22)</span><div class='page_container' data-page=22>

Nhận xét nếu các số nguyên <i>m, n, p </i> thỏa mãn <i>m</i>+<i>n</i> 2+ <i>p</i> 3=0thì <i> m = n = p = 0 </i>


2 2 2


2 2 2


2 2 2


2; 3; 6 ; , , p Q


2 2 3 2


2 3 2 3


2 6 2 3


2 3 0


<i>Q m n</i>


<i>mn</i> <i>p</i> <i>m</i> <i>n</i>


<i>mp</i> <i>n</i> <i>m</i> <i>p</i>


<i>pm</i> <i>m</i> <i>n</i> <i>p</i>



<i>m</i> <i>n</i> <i>p</i>


 <sub>∉</sub> <sub>∈</sub>

= − −


= − −


= − −

 <sub>+</sub> <sub>+</sub> <sub>=</sub>

0
0
2 3 0


<i>mn</i> <i>np</i> <i>pm</i>


<i>m</i> <i>n</i> <i>p</i>


<i>m</i> <i>n</i> <i>p</i>


= = =

⇒<sub></sub> ⇒ = = =
+ + =



Ta có:

(

)


(

)



2 2 2 2


2 2 '


' ' 0


'


IM' IM ' 2 ' 0


'
2 ' 0


<i>a</i> <i>b</i> <i>a</i> <i>b</i>


<i>b</i> <i>b</i>


<i>IM</i> <i>IM</i> <i>a</i> <i>a</i> <i>M</i> <i>M</i>


<i>a</i> <i>a</i>
<i>b</i> <i>b</i>
 + − − =
=


= ⇔ = ⇔<sub></sub> − = ⇔<sub> =</sub> ⇔ ≡



 <sub>−</sub> <sub>=</sub>


Xét tất cả các khoảng cách từcác điểm nguyên <i>I, </i>các khoảng cách này đôi một phân
biệt. Gọi S là tập hợp các số thực bằng các khoảng cách cách từ tất cả các điểm
nguyên đến <i>I.</i> Ta có thể chọn 2020 sốdương nhỏ thuộc S và được sắp theo thứ tự
tăng dần, nghĩa là tồn tại các sốdương<i>s s</i>1; 2;...<i>s</i>2020 thuộc tập S thỏa mãn <i>sp</i> <<i>sq</i>nếu


<i>p < q,</i> các số thuộc <i>S</i> \

{

<i>s s</i>1; 2;....<i>s</i>2020

}

đều lớn hơn <i>s s</i>1; 2;...<i>s</i>2020. Đặt
1


, 1; 2;3;...2019
2


<i>k</i> <i>k</i>


<i>k</i>


<i>s</i> <i>s</i>


<i>R</i> = + + <i>k</i> =


.


Ta có điều phải chứng minh.


<b>Đề</b>

<b> s</b>

<b>ố</b>

<b> 3 </b>



<b>Câu 1. </b>


a)


b)




c)


=21 ( Do và cơsi)
Vậy<i> T </i>có giá trị nhỏ nhất là 21 khi


(

2)(

2)



1 .(2

1)(

1)


(

1)(

2)



<i>x</i>

<i>x</i>



<i>P</i>

<i>x</i>

<i>x</i>



<i>x</i>

<i>x</i>


<sub>−</sub>

<sub>+</sub>


=

<sub></sub>

+

<sub></sub>




2

1



(2

1)(

1)



1



<i>x</i>



<i>P</i>

<i>x</i>

<i>x</i>


<i>x</i>



+



=





4

1


<i>P</i>

=

<i>x</i>



2019

4

1 2019


<i>P</i>

=

<i>x</i>

− =



505


<i>x</i>

=



10

10

10

2

18



4

1 (

)

1



5

5



<i>x</i>

<i>x</i>


<i>T</i>

<i>P</i>

<i>x</i>



<i>x</i>

<i>x</i>

<i>x</i>




= +

=

+

− =

+

+



10

2

18

10 2

18



(

)

1 2

.

.5 1



5

5

5

5



<i>x</i>

<i>x</i>

<i>x</i>



<i>T</i>



<i>x</i>

<i>x</i>



=

+

+

− ≥

+

<i>x</i>

5



</div>
<span class='text_page_counter'>(23)</span><div class='page_container' data-page=23>

<b>Câu 2. </b>


Hoành độ điểm I là nghiệm của phương trình .


<b>Chú ý: Ý trên học sinh có thể dùng quỹ tích I là đường trịn R = 1. </b>
<b>Câu 3. </b>


a) nên phương trình (1) ln có hai nghiệm phân biệt


Theo viét






b)


(<i> Phương trình có hai nghiệm phân biệt khác -1 với mọi m) </i>


<i>T nhỏ nhất là 1 khi m </i>= 0
<b>Câu 4. </b>


a) Đk ta có


b)




Với thế vào ta có


Vậy hệ có hai nghiệm là
<b>Câu 5. </b>


2
2


1

1

1



1


<i>m</i>


<i>x</i>

<i>mx</i>

<i>m</i>

<i>x</i>



<i>m</i>

<i>m</i>

<i>m</i>





+

=

+ ⇔ =


+


2
2 2

1

2


1

1


<i>m</i>

<i>m</i>


<i>do x</i>

<i>y</i>



<i>m</i>

<i>m</i>



=

⇒ =


+

+


2
2 2

1

2


(

;

)


1

1


<i>m</i>

<i>m</i>


<i>I</i>


<i>m</i>

<i>m</i>




+

+


2
2 2
2 2

1

2




(

)

(

)

1



1

1


<i>m</i>

<i>m</i>


<i>T</i>


<i>m</i>

<i>m</i>



=

+

=


+

+


2

8

0


<i>m</i>

<i>m</i>



∆ =

+ > ∀



1 2


1 2


2


1



<i>x</i>

<i>x</i>

<i>m</i>



<i>x x</i>

<i>m</i>



+

= −






<sub>= − −</sub>





2 2


1 2

2 2

(

1 2

)

8

(

1 2

)

4 .

1 2

8



<i>x</i>

<i>x</i>

=

<i>x</i>

<i>x</i>

= ⇔

<i>x</i>

+

<i>x</i>

<i>x x</i>

=



2 2


(

<i>m</i>

2)

− − −

4( 1

<i>m</i>

) 8

= ⇔

<i>m</i>

= ⇔ =

0

<i>m</i>

0



2 2 2


2 1 1 2 1 2 1 2


2 2 2


1 2 1 2 1 2


(

1)

(

1)

2

(

)

2 .

2(

)



(

1) .(

1)

( .

1)



<i>x</i>

<i>x</i>

<i>x</i>

<i>x</i>

<i>x x</i>

<i>x</i>

<i>x</i>


<i>T</i>




<i>x</i>

<i>x</i>

<i>x x</i>

<i>x</i>

<i>x</i>



+

+

+

+

+

+

+


=

=


+

+

+ +

+


2

4


1


4


<i>m</i>



<i>T</i>

=

+



2018



<i>x</i>

≥ −

4(

<i>x</i>

+

2018)

+

9(

<i>x</i>

+

2018)

=

5


2

<i>x</i>

+

2018

+

3

<i>x</i>

+

2018

= ⇔

5

<i>x</i>

+

2018

=

1



2017


<i>x</i>

= −



3 3 2 3 3


3

6

3

4

0

[(

1)

] 3(

1) 3

0



<i>x</i>

<i>y</i>

+

<i>x</i>

+

<i>x</i>

<i>y</i>

+ = ⇔

<i>x</i>

+

<i>y</i>

+

<i>x</i>

+ −

<i>y</i>

=



2 2


(

<i>x</i>

+ −

1

<i>y</i>

)[(

<i>x</i>

+

1)

+ +

(

<i>x</i>

1)

<i>y</i>

+

<i>y</i>

+ = ⇔ = +

3] 0

<i>y</i>

<i>x</i>

1




1



<i>y</i>

= +

<i>x</i>

<i>x</i>

2

+

<i>y</i>

2

3

<i>x</i>

=

1

2


0



2

0

<sub>1</sub>



2


<i>x</i>


<i>x</i>

<i>x</i>


<i>x</i>


=




− = ⇔


 =



1 3


(0;1),( ; )



</div>
<span class='text_page_counter'>(24)</span><div class='page_container' data-page=24>

a) Do MK và <i>ON </i>vng góc <i>JN (1) </i>
<i> NK </i>và <i>OM </i>vuông góc <i>JM (2) </i>


Nên từ <i>(1)</i> và <i>(2)</i> có Tứ giác <i>OMKN </i>là hình bình hành(3), suy ra <i>H</i> là
trung điểm OK.


b) Do OM =<i>ON (4) .</i>Từ (3)&(4)có tứ giác OMKN là hình thoi <i>(5) </i>
Mặt khác OJ =<i>2OM = 2a </i>suy ra <i> (6) </i>



Từ(5)và<i>(6)</i> đều


<i>K</i>thuộc đường tròn tâm O.
c) Do (M;r)nhận OJ là tuyến tuyến mà


Ta có


<i>( hoặc dùng hệ thức lượng trong tam giác vuông) </i>


d) Gọi IE,IFlà hai tiếp tuyến với (O) tại E,F và
Suy ra tứ giác IEOF là hình vng


Tính (Khơng đổi)(1)


Do Ocố định (2)


Từ (1) và (2) tập hợp Inằm trên đường tròn tâm O bán kính
<b>Câu 6. </b>


Do là ba số thực khơng âm thỏa mãn : .


Ta có (*)


Từ điều kiện trên ta có


/ /


/ /



<i>MK</i>

<i>ON</i>




<i>NK</i>

<i>OM</i>



<sub>⇒</sub>






0


60


<i>MOJ</i>



=



0


60



<i>MOK</i>

<i>OMK</i>



⇒ ∠

=



<i>OK</i>

<i>OM</i>

<i>R</i>

<i>a</i>



=

= = ⇒



<i>MH</i>

<i>JO</i>

=

<i>H</i>

⇒ =

<i>r</i>

<i>MH</i>



2 2 2 2



1

1

1

4



3



<i>MH</i>

=

<i>OM</i>

+

<i>JM</i>

=

<i>a</i>



3


2


<i>a</i>


<i>r</i>

=



<i>IE</i>

<i>IF</i>



2


<i>OI</i>

=

<i>a</i>



2


<i>a</i>



, ,



<i>x y z</i>

12

<i>x</i>

+

10

<i>y</i>

+

15

<i>z</i>

60



, ,

0


5


6


4


<i>x y z</i>


<i>x</i>



<i>y</i>


<i>z</i>






 ≤



 ≤



 ≤




2 2 2


4

4



<i>T</i>

=

<i>x</i>

+

<i>y</i>

+

<i>z</i>

<i>x</i>

<i>y</i>

<i>z</i>


E


M


K


J


N


F
I



O


</div>
<span class='text_page_counter'>(25)</span><div class='page_container' data-page=25>

Vậy GTLN của T bằng 12 đạt được khi


<b>Đề</b>

<b> s</b>

<b>ố</b>

<b> 4 </b>



<b>Câu 1 </b>
<b>1)</b> <b> </b>


Phương trình có hai nghiệm không âm


Gọi là hai nghiệm của phương trình đã cho, ta có:
(định lý Vi-et)


Với ta có:


Dấu xảy ra


Vậy khi


<b>2)</b> <b> </b>
Ta có:
Để


-1 1 -2 2 -3 3 -6 6


-3 -1 -4 0 -5 1 -8 4


tm tm tm tm tm tm tm tm



Vật tập hợp các giá trị của để y nguyên là
<b>Câu 2 </b>


(

5)

(

6)

(

4)

2

3



12

60



2

3

2

3

12



5

5



<i>x x</i>

<i>y y</i>

<i>z z</i>

<i>x</i>

<i>y</i>

<i>z</i>


<i>x</i>



<i>x</i>

<i>y</i>

<i>z</i>

<i>y</i>

<i>z</i>



=

− +

− +

− + +

+



≤ +

+

+

+

=



0

0



6 or

0



0

4



<i>x</i>

<i>x</i>


<i>y</i>

<i>y</i>


<i>z</i>

<i>z</i>




=

=





<sub>=</sub>

<sub>=</sub>





<sub>=</sub>

<sub>=</sub>





(

)



2 2


2 2


' 0 2 4 0 2


0 2 0 0 2


0 <sub>2</sub> <sub>4</sub> <sub>0</sub> <sub>1</sub> <sub>3</sub> <sub>0</sub> <sub>(</sub> <sub>...</sub> <sub>)</sub>


<i>m</i> <i>m</i> <i>m</i> <i>m</i>


<i>S</i> <i>m</i> <i>m</i> <i>m</i>


<i>P</i> <i><sub>m</sub></i> <i><sub>m</sub></i> <i><sub>m</sub></i> <i><sub>luon dung</sub></i>






∆ ≥ − + − ≥ ≥


 <sub></sub>




 


⇔<sub></sub> ≥ ⇔<sub></sub> ≥ ⇔ <sub></sub> ≥ ⇔ ≥


 <sub>≥</sub>  <sub>−</sub> <sub>+ ≥</sub> 


 <sub></sub> <sub></sub> − + ≥


1

;

2


<i>x x</i>



1 2


2
1 2


2


2 4



<i>x</i> <i>x</i> <i>m</i>


<i>x x</i> <i>m</i> <i>m</i>


+ =


 <sub>=</sub> <sub>−</sub> <sub>+</sub>




2 2


1 2

0

1 2

2

1 2

2

2

2

4



<i>P</i>

=

<i>x</i>

+

<i>x</i>

≥ ⇒

<i>P</i>

= + +

<i>x</i>

<i>x</i>

<i>x x</i>

=

<i>m</i>

+

<i>m</i>

<i>m</i>

+



2


<i>m</i>≥


(

)

(

)



2
2


2 2 2 4 2 2 2 2 4 8


8 2 2



<i>P</i> <i>m</i> <i>m m</i>


<i>P</i> <i>P</i>


= + − + ≥ − + =


⇒ ≥ ⇔ ≥


"=" ⇔<i>m</i>=2
2 2


<i>Min</i> <i>P</i>= <i>m</i>=2


2 2


2 4 6 6


2


2 2 2


<i>x</i> <i>x</i>


<i>y</i> <i>x</i>


<i>x</i> <i>x</i> <i>x</i>


+ − +



= = = − +


+ + +


(

2

)

( ) {

6 1; 2; 3; 6

}



<i>y</i>∈ ⇒ <i>x</i>+ ∈<i>U</i> = ± ± ± ±


2


<i>x</i>+


<i>x</i>


</div>
<span class='text_page_counter'>(26)</span><div class='page_container' data-page=26>

<b>1)</b> <b> </b>


Phương trình ln có nghiệm
<b>2)</b> <b>Giải phương trình </b>


Dễ thấy


Đểphương trình có nghiệm thì


vơ nghiệm
Vậy phương trình đã cho vơ nghiệm


<b>Câu 3. </b>


Giả sử chiều dài của hai cây nến là



Gọi thời gian đốt hai cây nến đểđượ phần còn lại của cây nến thứ hai gấp đơi phần cịn lai
của cây nến thứ nhất là (giờ)


Theo đề bài ta có, trong 1 giờthì đốt được độ dài các cây nến thứ nhất và thứ hai lần lượt


Trong giờ thì độ dài cây nến thứ nhất và thứhai đã đốt lần lượt là (cm)


Độ dài cây nến thứ nhất và thứ hai còn lại sau khi đã đốt (giờ) là:
Theo đềbài ta có phương trình


5


2 5 0


2


<i>a</i> <i>c</i>


<i>a</i>+ <i>b</i>+ <i>c</i>= ⇔ =<i>b</i> − −


(

)

2 <sub>2</sub>


2 2 2 2


2 10 25 6 25 3 16


4 4 0 ; ;


4 4 4



<i>a</i> <i>c</i> <i>c</i>


<i>a</i> <i>ac</i> <i>c</i> <i>a</i> <i>ac</i> <i>c</i>


<i>b</i> <i>ac</i> + + <i>ac</i> − + − + <i>a b c</i>


∆ = − = − = = ≥ ∀


⇒ 2


0
<i>ax</i> +<i>bx</i>+ =<i>c</i>


(

)

3


3 3 3


4 3 :


2


<i>x</i> − +<i>x</i> =<i>x</i>


(

)



(

)

(

)

(

)

(

)



3



3 3


3


3 3


3 <sub>2</sub> <sub>3</sub> <sub>2</sub>


3


4 3 :


2


2 2


1 4 4 3 . 4 4 3 . 1


3 1 3


<i>x</i> <i>x</i> <i>x</i>


<i>x</i>


<i>x</i> <i>x</i> <i>x</i> <i>x</i> <i>x</i> <i>x</i> <i>x</i>


<i>x</i>


− + =



 


⇔ + − + = ⇔ − + =<sub></sub> <sub></sub> ≠ −


+


 


3 3


2 2


1 1; 1 . 1 1


1 3 1 3


<i>x</i> <i>x</i>


<i>x</i> <i>x</i>


<i>x</i> <i>x</i>


  <sub>< ∀ ≠ −</sub> <sub>< ⇒</sub>  <sub><</sub> <sub>∀ ≠ −</sub>


 <sub>+</sub>   <sub>+</sub> 


   


2



4<i>x</i> −4<i>x</i>+ <3 1


(

)

2


2


4<i>x</i> 4<i>x</i> 2 0 2<i>x</i> 1 1 0


⇔ − + < ⇔ − + <


( )


<i>L cm</i>


<i>x</i>

(

<i>x</i>>0

)



, ( )


3 4


<i>L L</i>
<i>cm</i>


<i>x</i> ,


3 4


<i>xL xL</i>


⇒ <i>x</i> , ( )



3 4


<i>xL</i> <i>xL</i>


</div>
<span class='text_page_counter'>(27)</span><div class='page_container' data-page=27>

Vậy phải đốt hai cây nến trong 2,4 giờ hay phải đốt hai cây nến lúc giờ =1 giờ
36 phút chiều đểđược yêu cầu như bài tốn.


<b>Câu 4. </b>


Từ giả thiết ta có:


Tương tự ta có:


Cộng từng vế của hai phương trình trên ta được:


Xét


Dấu xảy ra


Vậy khi


<b>Câu 5. </b>


2


2 1 2


4 3 4 3


5 12



1 2, 4 ( )


12 5


<i>xL</i> <i>xL</i> <i>x</i> <i>x</i>


<i>L</i> <i>L</i>


<i>x</i>


<i>x</i> <i>tm</i>


 


− = <sub></sub> − <sub></sub>⇔ − = −


 


⇔ = ⇔ = =


4−2, 4=1, 6


(

)



(

)

(

)



2


2 2



2 2


2


2018 1


2018


1 2018 1


1
1


<i>y</i> <i>y</i>


<i>x</i> <i>x</i> <i>y</i> <i>y</i>


<i>y</i> <i>y</i>


<i>y</i> <i>y</i>


− +


+ + = = = + −


− +
+ +


(

)




2 2


1 2018 1


<i>y</i>+ + <i>y</i> = +<i>x</i> −<i>x</i>


(

)

(

2 2

)



2019 <i>x</i>+<i>y</i> =2017 1+<i>x</i> + 1+ <i>y</i>


(

)

2

(

)(

)



2 2 2 2 2 2


1 1 2 2 1 1


<i>A</i>= +<i>x</i> + +<i>y</i> = +<i>x</i> +<i>y</i> + +<i>x</i> +<i>y</i>


(

)

(

)



(

)



(

)



(

)



2


2 2



2


2


2


2 2 2 2


2 2


2


2 2 1 4


2017 4


2019( ) 2017 4


2019 2017 4


2019 2017 . 4


4.2017 2017 2017 2018


2.4036 2018 2018


<i>A</i> <i>x</i> <i>y</i> <i>xy</i> <i>x</i> <i>y</i>


<i>VP</i> <i>x</i> <i>y</i>



<i>VT</i> <i>x</i> <i>y</i> <i>x</i> <i>y</i>


<i>P</i> <i>P</i>


<i>P</i> <i>P</i>


<i>P</i> <i>P</i>


⇒ ≥ + + + + = + +


⇒ ≥ + +


⇒ = + ≥ + +


⇒ = +


⇒ ≥ +


⇒ ≥ ⇒ ≥ =


"="

2017 2018



4036


<i>x</i>

<i>y</i>



⇔ = =


2017 2018



2018




<i>MinP</i>

=

2017 2018



</div>
<span class='text_page_counter'>(28)</span><div class='page_container' data-page=28>

<b>1)</b> <b> </b>


<b>a)</b> <b>Tính diện tích nửa đường trịn đường kính BH</b>


Ta có: vng tại A( định lý Pytago đảo)


Áp dụng hệ thức lượng tròn tam giác vng ta có:
Diện tích nửa đường trịn đường kính BH là


<b>b)</b> <b>Chứng minh tứgiác BEFC nội tiếp và…… </b>
Gọi và lần lượt là trung điểm của và


Dễ nhận thấy AEHF là hình chữ nhật (tứ giác có 3 góc vng)


Mà (cùng phụ với


Mà nội tiếp chăn cung HE của đường tròn ; tạo bởi dây cung EH và EF ở
vị trí góc tại bởi tia tiếp tuyến và dây cung


là tiếp tuyến của đường tròn đường kính BH


Chứng minh tương tự ta có EF là tiếp tuyến của đường trịn đường kính CH


Vì AEHF là hình chữ nhật . Mà (cùng phụ với


. Mà



Tứ giác là tứ giác nội tiếp (Tứ giác có tổng hai góc đối bằng 180 )
<b>2)</b> <b>Cho nửa dường trịn dường kính AB=2R………</b>


<i><b>F</b></i>


<i><b>E</b></i>



<i><b>O2</b></i>



<i><b>O1</b></i>

<i><b>H</b></i>



<i><b>B</b></i>



<i><b>A</b></i>



<i><b>C</b></i>



2 2 2


<i>AB</i> +<i>AC</i> =<i>BC</i> ⇒ ∆<i>ABC</i>


2


16
5


<i>AB</i>
<i>BH</i>


<i>BC</i>



= =




(

)



2 2


1 1 8 32


2 2 2 5 25


<i>BH</i>


<i>S</i> =

π

<sub></sub> <sub></sub> =

π

<sub> </sub>  =

π

<i>dvdt</i>


   


1


<i>O</i>

<i>O</i>

<sub>2</sub> <i>BH</i> <i>CH</i>


 


<i>CAH</i> <i>FEH</i>


⇒ =


 



<i>CAH</i> = <i>ABC</i>

<i>BAH</i>

)

<i>FEH</i>

 

=

<i>ABC</i>





<i>ABC</i>

( )

<i>O</i><sub>1</sub>

<i>FEH</i>



<i>EF</i>




 


<i>AEF</i> <i>AHF</i>


⇒ =  <i>AHF</i> = <i>ACB</i>

<i>FHC</i>

)



 


<i>AEF</i> <i>ACB</i>


⇒ =   0   0


180 180


<i>AEF</i>+<i>BEF</i> = ⇒ <i>ACB</i>+<i>BEF</i> =


</div>
<span class='text_page_counter'>(29)</span><div class='page_container' data-page=29>

Đặt ta có:


Áp dụng hệ thức lượng trong tam giác vng AMB ta có:
Khi đó ta có:



Dấu bằng xảy ra


<b>Câu 6. </b>


Chứng minh tương tự ta có:


<i><b>P</b></i>


<i><b>Q</b></i>



<i><b>N</b></i>



<i><b>B</b></i>


<i><b>O</b></i>



<i><b>A</b></i>



<i><b>M</b></i>



2


<i>MN</i> = <i>x</i> <i>AQ</i>= −<i>R</i> <i>x</i>; <i>BQ</i>= +<i>R</i> <i>x</i>


2 2


.


<i>MQ</i>= <i>AQ BQ</i> = <i>R</i> −<i>x</i>


2 2 2 2 2 2



.

2


<i>MNPQ</i>


<i>S</i>

=

<i>MN MQ</i>

=

<i>x R</i>

<i>x</i>

<i>x</i>

+

<i>R</i>

<i>x</i>

=

<i>R</i>



2 2 2 2 2


2


<i>R</i>



<i>x</i>

<i>R</i>

<i>x</i>

<i>x</i>

<i>R</i>

<i>x</i>

<i>x</i>



⇔ =

=

⇔ =



2

2;



2


<i>R</i>



<i>MN</i>

<i>x</i>

<i>R</i>

<i>MQ</i>

<i>x</i>



=

=

= =



(

2 2

)

<sub>(</sub>

<sub>2</sub> <sub>2</sub>

<sub>)</sub>



2 2


2 2



1 1 2


27


5 2 2 <sub>27. 5</sub> <sub>2</sub> <sub>2</sub>


1 27 1 1


.


2 5 2 2 27


5 2 2


<i>a</i> <i>ab</i> <i>b</i> <i><sub>a</sub></i> <i><sub>ab</sub></i> <i><sub>b</sub></i>


<i>a</i> <i>ab</i> <i>b</i>


<i>a</i> <i>ab</i> <i>b</i>


+ ≥


+ + <sub>+</sub> <sub>+</sub>


 


⇔ ≤ <sub></sub> + <sub></sub>


+ +



 


+ +


2 2 2 2


2 2 2 2


1 27 1 1 1 27 1 1


;


2 5 2 2 27 2 5 2 2 27


5<i>b</i> 2<i>bc</i> 2<i>c</i> <i>b</i> <i>bc</i> <i>c</i> 5<i>c</i> 2<i>ca</i> 2<i>a</i> <i>c</i> <i>ca</i> <i>c</i>


   


≤ <sub></sub> + <sub></sub> ≤ <sub></sub> + <sub></sub>


+ + + +


   


+ + + +


(

2 2

) (

2 2

) (

2 2

)



27 1 1 1 1



.


2 5 2 2 5 2 2 5 2 2 9


<i>P</i>


<i>a</i> <i>ab</i> <i>b</i> <i>b</i> <i>bc</i> <i>c</i> <i>c</i> <i>ca</i> <i>a</i>


 


 


⇒ ≤ + + +


 <sub>+</sub> <sub>+</sub> <sub>+</sub> <sub>+</sub> <sub>+</sub> <sub>+</sub> 


</div>
<span class='text_page_counter'>(30)</span><div class='page_container' data-page=30>

Sử dụng BĐT ta có:


Ta lại có :


Chứng minh tương tự:


Dấu xảy ra . Vậy


<b>Đề</b>

<b> s</b>

<b>ố</b>

<b> 5 </b>



<b>Câu 1. </b>Cho



2
1


1 3 5


1
1 <sub>1</sub> <sub>4</sub>


<i>x</i>
<i>x</i>


<i>A</i>


<i>x</i> <i><sub>x x</sub></i> <i><sub>x</sub></i> <i><sub>x</sub></i> <i><sub>x</sub></i>


 
 
 <sub></sub> <sub></sub> 
 <sub></sub> 

<sub></sub>  <sub> </sub><sub></sub>  <sub></sub>
  
     
 <sub> </sub> <sub></sub>
 


với <i>x</i> 0;<i>x</i> 1.


a) Với <i>x</i> 0;<i>x</i> 1 ta có


1 1 1 1 1


9



<i>x</i> <i>y</i> <i>z</i> <i>x</i> <i>y</i> <i>z</i>


 


≤ <sub></sub> + + <sub></sub>


+ + <sub></sub> <sub></sub>


(

2 2

)

2

(

2

) (

2 2

)

2 2 2 2


2 2 2 2 2


2 2


1 1 1 1 1 1


9 3 2 2


5 2 2 3 2 2


1 1 1 1 1 1 1 1 1 1


. .


9 3 9 9


1 5 2 2


9 9 9 9



<i>a</i> <i>ab</i> <i>a</i> <i>a</i> <i>b</i>


<i>a</i> <i>ab</i> <i>b</i> <i>a</i> <i>ab</i> <i>a</i> <i>a</i> <i>b</i>


<i>a</i> <i>ab</i> <i>ab</i> <i>a</i> <i>a</i> <i>b</i> <i>b</i>


<i>a</i> <i>ab</i> <i>b</i>


 
= ≤ <sub></sub> + + <sub></sub>
+ +
+ + + + + +  
    
≤ <sub></sub> + <sub></sub> + + <sub></sub>+ <sub></sub> + + <sub></sub><sub></sub>
   
 
 
= <sub></sub> + + <sub></sub>
 
2 2


2 1 1 1


9 9


<i>Cauchy</i>


<i>ab</i> <i>a</i> <i>b</i>



 


≤ <sub></sub> + <sub></sub>


 


2 2 2 2 2 2 2 2


1 1 5 1 1 2 1 2 1


. .


5<i>a</i> 2<i>ab</i> 2<i>b</i> 9 9<i>a</i> 9<i>a</i> 9<i>a</i> 9<i>b</i> 9 3<i>a</i> 3<i>b</i>


   


⇒ ≤ <sub></sub> + + + <sub></sub>= <sub></sub> + <sub></sub>


+ + <sub></sub> <sub></sub> <sub></sub> <sub></sub>


2 2 2 2


2 2 2 2


1 1 2 1


5 2 2 9 3 3


1 1 2 1



5 2 2 9 3 3


<i>b</i> <i>bc</i> <i>c</i> <i>b</i> <i>c</i>


<i>c</i> <i>ca</i> <i>a</i> <i>c</i> <i>a</i>


 
≤ <sub></sub> + <sub></sub>
+ +  
 
≤ <sub></sub> + <sub></sub>
+ +  


2 2 2 2 2 2


2 2 2 2 2 2 2 2 2


1 1 1


5 2 5 2 2 5 2 2


1 2 1 1 2 1 1 2 1 1 1 1 1 1


. . .


9 3 3 9 3 3 9 3 3 9 9


27 1 1 3


.



2 9 9 3


<i>a</i> <i>ab</i> <i>b</i> <i>b</i> <i>bc</i> <i>c</i> <i>c</i> <i>ca</i> <i>a</i>


<i>a</i> <i>b</i> <i>b</i> <i>c</i> <i>c</i> <i>a</i> <i>a</i> <i>b</i> <i>c</i>


<i>P</i>
⇒ + +
+ + + + + +
       
≤ <sub></sub> + <sub></sub>+ <sub></sub> + <sub></sub>+ <sub></sub> + <sub></sub>= <sub></sub> + + <sub></sub>=
       
 
⇒ ≤ <sub></sub> + <sub></sub>=
 


"="


2 2 2


3


1 1 1


1


<i>a</i> <i>b</i> <i>c</i>


<i>a</i> <i>b</i> <i>c</i>



<i>a</i> <i>b</i> <i>c</i>


</div>
<span class='text_page_counter'>(31)</span><div class='page_container' data-page=31>

















2 2


2 2


2


1 1 4


1 3 5 1 3 5


1


1 <sub>1</sub> <sub>4</sub> 1 <sub>1</sub> <sub>1</sub> <sub>4</sub>



1 4 1 1


1 3 5 1




4 4


1 1 <sub>1</sub> <sub>1</sub>


<i>x</i> <i>x</i> <i>x</i>


<i>x</i> <i>x</i>


<i>A</i>


<i>x</i> <i><sub>x x</sub></i> <i><sub>x</sub></i> <i><sub>x</sub></i> <i><sub>x</sub></i> <i>x</i> <i><sub>x</sub></i> <i><sub>x</sub></i> <i><sub>x</sub></i>


<i>x</i> <i>x</i> <i>x</i>


<i>x</i> <i>x</i>


<i>x</i> <i>x</i> <i>x</i>


<i>x</i> <i>x</i> <i><sub>x</sub></i> <i><sub>x</sub></i>


  <sub></sub> <sub></sub>


 



 <sub></sub> <sub></sub>  <sub></sub> <sub></sub> <sub></sub>  


 <sub></sub>  <sub></sub> <sub></sub>




<sub></sub>  <sub> </sub><sub></sub>  <sub></sub> <sub></sub>  <sub></sub>


   


       


 <sub></sub> <sub></sub>  


 


 


  


  


    


  <sub></sub> <sub></sub>


Vậy <i>A</i> 1
<i>x</i>



 với <i>x</i> 0;<i>x</i> 1.


b) Đặt <i>B</i> 

<i>x</i>  <i>x</i> 1

<i>A</i>. Chứng minh rằng <i>B</i> 1 với <i>x</i> 0;<i>x</i> 1.


Với <i>x</i> 0;<i>x</i> 1 ta có



2


1
1


1 <i>x</i> <i>x</i> <i>x</i> <i>x</i> <i>x</i> 1


<i>B</i> <i>x</i> <i>x</i> <i>A</i>


<i>x</i> <i>x</i> <i>x</i>


 
 


       .


<b>Câu 2. </b>Trong mặt phẳng tọa độ Oxy, cho Parabol (P): <i><sub>y</sub></i> <sub></sub><i><sub>x</sub></i>2 <sub>và đường thẳng (d): </sub>


2 2 8


<i>y</i>  <i>x</i>  <i>m</i> (với m là tham số).


a) Khi <i>m</i>  4, tìm tọa độ giao điểm của đường thẳng (d) và Parabol (P) .



Xét phương trình hồnh độ giao điểm của

 

<i>P</i> và

 

<i>d</i> là


2 <sub>2</sub> <sub>2</sub> <sub>8</sub> 2 <sub>2</sub> <sub>2</sub> <sub>8</sub> <sub>0</sub>


<i>x</i>  <i>mx</i>  <i>m</i> <i>x</i>  <i>mx</i>  <i>m</i> 


Khi m = – 4, phương trình trên trở thành 2 <sub>8</sub> <sub>0</sub> 0
8


<i>x</i>


<i>x</i> <i>x</i>


<i>x</i>


 


 <sub>    </sub>





Với <i>x</i> 0 thì <i>y</i> 0 và với <i>x</i>  8 thì <i>y</i> 64


Vậy khi m = – 4 thì tọa độ giao điểm của

 

<i>P</i> và

 

<i>d</i> là

 

0;0 và

8;64

.


b) Chứng minh đường thẳng

 

<i>d</i> và Parabol

 

<i>P</i> ln cắt nhau tại hai điểm phân biệt có
hồnh độ <i>x x</i><sub>1</sub>; <sub>2</sub>. Tìm các giá trị của m để x<sub>1</sub> 2x<sub>2</sub> 2.


Xét phương trình hồn độ giao điểm <i><sub>x</sub></i>2 <sub></sub><sub>2</sub><i><sub>mx</sub></i><sub></sub><sub>2</sub><i><sub>m</sub></i><sub> </sub><sub>8</sub> <sub>0</sub><sub>. </sub>



Ta có <sub> </sub>' <i><sub>m</sub></i>2 <sub></sub><sub>2</sub><i><sub>m</sub></i><sub> </sub><sub>8</sub>

<i><sub>m</sub></i><sub></sub><sub>1</sub>

2 <sub> </sub><sub>7</sub> <sub>0</sub> <sub>với mọi m, do đó phương trình hồnh độ ln </sub>
có 2 nghiệm phân biệt. Suy ra

 

<i>d</i> và

 

<i>P</i> tại hai điểm phân biệt.


Theo hệ thức Vi – et 1 2
1 2


2
2 8


<i>x</i> <i>x</i> <i>m</i>


<i>x x</i> <i>m</i>


  



   





</div>
<span class='text_page_counter'>(32)</span><div class='page_container' data-page=32>

1 2 1


1 2 2


2 2 2
2 2 4 2


<i>x</i> <i>x</i> <i>m</i> <i>x</i> <i>m</i>



<i>x</i> <i>x</i> <i>x</i> <i>m</i>


 


     


 <sub></sub>


 


     


 


 


Thay vào hệ thức <i>x x</i><sub>1 2</sub>  2<i>m</i>8 ta được


<sub>2 2</sub><sub></sub> <i><sub>m</sub></i>



<sub>4</sub><i><sub>m</sub></i><sub></sub><sub>2</sub>

<sub> </sub><sub>2</sub><i><sub>m</sub></i><sub>  </sub><sub>8</sub> <sub>4</sub><i><sub>m</sub></i>2 <sub></sub><sub>7</sub><i><sub>m</sub></i><sub> </sub><sub>2</sub> <sub>0</sub>
Giải phương trình được <i>x</i> 0. Vậy 2; 1


4


<i>m</i> <sub></sub>  <sub></sub>


 


 


  là các giá trị cần tìm.



<b>Câu 3. Giải hệ phương trình </b>


2 2 <sub>2</sub> 2 <sub>3</sub>


4 1 0


<i>xy</i> <i>y</i> <i>x</i> <i>x</i>


<i>x</i> <i>y</i> <i>y</i>


    



    



 <b>Lời giải. </b>Điều kiện xác dịnh của hệ phương trình <i>y</i> 1. Phương trình thứ nhất tương
đương với


 







2 2 2 2


2


2



3 2 0 1 1 2 0
1


1 2 0


2


<i>xy</i> <i>y</i> <i>x</i> <i>x</i> <i>y x</i> <i>x</i> <i>x</i>


<i>x</i>


<i>x</i> <i>y</i> <i>x</i>


<i>x</i> <i>y</i>


          


  


    <sub>    </sub>





+ Thay <i>x</i>  1 vào phương trình thứ hai của hệ ta được


1 4 1 0 1

1 4

0 1 0 1


17



1 4


<i>y</i> <i>y</i>


<i>y</i> <i>y</i> <i>y</i> <i>y</i>


<i>y</i>
<i>y</i>


 <sub> </sub> <sub></sub> <sub></sub>


 <sub></sub>


          <sub></sub>  <sub> </sub>
   <sub></sub>


+ Thay <i><sub>x</sub></i> <sub></sub><i><sub>y</sub></i>2 <sub></sub><sub>2</sub> <sub>vào phương trình thứ hai của hệ ta được </sub><i><sub>y</sub></i>2 <sub>  </sub><sub>2</sub> <i><sub>y</sub></i> <sub>4</sub> <i><sub>y</sub></i><sub> </sub><sub>1</sub> <sub>0</sub><sub>. </sub>
Đặt <i><sub>y</sub></i><sub> </sub><sub>1</sub> <i><sub>a a</sub></i>

<sub></sub><sub>0</sub>

<sub> </sub><i><sub>y</sub></i> <i><sub>a</sub></i>2 <sub></sub><sub>1</sub><sub>, khi đó phương trình trên trở thành</sub>






2


2 2 4 2


3 2



1 2 1 4 0 3 4 0
0
3 4 0 1 4 0


1


<i>a</i> <i>a</i> <i>a</i> <i>a</i> <i>a</i> <i>a</i>


<i>a</i>


<i>a a</i> <i>a</i> <i>a a</i> <i>a</i> <i>a</i>


<i>a</i>


         


 


        <sub>   </sub>





Với <i>a</i> 0 ta được 2 2 1
1
1


<i>x</i>
<i>x</i> <i>y</i>



<i>y</i>
<i>y</i>


 


     


 


 <sub></sub>


 


   


 <sub></sub>





Với <i>a</i> 1 ta được


2 <sub>2</sub> <sub>0</sub>


2
2


<i>x</i>
<i>x</i> <i>y</i>


<i>y</i>


<i>y</i>


 


    


 


 <sub></sub>


 


   


 <sub></sub>





</div>
<span class='text_page_counter'>(33)</span><div class='page_container' data-page=33>

tốc của mỗi xe, biết thời gian đi cả quãng đường AB của xe thứ nhất nhiều hơn xe thứ hai
là 2 giờ 30 phút.


 <b>Lời giải. </b>Đổi 2 30 ' 5
2


<i>h</i>  <i>h</i>. Gọi vận tốc của xe thứ nhất và xe thứ hai lần lượt là




x; y km/h . Điều kiện <i>y</i> <i>x</i> 0.



Sau 3 giờ thì xe thứ nhất đi được 3<i>x km</i>

 

và xe thứ nhất đi được 3<i>y km</i>

 



Ta có phương trình 3<i>x</i> 3<i>y</i> 300  <i>x</i> <i>y</i> 100


Thời gian xe thứ nhất đi hết quãng đường AB là 300

 

<i>h</i>


<i>x</i> , thời gian xe thứ hai đi hết quãng


đường AB là 300

 

<i>h</i>


<i>y</i> . Ta có phương trình


300 300 5 60 60 1
2 2


<i>x</i>  <i>y</i>   <i>x</i>  <i>y</i> 


Kết hợp các phương trình ta có hệ phương trình


100 100


60 60 1 60 60 1
2 100 2


<i>x</i> <i>y</i> <i>y</i> <i>x</i>


<i>x</i> <i>y</i> <i>x</i> <i>x</i>


 



     


 


 


 <sub></sub>


 


     


 


 <sub></sub> <sub></sub>





Giải phương trình 60 60 1
100 2


<i>x</i>  <i>x</i>  ta được x 300; x40


+ Với <i>x</i> 300 ta được <i>y</i>  200(không thỏa mãn điều kiện)


+ Với <i>x</i> 40 ta được <i>y</i> 60(thỏa mãn điều kiện)


Vậy vận tốc của xe thứ nhất và xe thứ hai lần lượt là 40 km/h và 60 km/h.


<b>Câu 5. Cho đường trịn </b>

 

<i>O R</i>; có đường kính AB. Điểm C là điểm bất kỳ trên đường tròn


 

<i>O</i> , C không trùng với A, B. Tiếp tuyến tại C của

 

<i>O R</i>; cắt tiếp tuyến tại A, B của

 

<i>O R</i>;
lần lượt tại P, Q. Gọi M là giao điểm của OP với AC, N là giao điểm của OQ với BC.


a) Chứng minh tứ giác CMON là hình chữ
nhật và <sub>AP.BQ</sub><sub></sub><sub>MN</sub>2<sub>. </sub>


Ta có <i>OA</i><i>OC</i> <i>R</i> và <i>PA</i><i>PC</i> (tính
chất của hai tiếp tuyến cắt nhau). Suy ra
OP là đường trung trực của AC, do đó


 <sub>90</sub>0


<i>OP</i> <i>AC</i> <i>OMC</i> 


Chứng minh tương tự được <i><sub>ONC</sub></i> <sub></sub><sub>90</sub>0


E


O
N
M


Q


P


I
D



C


</div>
<span class='text_page_counter'>(34)</span><div class='page_container' data-page=34>

Lại có <i><sub>ACB</sub></i> <sub></sub><sub>90</sub>0<sub>. </sub>
Tứ giác CMON có


   <sub>90</sub>0


<i>OMC</i> <i>ONC</i> <i>MCN</i>  nên tứ giác
CMON là hình chữ nhật.


Vì CMON là hình chữ nhật nên <i><sub>POQ</sub></i> <sub></sub><sub>90</sub>0 <sub>và vì PQ là tiếp tuyến tại C của </sub>

 

<i><sub>O</sub></i> <sub> nên </sub>


<i>OC</i> <i>PQ</i>


Tam giác OPQ vng tại O có đường cao OC nên ta được <sub>PC.QC</sub><sub></sub><sub>OC</sub>2
Mà PAPC;QBQC và <i>MN</i> <i>OC</i>. Từ đó ta suy ra được <i><sub>AP BQ</sub></i><sub>.</sub> <sub></sub><i><sub>MN</sub></i>2<sub>. </sub>
b) Chứng minhAB là tiếp tuyến của đường trịn đường kính PQ.


Gọi I là trung điểm của PQ. Tam giác OPQ vuông tại O có OI là đường trung tuyến
Do đó ta có


2


<i>PQ</i>


<i>OI</i>  nên O thuộc đường tròn ;
2


<i>PQ</i>
<i>I</i>



 <sub></sub>


 <sub></sub>


 <sub></sub>


 <sub></sub>


 .


Vì AP và BQ là các tiếp tuyến của

 

<i>O</i> nên <i>AP</i> <i>AB BQ</i> , <i>AB</i> nên APQB là hình thang
vng


Mà OI là đường trung bình của hình thang APQB, do đó OI//AP nên suy ra <i>OI</i> <i>AB</i>


Do đó AB là tiếp tuyến tại O của ;
2


<i>PQ</i>
<i>I</i>


 <sub></sub>


 <sub></sub>


 <sub></sub>


 <sub></sub>



 .


c) Chứng minh PMNQ là tứ giác nội tiếp. Xác định vị trí điểm C để đường trịn ngoại tiếp
tứ giác PMNQ có bán kính nhỏ nhất.


+ Tam giác OCP vng tại C có đường cao CM nên ta được <sub>OC</sub>2 <sub></sub><sub>OM.OP</sub>
Tương tự ta có <sub>OC</sub>2 <sub></sub><sub>ON.OQ</sub><sub>. Từ đó ta được </sub><i><sub>OM OP</sub></i><sub>.</sub> <i><sub>ON OQ</sub></i><sub>.</sub> <i>OM</i> <i>ON</i>


<i>OQ</i> <i>OP</i>


  


Hai tam giác OMN và OQP có <i>POQ</i> chung và <i>OM</i> <i>ON</i>


<i>OQ</i> <i>OP</i>


Do đó OMN∽OQP suy ra ta được <i>OPQ</i> <i>ONM</i> nên tứ giác PMNQ nội tiếp đường
trịn.


</div>
<span class='text_page_counter'>(35)</span><div class='page_container' data-page=35>

Do đó <i>DI</i> <i>PQ DE</i>; <i>MN</i> nên ta được DI//OE và DE//OI, suy ra tứ giác OEDI là hình


bình hành.
Từ đó ta được


2


<i>R</i>
<i>DI</i> <i>OE</i>  .


+ Áp dụng định lí Pitago cho tam giác vng DIPta được


1, 2


<i>x x</i>


Dấu bằng xẩy ra khi <i>PQ</i> <i>AB</i> <i>OC</i> <i>AB</i> hay C là điểm chính giữa của nửa đường
trịn

 

<i>O</i> .


Vậy khi C là điểm chính giữa của nửa đường trịn

 

<i>O</i> thì đường trịn ngoại tiếp tứ giác
PMNQ có bán kính nhỏ nhất bằng 5


2


<i>R</i> <sub>. </sub>


<b>Câu 6. Cho ba số thực dương x, y, z thỏa mãn </b><sub>8</sub><i><sub>t</sub></i>2<sub></sub><i><sub>mt</sub></i><sub> </sub><sub>1</sub> <sub>0</sub>. Tìm giá trị nhỏ nhất của
biểu thức:


 

 



2 2 2 2 2 2


2 2 2 2 2 2


<i>y z</i> <i>z x</i> <i>x y</i>


<i>P</i>


<i>x y</i> <i>z</i> <i>y z</i> <i>x</i> <i>z x</i> <i>y</i>


  



   .


 <b>Lời giải. Ta có </b>


2 2 2 2 2 2


1 1 1


1 1 1 1 1 1


<i>P</i>


<i>x</i> <i>y</i> <i>z</i>


<i>y</i> <i>z</i> <i>x</i> <i>z</i> <i>x</i> <i>y</i>


 <sub></sub> <sub></sub> <sub></sub> <sub></sub> <sub></sub> <sub></sub>


  


 <sub></sub> <sub></sub>  <sub></sub> <sub></sub>  <sub></sub> <sub></sub>


 <sub></sub>  <sub></sub>  <sub></sub>


     


  


     





Đặt 1 <i>a</i>;1 <i>b</i>;1 <i>c</i>


<i>x</i>  <i>y</i>  <i>z</i>  , khi đó ta được 2 2 2 2 2 2


<i>a</i> <i>b</i> <i>c</i>


<i>P</i>


<i>b</i> <i>c</i> <i>a</i> <i>c</i> <i>a</i> <i>b</i>


  


   và


2 2 2 <sub>3</sub>


<i>a</i> <i>b</i> <i>c</i> 
Do đó <i>n</i> 3


Ta lại có bất đẳng thức

 





2
2


2 2



1 . . 2


1 <sub>0</sub>


2


3 2. 3


<i>x</i> <i>x x</i>


<i>x</i> <i><sub>x</sub></i>


<i>x</i> <i>x</i>


 


  


  (luôn đúng)


Suy ra 1 2 1 2 1 2 3


2 2 2 2


<i>P</i>  <i>a</i>  <i>b</i>  <i>c</i>  . Dấu bằng xẩy ra khi và chỉ khi


1 1


<i>a</i>       <i>b</i> <i>c</i> <i>x</i> <i>y</i> <i>z</i> .



Vậy P đạt giá trị nhỏ nhất là 3


2, đạt được tại <i>x</i>   <i>y</i> <i>z</i> 1.


<b>Đề</b>

<b> s</b>

<b>ố</b>

<b> 6 </b>



<b>Câu 1(2.0 điểm).</b>


</div>
<span class='text_page_counter'>(36)</span><div class='page_container' data-page=36>

 
<i>OMN</i> <i>BAC</i>


 <b>Phân tích và lời giải. </b>Đặt các phương trình đã cho là


2 2 2


2 2


2 2 1 0(1)
2 3 0(2)


<i>x</i> <i>ax</i> <i>a</i> <i>b</i>


<i>x</i> <i>bx</i> <i>b</i> <i>ab</i>


    


   


Để chứng minh có ít nhất một trong hai phương trình ẩn x sau vơ nghiệm thì thì ta


chứng minh một trong hai biệt thức delta của phương trình nhận giá trị âm. Muốn vậy ta
đi tính tổng hai biệt thức delta của hai hai phương trình nhận giá trị âm. Ta có






' 2 2 2 2 2


1


' 2 2 2


2


2 1 1


3 2


<i>a</i> <i>a</i> <i>b</i> <i>a</i> <i>b</i>


<i>b</i> <i>b</i> <i>ab</i> <i>b</i> <i>ab</i>


        



      



Khi đó ta được ' ' 2 2 2 2 2



1 2 <i>a</i> <i>b</i> 1 2<i>b</i> <i>ab</i> <i>a</i> <i>b</i> 1 <i>ab</i>


             


Mặt khác dễ thấy rằng <sub></sub><sub>2</sub><i><sub>a</sub></i>2 <sub></sub><sub>2</sub><i><sub>b</sub></i>2 <sub> </sub><sub>2</sub> <sub>2</sub><i><sub>ab</sub></i><sub>  </sub>

<i><sub>a b</sub></i>

2<sub> </sub><sub>2</sub> <i><sub>a</sub></i>2 <sub></sub><i><sub>b</sub></i>2 <sub></sub><sub>0</sub>
Do đó ta được ' '


1 2 0


    . Vậy trong hai phương trình tồn tại ít nhất một phương trình
vơ nghiệm.


2) Cho ba số thực x, y, z thỏa mãn các điều kiện <i>x</i>   <i>y</i> <i>z</i> 0 và <i>xyz</i> 0
Tính giá trị biểu thức <i>P</i> <sub>2</sub> <i>x</i><sub>2</sub>2 <sub>2</sub> <sub>2</sub> <i>y</i>2<sub>2</sub> <sub>2</sub> <sub>2</sub> <i>z</i>2<sub>2</sub> <sub>2</sub>


<i>y</i> <i>z</i> <i>x</i> <i>z</i> <i>x</i> <i>y</i> <i>x</i> <i>y</i> <i>z</i>


  


     


 <b>Phân tích và lời giải. </b>Để tính được <i><sub>y</sub></i>2 <sub></sub><i><sub>z</sub></i>2 <sub></sub><i><sub>x</sub></i>2 <sub>ta để ý đến giả thiết </sub>
0


<i>x</i>       <i>y</i> <i>z</i> <i>y</i> <i>z</i> <i>x</i> . Khi đó thực hiện bình phương hai vế ta được


2 2 <sub>2</sub> 2 2 2 2 <sub>2</sub>


<i>y</i> <i>z</i>  <i>yz</i> <i>x</i> <i>y</i> <i>z</i> <i>x</i>   <i>yz</i>.



Từ đó ta có biến đổi <sub>2</sub> <sub>2</sub>2 <sub>2</sub> <sub>2</sub> <sub>2</sub> <sub>2</sub>2 <sub>2</sub> 2


2
2


<i>x</i> <i>x</i> <i>x</i>


<i>yz</i>
<i>y</i> <i>z</i> <i>x</i>  <i>y</i> <i>z</i> <i>y</i> <i>z</i>  <i>yz</i> 
Hoàn tồn tương tự ta cũng có <sub>2</sub> 2<sub>2</sub> <sub>2</sub> 2


2


<i>y</i> <i>y</i>


<i>zx</i>
<i>z</i> <i>x</i> <i>y</i>   ;


2 2


2 2 2 <sub>2</sub>


<i>z</i> <i>z</i>


</div>
<span class='text_page_counter'>(37)</span><div class='page_container' data-page=37>



2 2 2 2 2 2


2 2 2 2 2 2 2 2 2



3 3 3 3 3 3 3 3


2 2 2


3


2 2 2


1 1 1


1 <sub>3</sub>


2


3


3 3 3


2 2 2


3
2


<i>x</i> <i>y</i> <i>z</i> <i>x</i> <i>y</i> <i>z</i>


<i>P</i>


<i>yz</i> <i>xz</i> <i>xy</i>



<i>y</i> <i>z</i> <i>x</i> <i>z</i> <i>x</i> <i>y</i> <i>x</i> <i>y</i> <i>z</i>


<i>x</i> <i>y</i> <i>z</i> <i>x</i> <i>y</i> <i>z</i> <i>x</i> <i>y</i> <i>z</i> <i>xyz</i>


<i>xyz</i> <i>xyz</i>


<i>x</i> <i>y</i> <i>z x</i> <i>y</i> <i>z</i> <i>xy</i> <i>yz</i> <i>z</i>


<i>xyz</i>
<i>xyz</i>
<i>x</i>
     
  
     
 <sub></sub> <sub></sub> <sub></sub>  <sub></sub> <sub></sub> <sub></sub>  <sub></sub> <sub></sub> <sub></sub> <sub></sub>
 <sub></sub>  <sub></sub>  <sub></sub>
  <sub></sub> <sub></sub><sub></sub>  <sub></sub>   <sub></sub><sub></sub>  <sub></sub>  <sub></sub><sub></sub>
  
  
 <sub> </sub> <sub></sub> <sub></sub> <sub></sub> <sub></sub> <sub></sub> 
 
    
 
 
 
  
 


Vậy 3



2


<i>P</i>   .
<b>Câu 2(2.5 điểm). </b>


1) Giải phương trình 1 3 ;1 3 ;1 3 <i>x</i>  <i>y</i>  <i>z</i>


1; 0


<i>y</i> <i>x</i>  <i>z</i> <b>Phân tích và lời giải. </b> Điều kiện xác định của phương trình là


2 <sub>4</sub> <sub>12</sub> <sub>0;</sub> <sub>1</sub>


<i>x</i>  <i>x</i>   <i>x</i>   . quan sát phương trình đã cho ta để ý đến biến đổi


2



2 <sub>4</sub> <sub>12</sub> <sub>2</sub> <sub>8</sub> <sub>1</sub>


<i>x</i>  <i>x</i>   <i>x</i>   <i>x</i>  . Khi đó ta viết lại được phương trình đã cho thành


2



2 8 1 2 2 1


<i>x</i>   <i>x</i>   <i>x</i>   <i>x</i>  . Phương trình có sự lặp lại của hai đại lương nên
ta sử dụng phép đặt ẩn phụ <i>x</i>  2 <i>a x</i>;  1 <i>b b</i>

0

. Khi đó phương trình trên trở
thành







2 2 2 2 2 2


2 2


8 2 8 4 4


3 4 7 0 3 7 0


3 7


<i>a</i> <i>b</i> <i>a</i> <i>b</i> <i>a</i> <i>b</i> <i>a</i> <i>ab</i> <i>b</i>


<i>a</i> <i>b</i>


<i>a</i> <i>ab</i> <i>b</i> <i>a</i> <i>b a b</i>


<i>a</i> <i>b</i>
       
 

       <sub>    </sub>



+ Với ( )

9 4 4

4 1 1
3


<i>f x</i>  <i>yz</i> <i>y</i> <i>z x</i> <i>yz</i> <sub></sub>  <i>x</i> <sub></sub><sub></sub>


  thì



2 2


2 0 2 <sub>5</sub> <sub>13</sub>
2 1


4 4 1 5 3 0 2


<i>x</i> <i>x</i>


<i>x</i> <i>x</i> <i>x</i>


<i>x</i> <i>x</i> <i>x</i> <i>x</i> <i>x</i>


 
     <sub></sub>
 
 
   <sub></sub> <sub></sub> <sub>  </sub> <sub></sub> <sub></sub> <sub> </sub>  
 
 
 

+ Với <i>g x</i>( )<i>ax</i> <i>b</i> thì


2 2


9 36 36 49 49 9 85 85 0
3 6 7 1



2 2


<i>x</i> <i>x</i> <i>x</i> <i>x</i> <i>x</i>


<i>x</i> <i>x</i>
<i>x</i> <i>x</i>
 
 <sub></sub> <sub></sub> <sub></sub> <sub></sub>  <sub></sub> <sub></sub> <sub></sub>
 
 
    <sub></sub> <sub></sub> <sub></sub> <sub></sub>
 
 
 

Hệ vô nghiệm với <i>x</i>  1


Thử lại vào phương trình đã cho ta có nghiệm của phương trình là <i>a</i>,.


2) Giải hệ phương trình



2 2


2 2


2


4 1 4 4


3 2 1 2 3



<i>x</i> <i>y</i> <i>xy</i> <i>xy</i>


<i>x</i> <i>y</i>


<i>x</i> <i>y</i> <i>y</i> <i>y</i> <i>y</i> <i>x</i>


</div>
<span class='text_page_counter'>(38)</span><div class='page_container' data-page=38>

 <b>Phân tích và lời giải. </b>Điều kiện xác định của hệ phương trình là <i><sub>x</sub></i> <sub></sub><i><sub>y y</sub></i><sub>;2</sub> 2 <sub>  </sub><i><sub>y</sub></i> <sub>1</sub> <sub>0</sub><sub>. </sub>
Quan sát phương trình thứ nhất của hệ ta nhận thấy phương trình có thể đưa về dạng
phương trình đa thức nên ta kiểm tra xem phương trình có phân tích được hay khơng.
Chú ý rằng trong phương trình có sự lặp lại của hai đại lượng là <i>x</i> <i>y</i> và <i>xy</i> nên ta viết


phương trình về dạng


2 2



2 4 1 4 4


<i>x</i> <i>y</i> <i>xy</i> <i>xy</i> <i>xy</i>


<i>x</i> <i>y</i>


 <sub></sub>


 <sub></sub>


   <sub></sub><sub> </sub>  <sub></sub><sub></sub> 


 



Đến đây ta sử dụng phép đặt ẩn phụ <i>x</i>  <i>y</i> <i>a xy</i>; <i>b</i>. Khi đó ta có phương trình




 



 



2 3 3


2 2


2


2


2 4 1 4 4 2 8 16 2 16 8 0


4


4 4 4 2 4 2 4 0


2 4 0


<i>a</i> <i>b</i> <i>b</i> <i>b</i> <i>a</i> <i>ab</i> <i>b</i> <i>a</i> <i>a</i> <i>ab</i> <i>a</i> <i>b</i>


<i>a</i>


<i>a</i>


<i>a a</i> <i>a</i> <i>a</i> <i>b</i> <i>a</i> <i>a</i> <i>b</i> <i>a</i>



<i>a</i> <i>b</i> <i>a</i>


 <sub></sub>
 <sub></sub>


  <sub></sub>  <sub></sub><sub></sub>          
 


 


         <sub>     </sub>



+ Với PDI = BAH  ta được <i>x</i>  <i>y</i> 4 hay <i>x</i>  <i>y</i> 4, thế vào phương trình thứ hai củahệ


ta được


2 2


3 2<i>y</i>   <i>y</i> 1 2<i>y</i>  <i>y</i> 3
Đặt 2 2.


.


<i>R</i> <i>OI</i>
<i>IM IK</i>


 <sub>, khi đó phương trình trên được viết lại thành</sub>



2 2 2 5


3 4 4 2 1 4 2 15 0 3;
2


<i>t</i> <i>t</i>    <i>t</i> <i>y</i>    <i>y</i> <i>y</i>  <i>y</i>   <i>y</i> <sub></sub>  <sub></sub>


 


 


Với <i>y</i> 3 ta được <i>x</i> 7 và với 5
2


<i>y</i>   ta được 3
2


<i>x</i>  . Các nghiệm thỏa mãn hệ phương
trình.


+ Với <i><sub>a</sub></i>2 <sub></sub><sub>2</sub><i><sub>b</sub></i><sub></sub><sub>4</sub><i><sub>a</sub></i> <sub></sub><sub>0</sub> <sub>ta được </sub><i><sub>x</sub></i>2 <sub></sub><i><sub>y</sub></i>2 <sub></sub><sub>4</sub>

<i><sub>x</sub></i> <sub></sub><i><sub>y</sub></i>

<sub></sub><sub>0</sub>


Vì <i><sub>x</sub></i> <sub></sub><i><sub>y x</sub></i><sub>;</sub> 2 <sub></sub><sub>0;</sub><i><sub>y</sub></i>2 <sub></sub><sub>0</sub><sub> nên </sub><i><sub>x</sub></i>2 <sub></sub><i><sub>y</sub></i>2 <sub></sub><sub>4</sub>

<i><sub>x</sub></i> <sub></sub><i><sub>y</sub></i>

<sub></sub><sub>0</sub> <sub>hay trường hợp này hệ phương trình vơ </sub>
nghiệm


Vậy các nghiệm của hệ phương trình là

   

; 7;3 , 3; 5
2 2


<i>x y</i>  <sub></sub>  <sub></sub><sub></sub>
 .



</div>
<span class='text_page_counter'>(39)</span><div class='page_container' data-page=39>

Đặt <i>x</i>  <i>y</i> <i>a xy</i>; <i>b</i>, do x, y là số ngun nên <i>a b</i>, <i>Z</i>. Khi đó phương trình trở thành






3 3


2 2


3 6 3 3 3 2


5


2 2 4 5 3 2 2 4 3


2


<i>a</i> <i>ab</i> <i>b</i> <i>a</i> <i>b a</i>


<i>a</i> <i>a</i> <i>a</i> <i>b a</i> <i>a</i> <i>a</i> <i>b</i>


<i>a</i>


       


             



Vì a, b là các số nguyên nguyên nên <i>a</i> 2 <i>U</i>(5)    <i>a</i> 2

5; 1;1;5

 <i>a</i>

7;3;1; 3

.
+ Với <i>a</i> 7 khi đó ta được 68


3


<i>b</i>   (khơng phải số nguyên)
+ Với <sub>( ) :</sub><i><sub>P</sub></i> <i><sub>y</sub></i> <sub></sub><sub>2</sub><i><sub>x</sub></i>2 <sub>khi đó ta được </sub><i><sub>b</sub></i> <sub> </sub><sub>8</sub><sub>, do đó ta được</sub>


2
3
3


8 3 8 0


<i>x</i> <i>y</i>


<i>x</i> <i>y</i>


<i>xy</i> <i>y</i> <i>y</i>




 


   <sub></sub>  


 <sub></sub>


 



      


 


 


(hệ
vơ nghiệm)


+ Với <i>a</i> 1 khi đó ta được 2
3


<i>b</i>  (không phải số nguyên)


+ Với <i>a</i> <b><sub></sub></b> khi đó ta được<i>b</i>  2 , suy ra 3 1; 2
2 2; 1


<i>x</i> <i>y</i> <i>x</i> <i>y</i>


<i>xy</i> <i>x</i> <i>y</i>


 


      


 <sub></sub> 


 <sub></sub>


   <sub></sub>   



 


Vậy các nghiệm nguyên của phương trình là

  

<i>x y</i>;  1;2 ,

 

2;1



<b>Câu 4. Cho tứ giác ABCD nội tiếp đường trịn tâm O và có hai tia BA và CD cắt nhau tại E, </b>
hai tia AD và BC cắt nhau tại F. Gọi M, N lần lượt là trung điểm của AC và BD. Các đường
phân giác trong của các góc<i>BEC</i>và góc<i>BFA</i>cắt nhau tại K.


a) Chứng minh DEF<i>DFE</i> <i>ABC</i> và tam giác EKF là tam giác vng.
Vì tứ giác ABCD nội tiếp nên


 


<i>ABC</i> <i>ADE</i> mặt khác vì góc
ADE là góc ngồi của tam giác
EDF nên


 <sub>DE</sub> 


<i>ADE</i>  <i>F</i> <i>DFE</i>.
Từ đó suy ra


  


DEF<i>DFE</i> <i>ABC</i>


Ta có <i>FEK</i><i>KFE</i> DEF<i>DFE</i><i>DFK</i><i>DEK</i> <i>ABC</i>

<i>DFK</i><i>DEK</i>

(1)
Vì các đường phân giác của các góc<i>BEC</i>và góc <i>BFA</i>cắt nhau tại K nên



F


E


K N


M


O
D


C


</div>
<span class='text_page_counter'>(40)</span><div class='page_container' data-page=40>

  1

 

1

<sub>360</sub>0   <sub>2</sub>

1

<sub>180</sub>0 <sub>2</sub>



2 2 2


<i>DFK</i> <i>DEK</i>  <i>AFB</i><i>AEC</i>  <i>ECB</i><i>FAB</i> <i>ABC</i>   <i>ABC</i> (2)


Từ (1) và (2) ta có    1

<sub>180</sub>0 <sub>2</sub>

<sub>90</sub>0
2


<i>FEK</i> <i>KFE</i> <i>ABC</i>   <i>ABC</i> 


b) Ta có 4034 2017


2


<i>F</i>   



 (cùng chắn cung AD) và <i>E</i> chung nên <i>EAC</i> ∽<i>EDB</i>. Mà


ta lại có M, N lần lượt là trung điểm của AC, BD nên EM, EN lần lượt là đường trung
tuyến của tam giác EAC và EBD. Do đó ta có <i>AC</i> <i>EA</i> <i>CM</i>


<i>BD</i>  <i>ED</i>  <i>BN</i> .


Mặt khác ta có <i>ECM</i> <i>EBN</i> nên tam giác EBN và ECM đồng dạng với nhau.
Từ đó ta được<i>EN</i> <i>BN</i> <i>BD</i>


<i>EM</i> <i>CM</i>  <i>AC</i> nên suy ra EM.BDEN.AC


c) Gọi <i><sub>K</sub></i>' <sub>là giao điểm của EK với MN. Ta có tam giác EBN và ECM đồng dạng với nhau.</sub>
Nên suy ra CEM BEN. Lại do EK là phân giác của góc <i>AED</i> nên suy ra <i>MEK</i> <i>NEK</i>


hay <i><sub>MEK</sub></i>' <sub></sub><i><sub>NEK</sub></i>'<sub>. </sub>


Tam giác MEN có <i><sub>EK</sub></i>' là phân giác nên theo tính chất đường phân giác ta có '
'


<i>MK</i> <i>ME</i>


<i>NE</i>
<i>NK</i> 


.


Gọi <i><sub>K</sub></i>" <sub>là giao điểm của FK với MN, hồn tồn tương tự ta có </sub> "
"



<i>MK</i> <i>MF</i>


<i>NF</i>
<i>NK</i>  .


Theo ý b) ta đã có <i>EM</i> <i>AC</i>


<i>EN</i>  <i>BD</i> . Chứng minh hoàn toàn tương tự ta cũng có


<i>MF</i> <i>AC</i>


<i>NF</i>  <i>BD</i> .


Kết hợp các kết quả trên ta được <i>MK</i><sub>"</sub>" <i>MK</i><sub>'</sub>'


<i>NK</i>  <i>NK</i> . Điều này dẫn đến các điểm


' "


, ,


<i>K K K</i> trùng


nhau hay ba điểm M, K, N thẳng hàng.


<b>Câu 5 (2,0 điểm). </b>


1) Cho các số thực dương a, b, c. Chứng minh rằng
1 1 1 3



3 2 3 2 3 2 5


<i>a</i> <i>a</i>  <i>b</i> <i>b b</i> <i>c</i> <i>c c</i> <i>a</i>  <i>abc</i>


</div>
<span class='text_page_counter'>(41)</span><div class='page_container' data-page=41>

1 1 1 3
5
5 3 2 5 3 2 5 3 2


<i>a</i> <i>b</i> <i>c</i>


<i>c b</i> <i>c</i> <i>a c</i> <i>b</i> <i>b a</i> <i>c</i>


  


 <sub></sub>  <sub></sub>  <sub></sub>


 <sub></sub> <sub></sub>  <sub></sub> <sub></sub>  <sub></sub> <sub></sub>


 <sub></sub>  <sub></sub>  <sub></sub>


 <sub></sub>  <sub></sub>  <sub></sub>


  


     


Để đơn giản hoán bất đẳng thức cần chứng minh ta đặt AH2


2 . Khi đó ta đi chứng minh





3
5


5 3 2 5 3 2 5 3 2


<i>x</i> <i>y</i> <i>z</i>


<i>z x</i> <i>y</i>  <i>x y</i> <i>z</i>  <i>y z</i> <i>x</i> 


Dự đoán dấu bằng xẩy ra tại <i>x</i>  <i>y</i> <i>z</i> ta có đánh giá 5 3

2

1

5 3 2


2


<i>z x</i>  <i>y</i>  <i>z</i>  <i>x</i>  <i>y</i>


đến đây áp dụng tương tự ta quy bài toán về chứng minh
2 2 2 3


5 3 2 5 3 2 5 3 2 5


<i>x</i> <i>y</i> <i>z</i>


<i>z</i>  <i>x</i>  <i>y</i>  <i>y</i> <i>y</i>  <i>z</i>  <i>x</i>  <i>z</i>  <i>x</i>  . Dễ thấy bất đẳng thức trên áp dụng


được bất đẳn thức Bunhiacopxki dạng phân thức.


 <b>Lời giải. B</b>ất đẳng thức cần chứng minh được viết lại thành
1 1 1 3



5
5 3 2 5 3 2 5 3 2


<i>a</i> <i>b</i> <i>c</i>


<i>c b</i> <i>c</i> <i>a c</i> <i>b</i> <i>b a</i> <i>c</i>


  


 <sub></sub>  <sub></sub>  <sub></sub>


 <sub></sub> <sub></sub>  <sub></sub> <sub></sub>  <sub></sub> <sub></sub>


 <sub></sub>  <sub></sub>  <sub></sub>


     


  


     


Đặt AH2


2 . Khi đó ta đi chứng minh




3
5



5 3 2 5 3 2 5 3 2


<i>x</i> <i>y</i> <i>z</i>


<i>z x</i> <i>y</i>  <i>x y</i> <i>z</i>  <i>y z</i> <i>x</i> 


Áp dụng bất đẳng thức AM – GM ta có 5 3

2

1

5 3 2


2


<i>z x</i> <i>y</i>  <i>z</i>  <i>x</i>  <i>y</i> .


Do đó ta được 2


5 3 2
5 (3 2 )


<i>x</i> <i>x</i>


<i>z</i> <i>x</i> <i>y</i>


<i>z x</i>  <i>y</i>    . Áp dụng tương tự ta được




2 2 2


5 3 2 5 3 2 5 3 2


5 3 2 5 3 2 5 3 2



<i>x</i> <i>y</i> <i>z</i> <i>x</i> <i>y</i> <i>z</i>


<i>z</i> <i>x</i> <i>y</i> <i>y</i> <i>y</i> <i>z</i> <i>x</i> <i>z</i> <i>x</i>


<i>z x</i>  <i>y</i>  <i>x y</i> <i>z</i>  <i>y z</i>  <i>x</i>         


Ta cần chứng minh 2 2 2 3


5 3 2 5 3 2 5 3 2 5


<i>x</i> <i>y</i> <i>z</i>


</div>
<span class='text_page_counter'>(42)</span><div class='page_container' data-page=42>

Áp dụng bất đẳng thức Bunhiacopxki dạng phân thức và để ý


2
1


3


<i>xy</i><i>yz</i> <i>zx</i>  <i>x</i>  <i>y</i> <i>z</i> ta được




 

 

<sub></sub>

<sub></sub>



<sub></sub>

<sub></sub>

<sub></sub>

<sub></sub>



2



2


2 2 2


2 2


2 <sub>2</sub> <sub>2</sub>


2 2 2


5 3 2 5 3 2 5 3 2


2 <sub>2(</sub> <sub>)</sub>


5 3 2 5 3 2 5 3 2 3 7


2( ) 2( ) 3


1 5


3 <sub>3</sub>


3


<i>x</i> <i>y</i> <i>z</i>


<i>z</i> <i>x</i> <i>y</i> <i>y</i> <i>y</i> <i>z</i> <i>x</i> <i>z</i> <i>x</i>


<i>x</i> <i>y</i> <i>z</i> <i><sub>x</sub></i> <i><sub>y</sub></i> <i><sub>z</sub></i>



<i>x z</i> <i>x</i> <i>y</i> <i>y x</i> <i>y</i> <i>z</i> <i>z y</i> <i>z</i> <i>x</i> <i>x</i> <i>y</i> <i>z</i> <i>xy</i> <i>yz</i> <i>zx</i>


<i>x</i> <i>y</i> <i>z</i> <i>x</i> <i>y</i> <i>z</i>


<i>x</i> <i>y</i> <i>z</i> <i>xy</i> <i>yz</i> <i>zx</i> <i><sub>x</sub></i> <i><sub>y</sub></i> <i><sub>z</sub></i> <i><sub>x</sub></i> <i><sub>y</sub></i> <i><sub>z</sub></i>


 


     


  <sub> </sub>


 


            


   


  


     <sub> </sub> <sub></sub> <sub> </sub>


Vậy bất đẳng thức được chứng minh. Dấu đẳng thức xảy ra khi và chỉ khi <i>a</i>  <i>b</i> <i>c</i>


2) Cho 5 số tự nhiên phân biệt sao cho tổng của ba số bất kỳ trong chúng lớn hơn tổng của
hai số còn lại. Chứng minh rằng tất cả 5 số đã cho đều không nhỏ hơn 5.


<b> Phân tích. Gọi 5 số đã cho là a, b, c, d, e</b>( , , , ,<i>a b c d e</i> <i>N</i>). Để chứng minh tất cả năm số tự


nhiên đều lớn hơn hoặc bằng 5 ta đi chứng minh có lần lượt một số, hai số, ba số, bốn số,


năm số trong các số tự nhiên trên nhỏ hơn 5 thì đều có mâu thuẫn với u cầu bài tốn. Ta
có thể giả sử a   b c d e. Khi đó ta đi xét các trường hợp


5; , 5; , , 5; , , , 5; , , , , 5


<i>a</i>  <i>a b</i>  <i>a b c</i> <i>a b c d</i>  <i>a b c d e</i> . Chú ý rằng ta luôn có
1 2 3 4


<i>a</i>        <i>b</i> <i>c</i> <i>d</i> <i>e</i> . Đến đây ta chỉ ra các mâu thuẫn dạng <i>a</i>    <i>b</i> <i>c</i> <i>d</i> <i>e</i>.


2 2


3<i>x</i> 6<i>y</i> 3<i>x</i> 12<i>y</i> <b>Lời giải. </b>Gọi 5 số đã cho là a, b, c, d, e( , , , ,<i>a b c d e</i> <i>N</i>). Khơng mất


tính tổng qt ta có thể giả sử


a    b c d e. Khi đó ta có <i>a</i>        <i>b</i> 1 <i>c</i> 2 <i>d</i> 3 <i>e</i> 4.
+ Trường hợp 1. Giả sử <i>AE</i> <i>AM</i> 2 <i>AE</i> 2<i>AC</i> <i>AC</i> <i>CE</i>


<i>AC</i>  <i>AF</i>      và <i>n</i> 0. Khi đó ta có


4 2 2


<i>a</i>         <i>b</i> <i>c</i> <i>d</i> <i>e</i> <i>d</i> <i>e</i>, điều này mâu thuẫn với đề bài.
+ Trường hợp 2. Giả sử <i>a b</i>, 5 và 1 3 1


t 1  t3  . Khi đó ta có


3 4 5 2 2 2



<i>a</i>              <i>b</i> <i>c</i> <i>c</i> <i>c</i> <i>d</i> <i>e</i> <i>e</i> <i>d</i>


Điều này mâu thuẫn với đề bài.


+ Trường hợp 3. Giả sử t 1 và <i>d e</i>, 5. Khi đó đó ta có
2 3 4 9 11


</div>
<span class='text_page_counter'>(43)</span><div class='page_container' data-page=43>

Điều này mâu thuẫn với đề bài.


+ Trường hợp 4. Giả sử <sub>x</sub> 2 <sub>3</sub> <sub>x</sub>2 <sub>3x</sub> <sub>2</sub> <sub>0</sub> <sub>x</sub> 3 17


x 2




        và


3 17
x 1; 2; x .


2


<i>x</i> 


    . Khi đó ta có


2; 2


<i>a</i>  <i>b</i> 



Điều này mâu thuẫn với đề bài.


+ Trường hợp 5. Giả sử <i><sub>A</sub></i><sub></sub><i><sub>n</sub></i>2018 <sub></sub><i><sub>n</sub></i>2008 <sub></sub><sub>1</sub><sub>, không tồn tại bộ số tự nhiên phân biệt thỏa </sub>
mãn yêu cầu bài ra.


Vậy điều giả sử là sai nên cả năm số đều lớn hơn hoặc bằng 5.


<b>Đề</b>

<b> s</b>

<b>ố</b>

<b> 7 </b>



<b>Câu 2. </b>


2) Đặt <i>Q x</i>

( )

=<i>P x</i>

( )

− −<i>x</i> 1. Dễ thấy <i>Q</i>

(

2016

)

=<i>Q</i>

(

2017

)

=0nên


( ) (

2016

)(

2017

)(

)



<i>Q x</i> = <i>x</i>− <i>x</i>− <i>x</i>−<i>a</i> .Suy ra


( ) (

2016

)(

2017

)(

)

1


<i>P x</i> = <i>x</i>− <i>x</i>− <i>x</i>−<i>a</i> + +<i>x</i> . Từđó, ta có


(

)

(

)

(

)

(

)



3<i>P</i> 2018 <i>P</i> 2019 6 2018 <i>a</i> 2019.3 6 2019 <i>a</i> 2020 4031


− + = − − − + − + = −


<b>Câu 3. </b>


Đặt a = 2x2, b = x + y, c = x + 2y. Phương trình (2) trở thành



(

)



(

) ( )

2 2


2


2 10 2 3


2 2


<i>a</i> <i>b</i> <i>c</i> <i>c</i> <i>a b</i> <i>a</i> <i>b</i> <i>c a</i> <i>b</i> <i>c</i>


<i>a</i> <i>b</i> <i>c</i> <i>c</i> <i>a</i> <i>b</i> <i>c hay y</i> <i>x</i>


+ + = − − ⇔ + + + =


⇔ + + = ⇔ + = =


</div>
<span class='text_page_counter'>(44)</span><div class='page_container' data-page=44>

<b>a) Tam giác IDA đồ</b>ng dạng với tam giác IAC nên <i>AC</i> <i>AI</i>


<i>AD</i> = <i>ID</i>


Tam giác IBA đồng dạng với tam giác IBC nên <i>BC</i> <i>IB</i>


<i>BD</i> = <i>ID</i>mà IA = ID suy ra đpcm


(tứgiác điều hòa)


b) Ta dễ có BOAI nội tiếp trong đường trịn tâm E đường kính OI. Qua K kẻđường


thẳng vng góc với KE cắt (E, OE) tại hai điểm U, V


2 2 2 2 2


. . ( )


<i>KA KB</i> <i>UK VK</i> <i>VK</i> <i>EV</i> <i>EK</i> <i>OE</i> <i>EK</i> <i>dpcm</i>


⇒ = = = − = −


<b>c) Ta có: IC.ID = IA</b>2<sub>, tam giác vng OAI có đườ</sub><sub>ng cao AH nên IA</sub>2<sub>= IH.IO do đó </sub>


tứgiác OHCD nội tiếp suy ra 1

( )

1


2


<i>DOC</i> <i>DHC</i> <i>DBC</i> <i>DHC</i>


∠ = ∠ ⇒ ∠ = ∠


Mặt khác ∠<i>OHD</i>= ∠<i>OCD</i>= ∠<i>ODC</i> = ∠<i>CHI</i> nên HA là phân giác ∠<i>DHC</i>

( )

2
Từ (1) và (2) suy ra ∠<i>DBC</i>= ∠<i>DHA</i>tam giác HAD đồng dạng tam giác BDC suy ra


đpcm.
<b>Câu 5. </b>


Bài bất đẳng thức này việc đầu tiên cũng như quan trọng nhất là phải tìm được dấu
bằng từđó sẽ tìm ra lời giải.


</div>
<span class='text_page_counter'>(45)</span><div class='page_container' data-page=45>

Ta có

(

2 2 2

)

2 2 2 2 2

( )




5 <i>x</i> + <i>y</i> +<i>z</i> =52+2<i>x</i> +<i>y</i> ≥52+ + =2 1 55⇒ <i>x</i> +<i>y</i> +<i>z</i> ≥11 1
Lại có

(

<i>x</i>−1

)(

<i>y</i>− ≥ ⇔1

)

0 <i>xy</i>+ ≥ +1 <i>x</i> <i>y</i>


Chứng minh tường tự: <i>yz</i>+ ≥ +1 <i>y</i> <i>z</i>; <i>xz</i>+ ≥ +1 <i>x</i> <i>z</i>


Cộng lại theo vếta được: 2

(

<i>xy</i>+<i>yz</i>+<i>zx</i>

)

+ ≥6 4

(

<i>x</i>+ +<i>y</i> <i>z</i>

)

( )

2
Lấy (1) + (2) ta được:

(

)

2

(

)



5 4 5


<i>x</i>+ +<i>y</i> <i>z</i> ≥ + <i>x</i>+ +<i>y</i> <i>z</i> ⇔ + + ≥<i>x</i> <i>y</i> <i>z</i>


<b>Đề</b>

<b> s</b>

<b>ố</b>

<b> 8 </b>



<b>CÂU</b> <b>NỘI DUNG</b> <b>ĐIỂM</b>


<b>1a </b> 2<i>x</i> 2 <i>x x</i> 1 <i>x x</i> 1


<i>P</i>


<i>x</i> <i>x</i> <i>x</i> <i>x</i> <i>x</i>


+ − +


= + −


− + <b>0,25 </b>


(

)(

)




(

)

(

)(

(

)

)



1 1 1 1


2 2


1 1


<i>x</i> <i>x</i> <i>x</i> <i>x</i> <i>x</i> <i>x</i>


<i>x</i>


<i>x</i> <i>x</i> <i>x</i> <i>x</i> <i>x</i>


− + + + − +


+


= + −


− + <b><sub>0,5 </sub></b>


(

1

) (

1

)



2<i>x</i> 2 <i>x</i> <i>x</i> <i>x</i> <i>x</i>


<i>x</i> <i>x</i> <i>x</i>


+ + − +



+


= + −


<b>0,5 </b>


2 2 2 2 2


2


<i>x</i> <i>x</i> <i>x</i>


<i>x</i> <i>x</i>


+ + +


= + =


<b>0,25 </b>


<b>1b </b> <sub>Ta có </sub><i><sub>x</sub></i><sub>= −</sub><sub>3 2 2</sub><sub>⇒</sub> <i><sub>x</sub></i> <sub>=</sub> <sub>2 1</sub><sub>−</sub> <b><sub>0,25 </sub></b>


Thay vào biểu thức 2

(

2 1

)

2 2
2 1


<i>P</i>= − + +


− <b>0,25 </b>



Tính được kết quả <i>P</i>=4 2+2 <b>0,25 </b>


<b>1c </b>


Đưa được 7 7


2 2 2


<i>x</i>


<i>P</i> = <i>x</i>+ + <i>x</i> <b>0,25 </b>


Đánh giá 2<i>x</i>+ +2 2 <i>x</i> >6 <i>x</i>, suy ra 0 7 7
6


2 2 2


<i>x</i>


<i>x</i> <i>x</i>


< <


+ + <b>0,25 </b>


Vậy 7


<i>P</i> chỉ nhận một giá trịnguyên đó là 1 khi


4


2


7 2 2 2 2 5 2 <sub>1</sub> <sub>1</sub>


4
2


<i>x</i>
<i>x</i>


<i>x</i> <i>x</i> <i>x</i> <i>x</i> <i>x</i>


<i>x</i>
<i>x</i>


 =  =


 


= + + ⇔ − + ⇔<sub></sub> ⇔


 =
=


 


 <b>0,25 </b>


</div>
<span class='text_page_counter'>(46)</span><div class='page_container' data-page=46>

Giải phương trình ta được hai nghiệm: <i>x</i>1 =2;<i>x</i>2 = −4 <b>0,5 </b>



<b>2b </b> <sub>Tính đượ</sub><sub>c </sub> <sub>2</sub>

<sub>(</sub>

<sub>)</sub>

3


' <i>m</i> <i>m</i> 1


∆ = − − <b><sub>0,25 </sub></b>


Đểphương trình có hai nghiệm phân biệt <sub>2</sub>

(

)

3


1 0 (*)


<i>m</i> <i>m</i>


⇔ − − > <b><sub>0,25 </sub></b>


Gọi <i>x x</i>1; 2 là hai nghiệm của phương trình, theo Viet ta có


(

)



1 2


3
1 2


2 (1)


1 (2)


<i>x</i> <i>x</i> <i>m</i>


<i>x x</i> <i>m</i>



+ =





= −





Giả sử

( )

2


1 2


<i>x</i> = <i>x</i> thay vào (2) ta được <i>x</i><sub>2</sub> = −<i>m</i> 1;<i>x</i><sub>1</sub>=

(

<i>m</i>−1

)

2 <b><sub>0,25 </sub></b>
Thay hai nghiệm <i>x x</i>1; 2 vào (1) ta được


(

) (

2

)

2 0


1 1 2 3 0


3


<i>m</i>


<i>m</i> <i>m</i> <i>m</i> <i>m</i> <i>m</i>


<i>m</i>


=




− + − = ⇔ − <sub>= ⇔ </sub>


=


Khẳng định hai giá trị <i>m</i> vừa tìm được thỏa mãn điều kiện (*), kết luận <b>0,25 </b>


<b>3 </b>


Điều kiện: <i>x</i>≠0, đưa phương trình trở thành:


2


2 <sub>2</sub>


2 9


2 3 0


2 9


<i>x</i> <i>x</i>


<i>x</i> <i><sub>x</sub></i>


+


+ − =



+ <b>0,25 </b>


Đặt ẩn phụ:


2


2 9


<i>x</i>


<i>t</i>
<i>x</i>


=


+ , phương trình trở thành:


(

)

(

)



3 2 2


1


2 3 1 0 1 2 1 0 <sub>1</sub>


2


<i>t</i>



<i>t</i> <i>t</i> <i>t</i> <i>t</i> <i>t</i>


<i>t</i>


=



− + = ⇔ − − − = ⇔


 =


 <b>0,25 </b>


Trường hợp: <i>t</i> =1 ta có <i>x</i>= 2<i>x</i>2+9 (vơ nghiệm) <b>0,25 </b>
Trường hợp: 1


2


<i>t</i> = − ta có 2


2


0 <sub>3 2</sub>


2 9 2


2


2 9



<i>x</i>


<i>x</i> <i>x</i> <i>x</i>


<i>x</i>


<


+ = − ⇔ <sub></sub> ⇔ = −


=


 <b>0,25 </b>


<b>4a </b>


Xét hai tam giác: <i>AEF </i>và <i>ACB </i>có góc <i>A </i>chung <b>0,25 </b>


Ta có    <i>AEF</i> = <i>AHF AHF</i>; = <i>ACB</i> suy ra  <i>AEF</i> = <i>ACB</i>


(hoặc <i>AFF</i>   = <i>AHE AHE</i>; =<i>ABC</i> suy ra  <i>AFE</i>= <i>ABC</i>) <b>0,25 </b>


<i><b>K</b></i>


<i><b>F</b></i>



<i><b>E</b></i>



<i><b>A</b></i>




<i><b>B</b></i>



<i><b>C</b></i>


<i><b>H</b></i>



<i><b>O</b></i>



</div>
<span class='text_page_counter'>(47)</span><div class='page_container' data-page=47>

Suy ra hai tam giác <i>AEF và ACB đồ</i>ng dạng <b>0,25 </b>
Từ tỷ sốđồng dạng <i>AE</i> <i>AF</i>


<i>AC</i> = <i>AB</i> ta có <i>AE.AB = AC.AF </i> <b>0,25 </b>


<b>4b </b> Xét hai tam giác <i>OHM </i>và <i>OFM</i> có <i>OM </i>chung, <i>OF</i> = <i>OH.</i> <b>0,25 </b>
Có <i>MF </i>= <i>MH </i>(vì tam giác <i>HFC </i>vuông tại F, trung tuyến <i>FM</i>) <b>0,25 </b>


Suy ra ∆<i>OHM</i> = ∆<i>OFM</i> (c.c.c) <b>0,25 </b>


Từđó  0


90


<i>MFO</i>= , <i>MF </i>là tiếp tuyến của đường trịn đường kính AH <b>0,25 </b>


<b>4c </b> <sub>Xét hai tam giác </sub><i><sub>AHM</sub></i><sub> và </sub><i><sub>BHO </sub></i><sub>có </sub>  0


90


<i>AHM</i> =<i>BHO</i>= <b>0,25 </b>



Trong tam giác vng <i>ABC, đườ</i>ng cao <i>AH </i>có


2


. .2 .2 <i>AH</i> <i>HM</i>


<i>AH</i> <i>HB HC</i> <i>AH OH</i> <i>HB HM</i>


<i>HB</i> <i>HO</i>


= ⇒ = ⇒ =


<b>0,25 </b>


Suy ra ∆<i>HBO</i>∆<i>HAM</i> <b>0,25 </b>


Suy ra  <i>HAM</i> =<i>HBO</i> <b>0,25 </b>


<b>4d </b> Gọi K là giao điểm của <i>AM </i>với đường trịn


Ta có <i>HBO</i>  =<i>HAM</i> =<i>MHK</i>, suy ra <i>BO </i>// HK <b>0,25 </b>


Mà <i>HK</i> ⊥ <i>AM</i> , suy ra <i>BO</i>⊥ <i>AM</i> , suy ra O là trực tâm của tam giác <i>ABM</i> <b>0,25 </b>


<b>5 </b> Giả sử <i>a</i>≥ ≥<i>b</i> <i>c</i>, từ giả thiết suy ra <i>ab</i>≥1. Ta có bất đẳng thức sau:


(

) (

)



(

)(

)

(

)




2


2 2 2 2


1


1 1 2


0


1 1 1 1 1 1


<i>a b</i> <i>ab</i>


<i>a</i> <i>b</i> <i>ab</i> <i>a</i> <i>b</i> <i>ab</i>


− −


+ ≥ ⇔ ≥


+ + + + + + (luôn đúng).


Vậy ta cần chứng minh: 2 1 <sub>2</sub> 3


1+<i>ab</i>+1+<i>c</i> ≥ 2 <b>0,25 </b>


2 2 2 2


3 3 3 3



<i>c</i> <i>ab</i> <i>abc</i> <i>c</i> <i>ca</i> <i>bc</i> <i>abc</i> <i>a</i> <i>b</i> <i>c</i> <i>abc</i>


⇔ + − ≥ ⇔ + + ≥ ⇔ + + ≥


Bất đẳng thức hiển nhiên đúng vì

(

)

(

)



( )


2


2
3


3 9


3


<i>a</i> <i>b</i> <i>c</i> <i>ab</i> <i>bc</i> <i>ca</i>


<i>ab</i> <i>bc</i> <i>ca</i> <i>abc</i>


 + + ≥ + + =





+ + ≥





hay <i>a</i>+ + ≥ ≥<i>b</i> <i>c</i> 3 3<i>abc</i>.



Dấu bằng xảy ra khi <i>a</i>= = =<i>b</i> <i>c</i> 1


<b>0,25 </b>


Cho các sốdương <i>a b c</i>, , thỏa mãn <i>a</i>+ + =<i>b</i> <i>c</i> 3.Chứng minh rằng:


2 2 2


3
2


3 3 3


<i>ab</i> <i>bc</i> <i>ca</i>


<i>c</i> <i>a</i> <i>b</i>


+ + ≤


</div>
<span class='text_page_counter'>(48)</span><div class='page_container' data-page=48>

<b>5 </b>


Ta có

(

)



2


3
3


<i>a</i> <i>b</i> <i>c</i>



<i>ab bc</i> <i>ca</i> <i>ab bc</i> <i>ca</i>


+ +
≥ + + ⇒ + + ≤
<b>0,25 </b>
Ta có

(

)(

)


2 2
1 1
2
3


<i>ab</i> <i>ab</i> <i>ab</i> <i>ab</i>


<i>a</i> <i>c</i> <i>b</i> <i>c</i>


<i>a</i> <i>c b</i> <i>c</i>


<i>c</i> <i>c</i> <i>ab</i> <i>bc</i> <i>ca</i>


 
≤ = ≤ <sub></sub> + <sub></sub>
+ +
 
+ +
+ + + +

(

)



1 1 3



2 2 2


<i>ab</i> <i>ab</i> <i>bc</i> <i>ca</i> <i>ca</i>


<i>VT</i> <i>a</i> <i>b</i> <i>c</i>


<i>a</i> <i>c</i> <i>b</i> <i>c</i> <i>c</i> <i>a</i> <i>c</i> <i>b</i> <i>a</i> <i>b</i>


 


≤ <sub></sub> + + + + <sub></sub>= + + =


+ + + + +


  (đpcm)


Dấu bằng xảy ra khi a = b = c = 1 <b>0,25 </b>


<b>Đề</b>

<b> s</b>

<b>ố</b>

<b> 9 </b>



<b>Câu 1. </b>


1). Ta có <sub> </sub><i><sub>m</sub></i>2<sub></sub><sub>8</sub><sub>. </sub>
0


 với mọi <i>m</i> nên phương trình đã cho ln có hai nghiệm phân biệt.
Theo định lý Vi-et ta có 1 2


1 2


2
1
.
2
<i>m</i>
<i>x</i> <i>x</i>
<i>x x</i>
  


 <sub> </sub>



và giả thiết cho 2 2


1 4 2


<i>x</i>  <i>x</i> .


Nên ta có


1 2
1 2
1 2
2
1
.
2
2
<i>m</i>


<i>x</i> <i>x</i>
<i>x x</i>
<i>x</i> <i>x</i>
  


 <sub> </sub>


 


(1) hoặc
1 2
1 2
1 2
2
1
.
2
2
<i>m</i>
<i>x</i> <i>x</i>
<i>x x</i>
<i>x</i> <i>x</i>
  


 <sub> </sub>



  


(2).


+ Giải (1):
Ta có

 

2 2


1
2


2


<i>x x</i>   (vơ nghiệm), nên hệ phương trình (1) vơ nghiệm.


+ Giải (2):


Ta có

2

2 2 1


2 1


1 <sub>1</sub> <sub>1</sub>


1 <sub>2</sub>


2


1



2 <sub>1</sub> <sub>1</sub>


2


<i>x</i> <i>x</i> <i>m</i>


<i>x x</i>


<i>x</i> <i>x</i> <i>m</i>



      

    
 <sub> </sub> <sub>    </sub>


.


Nhận xét: Bài toán áp dụng định lý Vi-ét trong phương trình bậc hai và biến đổi biểuthức.
Nhắc lại kiến thức và phương pháp:


• Phương trình bậc hai có hai nghiệm phân biệt khi và chỉ khi  0.
Phương trình bậc hai <sub>2</sub><i><sub>x</sub></i>2<sub></sub><i><sub>mx</sub></i><sub> </sub><sub>1 0</sub> <sub>có hai nghiệm phân biệt </sub>


1


<i>x</i> và <i>x</i><sub>2</sub> với mọi giá trị
của tham số <i>m</i> vì <sub>  </sub>

 

<i><sub>m</sub></i> 2<sub></sub><sub>4.2. 1</sub>

 

<sub> </sub><i><sub>m</sub></i>2<sub>  </sub><sub>8 8 0</sub> <sub>với mọi giá trị của </sub><i><sub>m</sub></i><sub>. </sub>



</div>
<span class='text_page_counter'>(49)</span><div class='page_container' data-page=49>

Ta có 1 2
1 2
2
1
.
2
<i>m</i>
<i>x</i> <i>x</i>
<i>x x</i>
  


 <sub> </sub>



kết hợp với đề bài cho 2 2 1 2


1 2
1 2
2
4
2
<i>x</i> <i>x</i>
<i>x</i> <i>x</i>
<i>x</i> <i>x</i>
 

 <sub>  </sub>
 .



• Giải các hệ phương trình.


+


1 2 1 2 1 2


1 2


2 2 2 2


1 2


2 2


2 2


1 2


2 2 2


2
2
1


. <sub>2</sub> 2 6 6


1 <sub>1</sub> 9


2 .



2 <sub>2.</sub>


2 <sub>6</sub> <sub>2</sub>


<i>m</i> <i><sub>x</sub></i> <i><sub>x</sub></i> <i><sub>x</sub></i> <i><sub>x</sub></i> <i><sub>x</sub></i> <i><sub>x</sub></i>


<i>x</i> <i>x</i>


<i>m</i> <i>m</i> <i>m</i>


<i>x</i> <i>x</i> <i>x</i> <i>x</i>


<i>x x</i>
<i>m</i>
<i>m</i>
<i>x x</i>
<i>x</i> <i>x</i>

 <sub></sub>
 
 <sub></sub> <sub></sub> <sub></sub>
        
 <sub></sub>  
 <sub></sub>  
 <sub></sub>  
 
 <sub></sub> <sub></sub> <sub></sub> <sub></sub> <sub></sub> <sub></sub> <sub></sub>
   
     
   


   
   <sub> </sub>  <sub> </sub>
 <sub></sub>  <sub> </sub>  <sub></sub> <sub></sub> <sub></sub>
   <sub>  </sub>
 <sub></sub><sub></sub> <sub>  </sub><sub></sub> <sub></sub>



(vơ nghiệm vì <i><sub>m</sub></i>2<sub></sub><sub>0</sub> <sub>nên khơng tồn tại </sub><i><sub>m</sub></i>2<sub> </sub><sub>9</sub><sub>). </sub>


+


1 2 1 2 1 2


1 2


2 2 2 2


1 2


2 2


2 2


1 2


2 2 2


2


2


1


. <sub>2</sub> 2 2 2


1 <sub>1</sub> 1


2 .


2 <sub>2.</sub>


2 <sub>2</sub> <sub>2</sub>


<i>m</i> <i><sub>x</sub></i> <i><sub>x</sub></i> <i><sub>x</sub></i> <i><sub>x</sub></i> <i><sub>x</sub></i> <i><sub>x</sub></i>


<i>x</i> <i>x</i>


<i>m</i> <i>m</i> <i>m</i>


<i>x</i> <i>x</i> <i>x</i> <i>x</i>


<i>x x</i>
<i>m</i>
<i>m</i>
<i>x x</i>
<i>x</i> <i>x</i>

 
 <sub></sub>
 
 <sub></sub> <sub></sub> <sub></sub>


 <sub></sub> <sub></sub> <sub></sub>   <sub></sub>     
 <sub></sub> <sub></sub> 
 <sub></sub> <sub></sub> 
 <sub></sub> <sub></sub> 
 
 <sub> </sub> <sub></sub> <sub></sub> <sub></sub> <sub> </sub> <sub></sub> <sub> </sub>
   
     
   
   
   <sub></sub>   
 <sub> </sub> <sub></sub> <sub> </sub>  <sub></sub> <sub></sub> <sub></sub>
  <sub></sub> <sub></sub> <sub> </sub>
  <sub></sub>  


1 2 2 2


1 1
2
1
1 1
1
1 1
2
2 2
1 1
2
<i>m</i>
<i>m</i> <i>m</i>
<i>m</i>



<i>x</i> <i>x</i> <i>x</i> <i>x</i>


<i>m</i> <i><sub>x</sub></i> <i><sub>x</sub></i>


<i>x</i>

 
 <sub> </sub> <sub>  </sub>
    
<sub></sub> <sub></sub> <sub></sub>
 <sub></sub> <sub></sub>
 <sub></sub> <sub></sub>

<sub></sub>   <sub></sub>   <sub></sub> 
  
  
       
 <sub></sub> <sub></sub>



(thỏa mãn, nhận).


Vậy <i>m</i> 1.


2). Ta có <sub> </sub><i><sub>m</sub></i>2<sub></sub><sub>8</sub><sub>. </sub>
0


 với mọi <i>m</i> nên phương trình đã cho ln có hai nghiệm phân biệt <i>x x</i>1; 2.
Theo định lý Vi-et ta có <sub>1</sub>. <sub>2</sub> 1



2


<i>x x</i>   suy ra <sub>1</sub> <sub>2</sub> 1


2
1
1


. 1 ,


2 1


<i>x</i>


<i>x x</i> <i>m</i>


<i>x</i>
 <sub></sub>

  <sub></sub> 

 .


Nhận xét: Bài toán áp dụng định lý Vi-ét trong phương trình bậc hai và kiến thức bất
phương trình,…


Nhắc lại kiến thức và phương pháp:


• Phương trình bậc hai có hai nghiệmphân biệt.



Phương trình bậc hai đã cho có hai nghiệm phân biệt với mọi giá trị của <i>m</i> (đã chứng
minh ở ý trên)


• Phương trình có nghiệm thỏa mãn điều kiện nào đó.
Ta có <i>x</i> 1 khi và chỉ khi <i>x x</i><sub>1</sub> <sub>2</sub> 1 vì <i>x</i><sub>1</sub> ; <i>x</i><sub>2</sub> dương.


• Định lý Vi-ét trong phương trình bậc hai 1 2
1 2
<i>b</i>
<i>x</i> <i>x</i>
<i>a</i>
<i>c</i>
<i>x x</i>
<i>a</i>
  


 <sub></sub>

.


Ta có <sub>1</sub>. <sub>2</sub> 1 <sub>1</sub>. <sub>2</sub> 1 1 <sub>1</sub>. <sub>2</sub> 1 1


2 2 2 2


</div>
<span class='text_page_counter'>(50)</span><div class='page_container' data-page=50>

Do đó <i>x</i> 1.
<b>Câu 2. </b>


1). Điều kiện: 1


3


<i>x</i> .


Phương trình đã cho tương đương với <sub>18</sub> 2 <sub>2</sub> 8 <sub>9</sub> 1 1 <sub>0</sub>


3 3 3


<i>x</i> <i>x</i>  <i>x</i> 


 <sub></sub> <sub></sub> <sub></sub>
       
 <sub></sub> <sub></sub> <sub></sub>
   
  <sub></sub> <sub></sub>


1 1


1 <sub>3 9</sub>


18 8 9 0


3 <sub>1 1</sub>


3 3
<i>x</i>
<i>x</i> <i>x</i>
<i>x</i>
 
 <sub></sub>



  <sub></sub>  <sub></sub><sub></sub> 
 
 


4 <sub>18</sub> 1 <sub>9</sub> 1 <sub>0</sub> 4


9 3 1 1 9


3 3


<i>x</i> <i>x</i> <i>x</i>


<i>x</i>
 
 
 
 <sub></sub>  <sub></sub>
   
<sub></sub>  <sub></sub><sub></sub><sub></sub> <sub></sub>  <sub></sub><sub></sub> <sub></sub>  
 <sub></sub>   <sub></sub>
 
 
 
.
Chứng minh: Với 1 18 1 9 1 0


3 3 <sub>1 1</sub>


3 3


<i>x</i> <i>x</i>
<i>x</i>
 <sub></sub>

  <sub></sub>  <sub></sub><sub></sub> 
 
 
.
Phương trình đã cho có nghiệm: 4


9


<i>x</i> .


Nhận xét: Bài tốn giải phương trình đưa về phương trình tích.
Nhắc lại kiến thức và phương pháp:


• Tìm điều kiện xác định: Điều kiện: 1 0 1


3 3


<i>x</i>   <i>x</i> .


• Tách, thêm bớt phân tích nhân tử.


2 17 1 2 8 1


18 2 9 0 18 2 9 3 0


3 3 3 3



<i>x</i>  <i>x</i>  <i>x</i>  <sub></sub> <i>x</i>  <i>x</i> <sub></sub><sub> </sub><sub></sub> <i>x</i>  <sub></sub>


 <sub> </sub> <sub></sub>


18 8

1 9 1 1 0


3 3 3


<i>x</i> <i>x</i>   <i>x</i> 
  <sub></sub><sub></sub>  <sub></sub><sub></sub> <sub></sub>   <sub></sub><sub></sub>


 




1 1


1 <sub>3 9</sub>


18 8 9 0


3 1 1


3 3
<i>x</i>
<i>x</i> <i>x</i>
<i>x</i>
 


 <sub></sub>

  <sub></sub>  <sub></sub><sub></sub> 
 
 
4


4 1 <sub>9</sub>


8 9 0


9 3 <sub>1</sub> <sub>1</sub>


3 3
<i>x</i>
<i>x</i> <i>x</i>
<i>x</i>

 <sub></sub> <sub></sub>
 <sub></sub> <sub></sub>
 <sub></sub>  <sub></sub><sub></sub><sub></sub>  <sub></sub><sub></sub> 
  
 


4 <sub>8</sub> 8 9 <sub>0</sub>


9 3 1 1


3 3
<i>x</i> <i>x</i>


<i>x</i>
 <sub></sub>
 <sub></sub>
 <sub></sub>

 <sub></sub><sub></sub> <sub></sub>
 <sub></sub><sub></sub> <sub></sub>
<sub></sub>  <sub></sub><sub></sub><sub></sub><sub></sub>   <sub></sub><sub></sub>
 <sub></sub> <sub></sub><sub></sub>
 <sub>  </sub>
 
 
.


</div>
<span class='text_page_counter'>(51)</span><div class='page_container' data-page=51>

+ Chứng minh 8 8 9


3 <sub>1 1</sub>


3 3


<i>x</i>


<i>x</i>


 


 


khác 0



Với 1
3


<i>x</i> ta có


8
8


8 9


3 <sub>8</sub> <sub>0</sub>


3


1 <sub>0</sub> 1 1


3 3 3


<i>x</i>


<i>x</i>


<i>x</i> <i>x</i>


 


 <sub></sub> <sub> </sub> <sub></sub>






 <sub> </sub>


  





.


+ Suy ra 4 8 8 9 0 4 0 4


9 3 1 1 9 9


3 3


<i>x</i> <i>x</i> <i>x</i> <i>x</i>


<i>x</i>


 <sub></sub>


 <sub></sub>


 <sub></sub>




 <sub></sub><sub></sub> <sub></sub>


 <sub></sub> <sub></sub><sub></sub> <sub> </sub> <sub></sub><sub>     </sub>



 <sub></sub><sub></sub> <sub></sub>


  


 <sub></sub> <sub></sub><sub></sub>


 <sub>  </sub>


 


 


.
• Đối chiếu với điều kiện xác định để đi đến kết luận nghiệm 4


9


<i>x</i> thỏa mãn điều kiện


1
3


<i>x</i> , nên phương trình nhận 4


9


<i>x</i> là nghiệm.


2). Phương trình tương đương với

<i>x</i>3<i>y</i>1



<i>x y</i>  3

15 (1).
Do <i>x y</i>; là số ngun khơng âm nên từ (1), ta có


3 1 5 3 1 3 3 1 15 3 1 1


3 3 3 5 3 1 3 15


<i>x</i> <i>y</i> <i>x</i> <i>y</i> <i>x</i> <i>y</i> <i>x</i> <i>y</i>


<i>x y</i> <i>x y</i> <i>x y</i> <i>x y</i>


   


               


 <sub></sub> <sub></sub> <sub></sub>


   


               


   


    .


Vậy

<i>x y</i>;

  

 2; 0 .


Nhận xét: Bài toán giải phương trình nghiệm nguyên bằng cách đưa về phương trình ước
số


Nhắc lại kiến thức và phương pháp:



• Phân tích một vế thành nhân tử, vế cịn lại là một số.
2 <sub>3</sub> 2 <sub>4</sub> <sub>4</sub> <sub>10</sub> <sub>12 0</sub>


<i>x</i>  <i>y</i>  <i>xy</i> <i>x</i> <i>y</i> 


<i><sub>x</sub></i>2 <sub>3</sub><i><sub>xy x</sub></i>

 

<i><sub>xy</sub></i> <sub>3</sub><i><sub>y</sub></i>2 <i><sub>y</sub></i>

<sub>3</sub><i><sub>x</sub></i> <sub>9</sub><i><sub>y</sub></i> <sub>3 15 0</sub>



          


3 1

 

3 1

 

3 3 1

15


<i>x x</i> <i>y</i> <i>y x</i> <i>y</i> <i>x</i> <i>y</i>


         


<i>x</i> 3<i>y</i> 1



<i>x y</i> 3

15


     


• Tìm các ước của số ở một vế.
15 1.15 3.5 5.3  


Vì <i>x y</i>; 0 nên ta không chọn các ước âm của 15.


Ta có 3 1 1


3 3


<i>x</i> <i>y</i>
<i>x y</i>



   



   


 nên khơng chọn các cặp tích có thừa số thứ hai nhỏ hơn 3.


• Cho các hệ số ở vế này bằng thừa số của vế kia tạo thành các hệ phương trình. Giải các
phương trình này tìm nghiệm.


</div>
<span class='text_page_counter'>(52)</span><div class='page_container' data-page=52>

3 1 1 3 0 18


3 15 12 6


3 1 3 3 2 2


3 5 2 0


3 1 5 3 4 2


3 3 0 2


<i>x</i> <i>y</i> <i>x</i> <i>y</i> <i>x</i>


<i>x y</i> <i>x y</i> <i>x</i>


<i>x</i> <i>y</i> <i>x</i> <i>y</i> <i>x</i>


<i>x y</i> <i>x y</i> <i>y</i>



<i>x</i> <i>y</i> <i>x</i> <i>y</i> <i>x</i>


<i>x y</i> <i>x y</i> <i>y</i>


<sub></sub><sub></sub> <sub></sub> <sub> </sub> <sub></sub><sub></sub> <sub></sub> <sub></sub> <sub></sub><sub></sub> <sub></sub>


<sub></sub> <sub></sub> <sub></sub>


<sub></sub> <sub>  </sub> <sub></sub> <sub> </sub> <sub></sub> <sub> </sub>


  


  


<sub></sub><sub></sub> <sub></sub> <sub> </sub> <sub></sub><sub></sub> <sub></sub> <sub></sub> <sub></sub><sub></sub> <sub></sub>


<sub></sub> <sub></sub><sub></sub> <sub></sub> <sub></sub>


<sub></sub> <sub></sub> <sub></sub>


     


<sub></sub><sub></sub> <sub></sub><sub></sub> <sub></sub><sub></sub>


 


<sub></sub><sub></sub> <sub></sub> <sub> </sub> <sub></sub><sub></sub> <sub></sub> <sub></sub> <sub></sub><sub></sub> <sub> </sub>


<sub></sub> <sub></sub> <sub></sub>



<sub></sub> <sub>  </sub> <sub></sub> <sub> </sub> <sub></sub> <sub></sub>


  


  











,


kết hợp với điều kiện ta có được <i>x</i>2 và <i>y</i>0.
<b>Câu 3.</b>Điều kiện: <i>x y</i> 0.


Phương trình thứ nhất tương đương với


2



4 <i>x y</i>  1 <sub></sub> 3 <i>x y</i>  <i>x y</i>  1<sub></sub> 0


 







2 1


2 1 2 1 0


3 1


<i>x y</i>


<i>x y</i> <i>x y</i>


<i>x y</i> <i>x y</i>


 


   


<sub></sub><sub></sub>   <sub> </sub><sub> </sub>   <sub></sub><sub></sub> 


   






1


2 1 2 1 0



3 1


<i>x y</i> <i>x y</i>


<i>x y</i> <i>x y</i>


 


 


 


<sub></sub><sub></sub>   <sub></sub><sub> </sub>    




   


 


 


(*).


Do <i>x y</i> 0, nên (*) tương đương với 2

<i>x y</i>   

1 0 2<i>y</i>2<i>x</i>1, thế vào phương trình
thứ hai, ta có <sub>6</sub><i><sub>x</sub></i>2<sub>  </sub><i><sub>x</sub></i> <sub>1 0</sub>


1 <sub>0</sub>



2


1 5


3 6


<i>x</i> <i>y</i>


<i>x</i> <i>y</i>




   


 


 <sub>    </sub>






.
Hệ phương trình có nghiệm:

<sub></sub>

;

<sub></sub>

1; 0 , 1; 5


2 3 6


<i>x y</i> <sub></sub> <sub></sub> <sub></sub> <sub></sub>  <sub></sub><sub></sub>


   .



Nhận xét: Bài toán giải hệ phương trình bằng phương pháp nhân liên hợp.
Nhắc lại kiến thức và phương pháp:


• Điều kiện xác định: Biểu thức dưới mẫu khác 0; Biểu thức dưới dấu căn không âm.
Điều kiện xác định: 1 0 0


0


<i>x y</i>


<i>x y</i>
<i>x y</i>


   


 <sub>  </sub>


  


 .


• Nhẩm nghiệm để tìm lượng liên hợp.
+ Ta thấy 1


2


<i>x y</i>  thì phương trình thứ nhất của hệ bằng 0. Do đó phương trình đó


thứ nhất của hệ phương trình có nhân tử 1


2


<i>x y</i>  hay 2

<i>x y</i> 

1.


2



1 1 4 3


<i>x y</i>    <i>x y</i>  <i>x y</i> <sub></sub>4

<i>x y</i>

2 1<sub> </sub>  3

<i>x y</i> 

<i>x y</i>  1<sub></sub> 0


 


  .


+ Hằng đẳng thức hiệu hai bình phương <i><sub>a</sub></i>2<sub>  </sub><i><sub>b</sub></i>2

<i><sub>a b a b</sub></i>



<sub></sub>

<sub>. </sub>

2



4 <i>x y</i><sub></sub> <sub> </sub>1 2<sub></sub> <i>x y</i><sub></sub> <sub></sub>1 2<sub> </sub>  <i>x y</i><sub></sub> <sub></sub>1<sub></sub>


   .


</div>
<span class='text_page_counter'>(53)</span><div class='page_container' data-page=53>



3 <i>x y</i>  <i>x y</i> 1





3 1 3 1


3 1



<i>x y</i> <i>x y</i> <i>x y</i> <i>x y</i>


<i>x y</i> <i>x y</i>


 <sub> </sub> <sub> </sub>   <sub></sub> <sub></sub> <sub> </sub> 
   
   

   



 



2 2


3 1 <sub>3</sub> <sub>1</sub>


3 1 3 1


<i>x y</i> <i>x y</i> <i><sub>x y</sub></i> <i><sub>x y</sub></i>


<i>x y</i> <i>x y</i> <i>x y</i> <i>x y</i>


 <sub></sub>  <sub></sub> <sub> </sub>
     
 
 
       




2 1
3 1
<i>x y</i>
<i>x y</i> <i>x y</i>


 


    ,


suy ra

2



1 1 4 3


<i>x y</i>    <i>x y</i>  <i>x y</i>


2



4 <i>x y</i> 1 3 <i>x y</i> <i>x y</i> 1 0


   
<sub></sub>   <sub> </sub>     <sub></sub>
 
 



2 1


2 1 2 1 0



3 1


<i>x y</i>


<i>x y</i> <i>x y</i>


<i>x y</i> <i>x y</i>


 
   
<sub></sub><sub></sub>   <sub> </sub><sub> </sub>   <sub></sub><sub></sub> 
   



1


2 1 2 1 0


3 1


<i>x y</i> <i>x y</i>


<i>x y</i> <i>x y</i>


 
 
 
<sub></sub><sub></sub>   <sub></sub><sub> </sub>    


   
 
 
.


• Phương trình tích


1
2
1 2


0
0


. . . 0


0


<i>n</i>


<i>n</i>
<i>A</i>
<i>A</i>
<i>A A</i> <i>A</i>


<i>A</i>
 <sub></sub>

 <sub></sub>


  

 <sub></sub>


 .


2



1 1 4 3


<i>x y</i>    <i>x y</i>  <i>x y</i>






1


2 1 2 1 0


3 1


<i>x y</i> <i>x y</i>


<i>x y</i> <i>x y</i>


 
 
 


<sub></sub><sub></sub>   <sub></sub><sub> </sub>    

   
 
 





2 1 0


1


2 1 0


3 1


<i>x y</i>
<i>x y</i>


<i>x y</i> <i>x y</i>


 <sub>  </sub>


  <sub>  </sub> <sub></sub>

    

.


• Chứng minh một phương trình vơ nghiệm.


Ta có <i>x y</i>  0 2

<i>x y</i> 

0 và




1 <sub>0</sub>


3 <i>x y</i>  <i>x y</i> 1 , suy ra




1
2 0
3 1
<i>x y</i>


<i>x y</i> <i>x y</i>


  


   



1


2 1 0


3 1


<i>x y</i>



<i>x y</i> <i>x y</i>


    


    .


Do đó phương trình





1


2 1 0


3 1


<i>x y</i>


<i>x y</i> <i>x y</i>


   


    vô nghiệm.


</div>
<span class='text_page_counter'>(54)</span><div class='page_container' data-page=54>



2 <sub>2</sub> <sub>2</sub> <sub>2</sub>


1



1


2 1 0 <sub>2</sub>


2
1


4 2 1 <sub>4</sub> <sub>2</sub> <sub>1</sub> <sub>4</sub> <sub>2</sub> <sub>1</sub>


2


<i>y x</i>


<i>x y</i> <i>y x</i>


<i>x</i> <i>xy</i> <i><sub>x</sub></i> <i><sub>x x</sub></i> <i><sub>x</sub></i> <i><sub>x</sub></i> <i><sub>x</sub></i>


   




 <sub></sub>


     <sub></sub>  


 <sub></sub> <sub></sub>


  <sub></sub> <sub></sub> 



 <sub></sub> <sub></sub>  <sub></sub> <sub></sub> 


  


 <sub></sub>  <sub></sub><sub></sub>  <sub></sub><sub></sub><sub></sub> <sub></sub>   





2


1


1 <sub>2</sub>


2 <sub>0</sub>


6 1 0


1


1 <sub>3</sub> 1


2 <sub>1</sub> 3


5
2


6


<i>x</i>


<i>x</i>


<i>y</i>
<i>x</i> <i>x</i>


<i>x</i>


<i>y x</i> <i>x</i>


<i>y x</i>


<i>y</i>





 


 <sub></sub>


  


 <sub></sub>


 <sub></sub>




  


     



 <sub></sub> <sub></sub>


  <sub></sub>


<sub></sub> <sub> </sub> <sub></sub><sub></sub>   <sub></sub> <sub> </sub>


  <sub></sub>


  


 <sub></sub> <sub></sub>




  


 <sub></sub> <sub> </sub>


 


 <sub></sub>





(nhận, thỏa mãn).


<b>Câu 4.</b> Phương trình tương đương với <i><sub>x</sub></i>2<sub></sub><i><sub>y</sub></i>2<sub></sub><sub>4</sub><i><sub>x</sub></i><sub></sub><sub>2</sub><sub> (1). </sub>
Ta có <i><sub>x</sub></i>2<sub></sub><sub>4</sub><i><sub>x</sub></i><sub>   </sub><sub>2</sub> <i><sub>y</sub></i>2 <sub>0</sub> <sub> </sub>

<i><sub>x</sub></i> <sub>6 2</sub><sub></sub>



<i><sub>x</sub></i><sub></sub> <sub>6 2</sub><sub> </sub>

<sub>0</sub>


2 6 <i>x</i> 2 6


    


10 4 6 4<i>x</i> 2 10 4 6


      (2).


Từ (1) và (2), suy ra <sub>10 4 6</sub><sub></sub> <sub></sub><i><sub>x</sub></i>2<sub></sub><i><sub>y</sub></i>2<sub></sub><sub>10 4 6</sub><sub></sub> <sub>. </sub>


Nhận xét: Bài toán áp dụng biến đổi tương đương một phương trình, giải bất phương
trình bậc hai.


Nhắc lại kiến thức và phương pháp:


• Biến đổi tương đương một phương trình.


2 2 <sub>4</sub> <sub>2 0</sub> 2 2 <sub>4</sub> <sub>2</sub>


<i>x</i> <i>y</i>  <i>x</i>  <i>x</i> <i>y</i>  <i>x</i> (1).


2 <sub>4</sub> <sub>2</sub> 2


<i>x</i> <i>x</i> <i>y</i>


     (2).


• Bất đẳng thức.



Ta có <i><sub>y</sub></i>2<sub>   </sub><sub>0</sub> <i><sub>y</sub></i>2 <sub>0</sub> <sub>kết hợp với (2) ta có </sub><i><sub>x</sub></i>2<sub></sub><sub>4</sub><i><sub>x</sub></i><sub> </sub><sub>2 0</sub><sub>. </sub>
• Giải bất phương trình bậc hai.






2 <sub>4</sub> <sub>2 0</sub> <sub>6 2</sub> <sub>6 2</sub> <sub>0</sub> <sub>2</sub> <sub>6</sub> <sub>2</sub> <sub>6</sub>


<i>x</i>  <i>x</i>   <i>x</i>  <i>x</i>       <i>x</i>


• Biến đổi tương đương bất phương trình.


2 6  <i>x</i> 2 610 4 6 4  <i>x</i> 2 10 4 6 .


Kết hợp với (1) ta có <sub>10 4 6</sub><sub></sub> <sub></sub><i><sub>x</sub></i>2<sub></sub><i><sub>y</sub></i>2<sub></sub><sub>10 4 6</sub><sub></sub> <sub>(điều phải chứng minh).</sub>
<b>Câu 5. </b>


<i><b>P</b></i>


<i><b>K</b></i>
<i><b>O</b></i>


<i><b>C</b></i>
<i><b>A</b></i>


<i><b>B</b></i>


</div>
<span class='text_page_counter'>(55)</span><div class='page_container' data-page=55>

1). Ta có <i>MNC MBC</i>  (1) (cùng <i>MC</i>).


 



<i>MBC BCN</i> (do cùng phụ với góc <i>ABC</i>) (2).


Từ (1) và (2), ta có <i>MNC BCN</i>  suy ra tam giác <i>KNC</i>cân tại <i>K</i>.


Nhận xét: Chứng minh một tam giác cân bằng cách chứng minh tam giác đó có hai góc
bằng nhau.


Nhắc lại kiến thức và phương pháp:


• Trong một đường trịn, hai góc nội tiếp cùng chắn một cung thì bằng nhau.


 


<i>MNC MBC</i> (hai góc nội tiếp cùng chắn cug <i>MC</i> của ( )<i>O</i> ).
• Góc nội tiếp chắn nửa đường trịn bằng 90




<i>MBA</i> là góc nội tiếp chắn nửa đường tròn được chia bởi đường kính <i>AM</i> nên


 <sub>90</sub>   <sub>90</sub>


<i>MBA</i>  <i>MBC CBA</i>  .


• Hai góc cùng cộng với một góc bằng được hai góc bằng nhau thì hai góc ban đầu bằng
nhau.


+  <i>MBC CBA</i>  90 (chứng minh trên);
+ <i>ABC ACB</i>   90 (do <i>CN AB</i> );
Suy ra <i>MBC ACB</i>



• Tam giác có hai góc bằng nhau là tam giác cân.


 


   


<i>MNC MBC</i>


<i>MNC ACB</i>
<i>MBC ACB</i>


 <sub></sub>


 <sub></sub> <sub></sub>


 <sub></sub>


 hay


 


<i>KNC KCN</i> suy ra <i>KCN</i> cân tại <i>K</i> (điều phải
chứng minh).


2). Ta có <i>ON OC</i> (3).
Từ trên suy ra <i>KN KC</i> (4).
Từ (3) và (4), ta có <i>OK NC</i> .


Do <i>NC BM</i> (cùng vng góc với <i>AB</i>).



Nhận xét: Chứng minh một tam giác cân bằng cách chứng minh tam giác đó có hai góc
bằng nhau.


Nhắc lại kiến thức và phương pháp:


• Hai điểm cùng thuộc một đường trịn thì khoảng cách đến tâm bằng nhau.
Ta có <i>N C</i>; thuộc ( )<i>O</i> nên <i>ON OC</i> .


</div>
<span class='text_page_counter'>(56)</span><div class='page_container' data-page=56>

• Khoảng cách từ một điểm đến hai đầu mút của một đoạn thẳng bằng nhau thì điểm
đó thuộc trung trực của đoạn thẳng đó.


+ <i>ON OC</i> nên <i>O</i> thuộc trung trực của <i>NC</i>;


+ <i>KN KC</i> nên <i>K</i> thuộc trung trực của <i>NC</i>;
Suy ra <i>OK</i> là trung trực của <i>NC</i> nên <i>OK NC</i> .


• Hai đường thẳng cùng vng góc với một đường thẳng thì song song với nhau.


<i>NC AB</i>


<i>NC BM</i>
<i>BM AB</i>


 


 <sub></sub>


 



  .


• Quan hệ từ vng góc đến song song <i>a b</i>


<i>a c</i>


 



  thì <i>b c</i> .


<i>OK NC</i>


<i>OK BM</i>
<i>NC BM</i>


 


 <sub></sub> <sub></sub>





  (điều cần chứng minh).


3). Ta có


+<i>BNM BAM</i> (<i>MB</i>) (5).
+ <i>BMN BCN</i> (<i>NB</i>) (6).


+ <i>BAM NCB</i> (do cùng phụ với góc <i>ABC</i>) (7).


Từ (5), (6) và (7), suy ra <i>BNM BMN</i> nên <i>BM BN</i> .


Từ giả thiết ta có <i>ON OM</i> và <i>PM PN</i> nên 3 điểm <i>P B O</i>; ; nằm trên đường trung trực
đoạn <i>MN</i> vậy <i>P B O</i>; ; thẳng hàng.


Nhận xét: Chứng minh ba điểm thẳng hàng ta chứng minh chúng cùng thuộc một đường
thẳng cố định.


Nhắc lại kiến thức và phương pháp:


• Trong một đường trịn, hai góc nội tiếp cùng chắn một cung thì bằng nhau.
+ <i>BNM BAM</i> (hai góc nội tiếp cùng chắn cung <i>BM</i> của đường tròn ( )<i>O</i> ).
+ <i>BMN BCN</i> (hai góc nội tiếp cùng chắn cung <i>BN</i> của đường trịn ( )<i>O</i> ).


• Tam giác có hai góc bằng nhau là tam giác cân. Tam giác cân có hai cạnh bên bằng
nhau.


 


 


 


 


<i>BNM BAM</i>


<i>BMN BCN</i> <i>BNM BMN</i> <i>BMN</i>
<i>BAM NCB</i>



 <sub></sub>





 <sub></sub> <sub></sub> <sub></sub> <sub> </sub>





 <sub></sub>





cân tại <i>B</i>nên <i>BM BN</i> .


• Khoảng cách từ một điểm đến hai đầu mút của một đoạn thẳng bằng nhau thì điểm
đó thuộc trung trực của đoạn thẳng đó.


+ <i>BM BN</i> nên <i>B</i> nằm trên đường trung trực của <i>MN</i>;


+ <i>OM ON</i> (do <i>M N</i>; cùng nằm trên ( )<i>O</i> ) nên <i>O</i> nằm trên trung trực của <i>MN</i>;


+ <i>PM PN</i> (do tính chất hai tiếp tuyến cắt nhau) nên <i>P</i> nằm trên trung trực của <i>MN</i>;


</div>
<span class='text_page_counter'>(57)</span><div class='page_container' data-page=57>

<b>Câu 6. </b>


Hạ <i>CK</i> vng góc <i>AB</i> tại <i>K</i> (giải thích tam giác <i>ABC</i> khơng tù tại <i>B</i> hay <i>C</i>).
Ta có <i>CK</i>2 3<i>a</i>.


Nên ta có <sub>3</sub> 2 <sub>3</sub>



<i>ABC</i>


<i>S</i><sub></sub>  <i>a</i> (đvdt).


Ta có <i>BK a</i> , suy ra <i><sub>BC</sub></i><sub></sub> <i><sub>BK</sub></i>2<sub></sub><i><sub>CK</sub></i>2 <sub></sub><i><sub>a</sub></i> <sub>13</sub><sub>. </sub>


2 6 39


13
<i>ABC</i>


<i>S</i> <i>a</i>


<i>AH</i>


<i>BC</i>


   .


Nhận xét: Chứng minh ba điểm thẳng hàng ta chứng minh chúng cùng thuộc một đường
thẳng cố định.


Nhắc lại kiến thức và phương pháp:


• Trong một tam giác, cạnh đối diện với góc lớn hơn thì lớn hơn.


+ Giả sử, <i>ABC</i> có <i>C</i> là góc tù thì <i>C</i> là góc lớn nhất nên <i>C B</i>  suy ra <i>AB AC</i> mà
3


<i>AB</i> <i>a</i> và <i>AC</i>4<i>a</i> do đó có 3<i>a</i>4<i>a</i> 3 4 (vơ lý). Suy ra <i>ABC</i> khơng thể có <i>C</i> là


góc tù;


+ Chứng minh tương tự ta có <i>ABC</i> khơng thể có <i>B</i> là góc tù;
Suy các đường cao của <i>ABC</i> đều nằm bên trong tam giác.


• Trong tam giác vng, độ dài cạnh góc vng bằng tích cách huyền với sin góc đối
diện với cạnh góc vng đó.


<i>AKC</i>


 vng tại <i>K</i> có sin sin60 4 . 3 2 3
2


<i>AK AC</i> <i>A AC</i>   <i>a</i>  <i>a</i> .


• Định lý Py-ta-go trong tam giác vng: “Bình phương cạnh huyền bằng tổng bình
phương hai cạnh góc vng”.


+ <i>AKC</i> vng tại <i>K</i> có <i><sub>AC</sub></i>2<sub></sub><i><sub>KA</sub></i>2<sub></sub><i><sub>KC</sub></i>2<sub> </sub>


2

 

2


2 <sub>2 3</sub> <sub>4</sub> 2 <sub>16</sub> 2 <sub>12</sub> 2 <sub>4</sub> 2 <sub>2</sub>


<i>KA</i> <i>a</i> <i>a</i> <i>KA</i> <i>a</i> <i>a</i> <i>a</i> <i>KA</i> <i>a</i>


         <i>KB AB KA</i>    3<i>a</i> 2<i>a a</i>.


+ <i>BKC</i> vuông tại <i>K</i> có <i><sub>BC</sub></i>2<sub></sub><i><sub>KB</sub></i>2<sub></sub><i><sub>KC</sub></i>2<sub></sub><i><sub>a</sub></i>2<sub></sub>

<sub>2 3</sub><i><sub>a</sub></i>

2
<i><b>A</b></i>


<i><b>B</b></i>


<i><b>C</b></i>
<i><b>K</b></i>


</div>
<span class='text_page_counter'>(58)</span><div class='page_container' data-page=58>

2 2 <sub>12</sub> 2 <sub>13</sub> 2 <sub>13</sub>


<i>BC</i> <i>a</i> <i>a</i> <i>a</i> <i>BC a</i>


     


• Diện tích tam giác bằng nửa tích đường cao với cạnh tương ứng
+ 1<sub>. .</sub> 1<sub>.2 3.3</sub> <sub>3</sub> 2 <sub>3</sub>


2 2


<i>ABC</i>


<i>S</i><sub></sub>  <i>CK AB</i> <i>a</i> <i>a</i> <i>a</i> (đvdt);


+ 1. . 1. . 13


2 2


<i>ABC</i>


<i>S</i><sub></sub>  <i>AH BC</i> <i>AH a</i> (đvdt);


Suy ra 1<sub>.</sub> <sub>. 13 3</sub> 2 <sub>3</sub> 6 2 3 69 3



2 13 13


<i>a</i> <i>a</i>


<i>AH a</i> <i>a</i> <i>AH</i>


<i>a</i>


    .


<b>Câu 7.</b>Đặt <i>x</i> 1;<i>y</i> 1;<i>z</i> 1


<i>a</i> <i>b</i> <i>c</i>


   suy ra <i>xyz</i>1 và <i>x y z</i>; ; dương.
Bất đẳng thức tương đương với




2 2 2


9 9


2
2


<i>y</i>


<i>x</i> <i>z</i>



<i>P</i>


<i>x y z</i>


<i>y</i> <i>z</i> <i>x</i>


    


  (*).


Ta có 1 <i>x</i><sub>2</sub> 2 1; <i>y</i><sub>2</sub> 2 1; <i>z</i><sub>2</sub> 2


<i>x</i><i>y</i> <i>y y</i><i>z</i> <i>z z</i><i>x</i> <i>x</i>


2 2 2


1 1 1


<i>y</i>


<i>x</i> <i>z</i> <i><sub>xy yz zx</sub></i>


<i>x y z</i>


<i>y</i> <i>z</i> <i>x</i>


         .


Ta có <i>x y z xyz x y z</i>  

 




       

     

2 2 2
( )


<i>xy zx</i>  <i>yz yx</i>  <i>zx zy</i> <i>xy</i> <i>yz</i> <i>zx</i>


   



2


3


<i>xy yz zx</i>


<i>x y z</i>  


    .




2
27
2


<i>P</i> <i>xy yz zx</i>


<i>xy yz zx</i>


   



  .


Do



2


27 9


2
2


<i>xy yz zx</i>


<i>xy yz zx</i>


   


 


9
2


<i>P</i>


  .


Cô-Si cho 3 số .


Dấu “” xảy ra khi <i>a b c</i>  1.



Nhận xét: Bài toán chứng minh bất đẳng thức bằng phương pháp đổi biến và áp dụng bất
đẳng thức Cơ-si.


Nhắc lại kiến thức và phương pháp:


• Đổi biến, biến đổi để có giả thiết mới, điều kiện cho biến mới, điều cần chứng minh
mới.


Đặt <i>x</i> 1; <i>y</i> 1;<i>z</i> 1


<i>a</i> <i>b</i> <i>c</i>


   suy ra <i>xyz</i>1 và <i>x y z</i>; ; dương.
Cần phải chứng minh




2 2 2


9 9


2
2


<i>y</i>


<i>x</i> <i>z</i>


<i>P</i>



<i>x y z</i>


<i>y</i> <i>z</i> <i>x</i>


    


  .


• Bất đẳng thức Cô-si cho hai số dương: <i><sub>A</sub></i>2<sub></sub><i><sub>B</sub></i>2<sub></sub><sub>2</sub><i><sub>AB</sub></i> <sub>với </sub><i><sub>A B</sub></i><sub>,</sub> <sub>dương.</sub>
+ Với x, y là các số dương có 1


<i>x</i> và 2


<i>x</i>


<i>y</i> là các số dương nên ta có 2


1 <i>x</i> 2


<i>x</i><i>y</i> <i>y</i>.


</div>
<span class='text_page_counter'>(59)</span><div class='page_container' data-page=59>

+ 1 <i>y</i><sub>2</sub> 2


<i>y</i><i>z</i> <i>z</i>


+ 1 <i>z</i><sub>2</sub> 2


<i>z</i><i>x</i> <i>x</i>


• Cộng vế theo vế của các bất đẳng thức cùng chiều ta được bất đẳng thức mới cùng


chiều.


Cộng vế theo vế của các bất đẳng thức vừa chứng minh trên ta được:


2 2 2


2 2 2 1 1 1


<i>y</i>


<i>x</i> <i>z</i>


<i>x y z x y z</i>


<i>y</i> <i>z</i> <i>x</i>      


2 2 2


1 1 1


<i>y</i> <i>xyz xyz xyz</i>


<i>x</i> <i>z</i> <i><sub>xy yz zx</sub></i>


<i>x y z</i> <i>x</i> <i>y</i> <i>z</i>


<i>y</i> <i>z</i> <i>x</i>


            (vì <i>xyz</i>1).



• Biến biểu thức để có được đẳng thức đúng.




<i>x y z xyz x y z</i>     (vì <i>xyz</i>1)

       

<i>xy zx</i>  <i>yz yx</i>  <i>zx zy</i>( ).
• Áp dụng ngược chiều bất đẳng thức Cô-si.


      


      


 

   



2 2


2 2


2 2


2
2


2 ( )


<i>xy zx</i>
<i>yz yx</i>
<i>zx</i>


<i>xy</i> <i>zx</i>
<i>yz</i> <i>xy</i>
<i>yz</i> <i>zx</i>
<i>zy</i>



 <sub></sub> <sub></sub>





 <sub></sub> <sub></sub>






  





       

     

2 2 2
( )


<i>xy zx</i>  <i>yz yx</i>  <i>zx zy</i> <i>xy</i> <i>yz</i> <i>zx</i>


   


2
3


<i>xy yz zx</i>


<i>x y z</i>  


    .



• Kết hợp các bất đẳng thức nhỏ thành bất đẳng thứclớn.




2
27
2


<i>P</i> <i>xy yz zx</i>


<i>xy yz zx</i>


   


  .




2


27 9


2
2


<i>xy yz zx</i>


<i>xy yz zx</i>


   



  .


• Với <i>a b</i> ; <i>b c</i> thì <i>a c</i> .


Ta có










2


2
27


2 <sub>9</sub>


27 9 2


2
2


<i>P</i> <i>xy yz zx</i>


<i>xy yz zx</i>


<i>P</i>
<i>xy yz zx</i>


<i>xy yz zx</i>


    



  


 <sub> </sub>





 <sub></sub> <sub></sub> <sub></sub> <sub></sub>





  





(điều phải chứng minh).


<b>Đề</b>

<b> s</b>

<b>ố</b>

<b> 10 </b>



<b>Bài 1. </b>


</div>
<span class='text_page_counter'>(60)</span><div class='page_container' data-page=60>

(

) (

) (

)


(

)(

)




(

)(

)

(

)



2 3 5 7 2 3


:


2 2 1 2 3 2 5 10
2 2 1 3 2 5 7 <sub>5</sub> <sub>10</sub>


.


2 3
2 2 1


5 2


2 3
.


2 3
2 2 1


5
2 1


<i>x</i> <i>x</i>


<i>A</i>



<i>x</i> <i>x</i> <i>x</i> <i>x</i> <i>x</i> <i>x</i>


<i>x</i> <i>x</i> <i>x</i> <i><sub>x</sub></i> <i><sub>x</sub></i>


<i>x</i>


<i>x</i> <i>x</i>


<i>x</i> <i>x</i>
<i>x</i>


<i>x</i>


<i>x</i> <i>x</i>


<i>x</i>
<i>x</i>


 −  +


=<sub></sub><sub></sub> + − <sub></sub><sub></sub>


− + − − −


 


+ + − − − <sub>−</sub>


=



+


− +



+


=


+


− +


=
+


2. Vì<i>x</i>> ⇒0 5 <i>x</i> >0; 2 <i>x</i>+ > ⇒ >1 0 <i>A</i> 0


Mặt khác, xét 3 5 3 2

(

1

)

3 0


2 1 2 1


<i>x</i> <i>x</i> <i><sub>x</sub></i>


<i>A</i>


<i>x</i> <i>x</i>


− + <sub>−</sub> <sub>−</sub>



− = = <


+ + ∀ > ⇒ <<i>x</i> 0 <i>A</i> 3
Vậy 0 < A < 3


Do đó A nguyên ⇔ A = 1 hoặc A = 2.


5 1 1


1 1 5 2 1 3 1


3 9


2 1
<i>x</i>


<i>A</i> <i>x</i> <i>x</i> <i>x</i> <i>x</i> <i>x</i>


<i>x</i>


= ⇔ = ⇔ = + ⇔ = ⇔ = ⇔ =


+ (thỏa mãn)


5


2 2 5 2(2 1) 2 2 4


2 1
<i>x</i>



<i>A</i> <i>x</i> <i>x</i> <i>x</i> <i>x</i> <i>x</i>


<i>x</i>


= ⇔ = ⇔ = + ⇔ = ⇔ = ⇔ =


+ (loại)


Vậy 1


9
<i>A</i>∈ ⇔ = <i>x</i>


<b>Bài 2.</b>


1. Khi m = –5 ⇒ (d) : y = –4x + 12


Khi đó , phương trình hoành độgiao điểm của (P) và (d) là:


2 2


4 12 4 12 0 ( 6)( 2) 0


<i>x</i> = − +<i>x</i> ⇔<i>x</i> + <i>x</i>− = ⇔ <i>x</i>+ <i>x</i>− =


⇔ x = –6 hoặc x = 2
Khi x = –6 ⇒ y = 36
Khi x = 2 ⇒ y = 4



Vậy tọa độgiao điểm của (P) và (d) là (–6;36) và (2;4)
2. Phương trình hồnh độgiao điểm của (P) và (d):


2 2


2( 3) 2 2 2( 3) 2 2 0


</div>
<span class='text_page_counter'>(61)</span><div class='page_container' data-page=61>

(d) cắt (P) tại hai điểm phân biệt ⇔ (1) có hai nghiệm phân biệt


2
2


2
2


' ( 3) (2 2) 0
( 6 9) (2 2) 0


4 7 0
( 2) 3 0


<i>m</i> <i>m</i>


<i>m</i> <i>m</i> <i>m</i>


<i>m</i> <i>m</i>


<i>m</i>


⇔ ∆ = + − − >



⇔ + + − − >


⇔ + + >


⇔ + + >


(luôn đúng ∀ m)


Vậy (d) luôn cắt (P) tại hai điểm phân biệt có hồnh độ là <i>x x</i>1, 2 với<i>x x</i>1, 2là hai nghiệm của


phương trình (1)


Hai giao điểm có hồnh độdương ⇔ (1) có hai nghiệm dương


1 2


1 2


2( 3) 0 3


1


2 2 0 1


<i>x</i> <i>x</i> <i>m</i> <i>m</i>


<i>m</i>


<i>x x</i> <i>m</i> <i>m</i>



+ = + > > −


 


⇔<sub></sub> ⇔<sub></sub> ⇔ >


= − > <sub></sub> >


Vậy m > 1.


3. Gọi

(

<i>x y</i>0; 0

)

là điểm cốđịnh mà (d) luôn đi qua ∀ m
Khi đó:


0 0


0 0 0


0 0 0


0 0 0 0


2( 3) 2 2( )


(2 2) (6 2 ) 0( )


2 2 0 1 1


6 2 0 6.1 2 0 8



<i>y</i> <i>m</i> <i>x</i> <i>m</i> <i>m</i>


<i>m</i> <i>x</i> <i>x</i> <i>y</i> <i>m</i>


<i>x</i> <i>x</i> <i>x</i>


<i>x</i> <i>y</i> <i>y</i> <i>y</i>


= + − + ∀
⇔ − + + − = ∀
− = = =
  
⇔<sub></sub> ⇔<sub></sub> ⇔<sub></sub>
+ − = + − = =
  


Vậy (d) luôn đi qua điểm (1;8) ∀m.
<b>Bài 3.</b>


2 2


2 2


2 3 2 5(2 ) 0(1)
( )
2 3 15 0(2)


<i>x</i> <i>xy</i> <i>y</i> <i>x</i> <i>y</i>



<i>I</i>


<i>x</i> <i>xy</i> <i>y</i>


 + − − − =


− − + =

Ta có:


(1) (2 )( 2 ) 5(2 ) 0
(2 )( 2 5) 0


2
5 2


<i>x</i> <i>y x</i> <i>y</i> <i>x</i> <i>y</i>


<i>x</i> <i>y x</i> <i>y</i>


<i>y</i> <i>x</i>
<i>x</i> <i>y</i>
⇔ − + − − =
⇔ − + − =
=

⇔  <sub>= −</sub>



Do đó:( ) <sub>2</sub> 2 <sub>2</sub> ( )


2 .2 3(2 ) 15 0


<i>y</i> <i>x</i>


<i>I</i> <i>II</i>


<i>x</i> <i>x x</i> <i>x</i>


=

⇔ 


− − + =


 hoặc 2 2


5 2


( )


(5 2 ) 2(5 2 ) 3 15 0


<i>x</i> <i>y</i>


<i>III</i>


<i>y</i> <i>y y</i> <i>y</i>



= −


− − − − + =

2


2 1; 2


( )


1; 2


15 15 0


<i>y</i> <i>x</i> <i>x</i> <i>y</i>


</div>
<span class='text_page_counter'>(62)</span><div class='page_container' data-page=62>

2


5 2 2; 1


( )


4; 3


5 30 40 0


<i>x</i> <i>y</i> <i>y</i> <i>x</i>


<i>III</i>



<i>y</i> <i>x</i>


<i>y</i> <i>y</i>


= − = =


 


⇔<sub></sub> ⇔<sub> =</sub>


= −


− + = <sub></sub>




Vậy hệphương trình (I) có nghiệm (1;2) , (-1;-2) , (-3;4)


<b>Bài 4.</b>


1. Vì TB là tiếp tuyến của (O) nên


BAD = DBT (góc nội tiếp và góc ở tâm cùng chắn cùng BD)
Xét ∆ ABT và ∆ BDT có:


( )


~ ( . )
( )



<i>ATB chung</i>


<i>ABT</i> <i>BDT g g</i>
<i>DBT BAT cmt</i>




⇒ ∆ ∆


 <sub>=</sub>



2. Vì


2


~ <i>AB</i> <i>AT</i> <i>BT</i> <i>AB</i> <i>AT BT</i>. <i>AT</i>


<i>ABT</i> <i>BDT</i>


<i>BD</i> <i>BT</i> <i>DT</i> <i>BD</i> <i>BT DT</i> <i>DT</i>


 


∆ ∆ ⇒ = = ⇒<sub></sub> <sub></sub> = =


 


Chứng minh tương tự ta có:



2


<i>AC</i> <i>AT</i>


<i>CD</i> <i>DT</i>


 <sub> =</sub>


 


</div>
<span class='text_page_counter'>(63)</span><div class='page_container' data-page=63>

Do đó


2 2


. .


<i>AB</i> <i>AC</i> <i>AB</i> <i>AC</i>


<i>AB CD</i> <i>BD AC</i>


<i>BD</i> <i>CD</i> <i>BD</i> <i>CD</i>


  <sub>=</sub>  <sub>⇒</sub> <sub>=</sub> <sub>⇒</sub> <sub>=</sub>


   


   


3. Gọi I1, I2 lần lượt là giao điểm của BC với tia phân giác góc BAC và góc BDC.


Xét ∆ ABC có tia phân giác AI1, theo tính chất đường phân giác ta có:


1
1


<i>I B</i> <i>AB</i>
<i>I C</i> = <i>AC</i>


Chứng minh tương tự ta có: 2
2


<i>I B</i> <i>DB</i>
<i>I C</i> = <i>DC</i>


Theo câu 2) ta có 1 2


1 2


. . <i>AB</i> <i>DB</i> <i>I B</i> <i>I B</i>
<i>AB CD</i> <i>BD AC</i>


<i>AC</i> <i>DC</i> <i>I C</i> <i>I C</i>


= ⇒ = ⇒ =


Mà I1, I2 cùng thuộc đoạn BC nên chúng chia trong đoạn BC theo các tỉ số bằng nhau.


⇒ I1≡ I2


⇒Đường phân giác góc BAC, đường phân giác góc BDC và đường thẳng BC đồng quy.



4. Gọi M’ là điểm thuộc đoạn BC sao cho CAM’ = BAD . Ta chứng minh M’ ≡ M.
Vì CAM’ = BAD => BAM’ = CAD


Vì ABDC là tứ giác nội tiếp nên ADB = ACM’ (hai góc nội tiếp cùng chắn cung AB)
Mà CAM’ = BAD => ∆ADB ~ ∆ACM’ (g.g) . . '


'


<i>BD</i> <i>AD</i>


<i>BD AC</i> <i>AD CM</i>


<i>CM</i> <i>AC</i>


⇒ = ⇒ = (1)


Chứng minh tương tựta có: AB.CD = AD.BM’ (2)
Từ (1) và (2) với chú ý BD.AC = AB.CD => AD.CM’ = AD.BM’ => CM’ = BM’


⇒M’ ≡ M
=> BAD = MAC


<b>Bài 5. V</b>ới mọi a, b, c > 0, ta có:


2 2 2 2 2 2


2 2 2


2 2 2 2 2 2



2 2 2 2


( ) ( ) ( ) 0 2 2 2 2 2 2 0


2( ) 2 2 2


3( ) 2 2 2


3( ) ( ) (*)


<i>a b</i> <i>b c</i> <i>c a</i> <i>a</i> <i>b</i> <i>c</i> <i>ab</i> <i>bc</i> <i>ca</i>


<i>a</i> <i>b</i> <i>c</i> <i>ab</i> <i>bc</i> <i>ca</i>


<i>a</i> <i>b</i> <i>c</i> <i>a</i> <i>b</i> <i>c</i> <i>ab</i> <i>bc</i> <i>ca</i>


<i>a</i> <i>b</i> <i>c</i> <i>a b c</i>


− + − + − ≥ ⇔ + + − − − ≥


⇔ + + ≥ + +


⇔ + + ≥ + + + +


⇔ + + ≥ + +


</div>
<span class='text_page_counter'>(64)</span><div class='page_container' data-page=64>

3


3



3 0


1 1 1


( ) 9


1 1 1 1


3 0


1 1 1 9


(**)


<i>a b c</i> <i>abc</i>


<i>a b c</i>


<i>a</i> <i>b</i> <i>c</i>


<i>a</i> <i>b</i> <i>c</i> <i>abc</i>


<i>a</i> <i>b</i> <i>c</i> <i>a b c</i>


 + + ≥ >


 <sub>⇒</sub> <sub>+ +</sub>  <sub>+ +</sub> <sub>≥</sub>


  



 


+ + ≥ >





⇒ + + ≥


+ +


Áp dụng BĐT (*) với a = x, b = y, c = z và từđiều kiện của x, y, z ta có:


2


2 2 2


2


( )


18


3
( ) 3( ) 54 0


( 9)( 6) 0


<i>x</i> <i>y</i> <i>z</i>



<i>x</i> <i>y</i> <i>z</i> <i>x</i> <i>y</i> <i>z</i> <i>x</i> <i>y</i> <i>z</i>


<i>x</i> <i>y</i> <i>z</i> <i>x</i> <i>y</i> <i>z</i>


<i>x</i> <i>y</i> <i>z</i> <i>x</i> <i>y</i> <i>z</i>


+ +


≥ + + + + + ≥ + + +


⇒ + + + + + − ≤


⇒ + + + + + − ≤


6
<i>x</i> <i>y</i> <i>z</i>


⇒ + + ≤ (do x + y + z + 9 > 0) (***)


Áp dụng BĐT (**) với a = x + y + 1, b = y + z + 1, c = z + x + 1, ta có:


1 1 1 9 9


1 1 1 1 1 1 2( ) 3


<i>B</i>


<i>x</i> <i>y</i> <i>y</i> <i>z</i> <i>z</i> <i>x</i> <i>x</i> <i>y</i> <i>y</i> <i>z</i> <i>z</i> <i>x</i> <i>x</i> <i>y</i> <i>z</i>



= + + ≥ =


+ + + + + + + + + + + + + + + + +


Áp dụng (***) ta có: 9 3


2.6 3 5


<i>B</i>≥ =


+


Dấu bằng xảy ra 1 1 1 2


6


<i>x</i> <i>y</i> <i>z</i>


<i>x</i> <i>y</i> <i>y</i> <i>z</i> <i>z</i> <i>x</i> <i>x</i> <i>y</i> <i>z</i>


<i>x</i> <i>y</i> <i>z</i>


= =




⇔<sub></sub> + + = + + = + + ⇔ = = =
 + + =





Vậy giá trị nhỏ nhất của B là3


5 , xảy ra khi và chỉkhi x = y = z = 2.


<b>Đề</b>

<b> s</b>

<b>ố</b>

<b> 11 </b>



<b>Bài 1.</b>


2


2


6 10
) 5 6 10 3 2 2 ( )


5 5


5 6 2 10 3 2 2 6
5( 2) 3( 2)


( 2)(2 3) 0
5 6 2 10 3 2


5 3


(x 2)( 2 3) 0


5 6 2 10 3 2


2( )


5 3


2 3 0(*)
5 6 2 10 3 2


<i>a</i> <i>x</i> <i>x</i> <i>x</i> <i>x</i> <i>x</i>


<i>x</i> <i>x</i> <i>x</i> <i>x</i>


<i>x</i> <i>x</i>


<i>x</i> <i>x</i>


<i>x</i> <i>x</i>


<i>x</i>


<i>x</i> <i>x</i>


<i>x</i> <i>TM</i>


<i>x</i>


<i>x</i> <i>x</i>


− + − = − − ≤ ≤


<=> − − + − − = − −



− −


<=> − − − + =


− + − +


<=> − − − − =


− + − +


=



<=>  − − − =


 − + − +


</div>
<span class='text_page_counter'>(65)</span><div class='page_container' data-page=65>

6 10 5 5 5


5 6 2 2 3 0


5 5 5 6 2 2 5 6 2


6 10 3


2 0


5 5 10 3 2



5 3


2 3 0 (*)
5 6 2 10 3 2


<i>x</i> <i>x</i>
<i>x</i> <i>x</i>
<i>x</i> <i>x</i>
<i>x</i>
<i>x</i> <i>VN</i>
<i>x</i> <i>x</i>


≤ ≤ => − + ≥ => ≤ => − <


− + − +


≤ ≤ => − − <


− +


=> − − − < =>


− + − +


Vậy tập nghiệm của phương trình (1) là {2}
2)


3 2



2 2


8 96
32 48


<i>x</i> <i>xy</i> <i>y</i>


<i>x</i> <i>y</i>


 + =





+ =


 <sub> </sub>(I)


3 2 3 2 2 2


2 2 2 2


3 2 2 3


3 3


2 2


2 2 2 2



8 48.2 8 2 ( 32 )(*)
(I)


32 48 32 48


(*) 2 8 64 0


(4 ) 2 ( 4 ) 0
( 4 )( 2 16 ) 0


4 4


2 16 0 ( ) 15 0
4


<i>x</i> <i>xy</i> <i>y</i> <i>x</i> <i>xy</i> <i>y x</i> <i>y</i>


<i>x</i> <i>y</i> <i>x</i> <i>y</i>


<i>x</i> <i>x y</i> <i>xy</i> <i>y</i>


<i>x</i> <i>y</i> <i>xy x</i> <i>y</i>


<i>x</i> <i>y x</i> <i>xy</i> <i>y</i>


<i>x</i> <i>y</i> <i>x</i> <i>y</i>


<i>x</i> <i>xy</i> <i>y</i> <i>x</i> <i>y</i> <i>y</i>


<i>x</i> <i>y</i>



<i>x</i>


 + =  + = +


 


<=><sub></sub> <=><sub></sub>


+ = + =


 


 


<=> − + − =


<=> − − − =


<=> − + + =


= =


 


<=><sub></sub> <=><sub></sub>


+ + = + + =
 
=


<=>
0
<i>y</i>

 = =


Vì x = y = 0 khơng thỏa mãn hệphương trình nên x = 4y


2 2 2 2


2


4 4


( )


32 48 16 32 48


4
1
4


1 4


1


<i>x</i> <i>y</i> <i>x</i> <i>y</i>


<i>I</i>



<i>x</i> <i>y</i> <i>y</i> <i>y</i>


<i>x</i>
<i>y</i>
<i>x</i> <i>y</i>
<i>y</i> <i>x</i>
<i>y</i>
= =
 


<=><sub></sub> <=><sub></sub>


+ = + =
 
 =

 <sub>=</sub>
=
 <sub></sub><sub></sub>


<=><sub></sub> <=><sub></sub>


=  = −


 <sub></sub>


= −




Vậy hệphương trình có nghiệm (4;1), (–4;–1)
<b>Bài 2.</b>


1) Phương trình x2 – 2x – 4 = 0 có hai nghiệm x<sub>1</sub>; x<sub>2</sub>. Theo định lý Vi–ét ta có: 1 2
1 2
2
4
<i>x</i> <i>x</i>
<i>x x</i>
+ =

 <sub>= −</sub>

Ta có:


3 3 3


1 2 1 2 1 2 1 2


3 3 3 3


1 2 1 2 1 2 1 2


( ) 3 ( )


( ) 3 ( ) 2 3.( 4).2 32


<i>x</i> <i>x</i> <i>x</i> <i>x</i> <i>x x x</i> <i>x</i>



<i>x</i> <i>x</i> <i>x</i> <i>x</i> <i>x x x</i> <i>x</i>


+ = + + +


=> + = + − + = − − =




2 2 2 2


1 2 1 2 1 2


4 4 2 2 2 2 2 2 2


1 2 1 2 1 2


( ) 2 2 2.( 4) 12


( ) 2 12 2.( 4) 112


<i>x</i> <i>x</i> <i>x</i> <i>x</i> <i>x x</i>


<i>x</i> <i>x</i> <i>x</i> <i>x</i> <i>x x</i>


+ = + − = − − =


+ = + − = − − =


</div>
<span class='text_page_counter'>(66)</span><div class='page_container' data-page=66>

3 3 4 4 7 7 3 4 4 3



1 2 1 1 1 2 1 2 1 2


7 7 3 3 4 4 3


1 2 1 2 1 1 1 2 1 2


3


( )( )


( )( ) ( ) ( )
32.112 ( 4) .2 3712


<i>x</i> <i>x</i> <i>x</i> <i>x</i> <i>x</i> <i>x</i> <i>x x</i> <i>x x</i>


<i>S</i> <i>x</i> <i>x</i> <i>x</i> <i>x</i> <i>x</i> <i>x</i> <i>x x</i> <i>x</i> <i>x</i>


+ + = + + +


=> = + = + + − +


= − − =


Vậy S = 3172.
2) Ta có


2 2 2 2 2 2 2 2


2 2



2 2


( ) ( ) ( )( )(*)


<i>a</i> <i>ab b</i> <i>c</i> <i>cd</i> <i>d</i> <i>a</i> <i>ab b</i> <i>c</i> <i>xd</i> <i>d</i> <i>ab cd</i>


<i>ab cd</i> <i>a</i> <i>b</i> <i>c</i> <i>d</i> <i>a</i> <i>b</i> <i>c</i> <i>d a</i> <i>b c</i> <i>d</i>


+ + = + + => + + = + + + −


<=> − = + − − = + + + + − −


Nếu ab-cd=0: Do a+b+c+d>0=>a+b-c-d=0=>a+b+c+d=2(c+d) là hợp sốdo c + d ∈ℕ* và c + d >
1


Nếu ab -cd ≠0:Từ (*) ⇒ ab – cd ⋮ (a + b + c + d).


2 2 2 2 2 2 2 2


2 2


3( ) ( 2 ) 2


3( ) ( ) ( ) ( )( ) 0


<i>a</i> <i>ab b</i> <i>c</i> <i>cd</i> <i>d</i> <i>ab cd</i> <i>a</i> <i>ab b</i> <i>c</i> <i>cd</i> <i>d</i>


<i>ab cd</i> <i>c</i> <i>d</i> <i>a b</i> <i>c</i> <i>d</i> <i>a b c</i> <i>d</i> <i>a b</i>


+ + = + + => − + − + = − +



=> − = − − − = − + − − − + ≠


⇒ (c – d + a – b)(c – d – a + b) ⋮ (a + b + c + d)
Giả sử a + b + c + d là số nguyên tố thì ta có


c – d + a – b ⋮ a + b + c + d hoặc c – d – a + b ⋮ a + b + c + d


Điều này mâu thuẫn do –(a + b + c + d) < c – d + a – b < a + b + c + d ;


–(a + b + c + d) < c – d – a + b < a + b + c + d và (c – d + a – b)(c – d – a + b) ≠ 0
Vậy a + b + c + d là hợp số.


<b>Bài 3.</b>


Thay 1 = a + b + c ta có:
A+bc=a(a+b+c)+bc=(a+b)(a+c)
Do đó:


2 2 2


1 1


( )( )


<i>a bc</i> <i>a bc</i> <i>bc</i> <i>bc</i> <i>bc</i>


<i>a bc</i> <i>a bc</i> <i>a bc</i> <i>a b a</i> <i>c</i>


− <sub>=</sub> + − <sub>= −</sub> <sub>= −</sub>



+ + + + +


Ta có 2 đẳng thức tương tự


2
1


( )( )


2
1


(c a)(c b)


<i>b ca</i> <i>ca</i>


<i>b</i> <i>ca</i> <i>b</i> <i>c b</i> <i>a</i>


<i>c</i> <i>ab</i> <i>ab</i>


<i>c</i> <i>ab</i>




= −


+ + +


− <sub>= −</sub>



+ + +


Cộng từng vế của 3 đẳng thức trên ta có:
3 2


(a )(a ) ( )( ) (c a)(c b)


<i>a bc</i> <i>b ca</i> <i>c ab</i> <i>bc</i> <i>ca</i> <i>ab</i>


<i>a bc</i> <i>b ca</i> <i>c</i> <i>ab</i> <i>b</i> <i>c</i> <i>b c b a</i>


 


− − −


+ + = − <sub></sub> + + <sub></sub>


+ + + <sub></sub> + + + + + + <sub></sub>


Do đó:


2 2 2 2 2 2 2 2 2 2 2 2


2 2 2


3 3


2 (a )(a ) ( )( ) (c a)(c b) 4



( ) ( ) ( ) 3


( )( )( ) 4


4( ) 3( 2 )


<i>a bc</i> <i>b ca</i> <i>c</i> <i>ab</i> <i>bc</i> <i>ca</i> <i>ab</i>


<i>a bc</i> <i>b ca</i> <i>c</i> <i>ab</i> <i>b</i> <i>c</i> <i>b c b</i> <i>a</i>


<i>bc b c</i> <i>ca c</i> <i>a</i> <i>ab a b</i>


<i>a b b c c</i> <i>a</i>


<i>b c bc</i> <i>c a</i> <i>ca</i> <i>a b</i> <i>ab</i> <i>a b</i> <i>ab</i> <i>b c bc</i> <i>c a</i> <i>ca</i> <i>abc</i>


<i>b c bc</i> <i>c a</i> <i>c</i>


 


− <sub>+</sub> − <sub>+</sub> − <sub>≤ <=></sub> <sub>+</sub> <sub>+</sub> <sub>≥</sub>


 


+ + + <sub></sub> + + + + + + <sub></sub>


+ + + + +


<=> ≥



+ + +


<=> + + + + + ≥ + + + + + +


<=> + + + 2 2 2


6 (*)


</div>
<span class='text_page_counter'>(67)</span><div class='page_container' data-page=67>

Áp dụng BĐT Cô–si cho ba sốdương ta có:


2 2 2


2 2 2


3


(*)
3


<i>b c c a</i> <i>a b</i> <i>abc</i>


<i>bc</i> <i>ca</i> <i>ab</i> <i>abc</i>


 + + ≥


 <sub>=></sub>




+ + ≥



 đúng


Vậy BĐT đã cho được chứng minh.
Dấu bằng xảy ra khi 1


3
<i>a</i>= = =<i>b</i> <i>c</i>


<b>Bài 4.</b>


1) Vì AI // DC (do ABCD là hình bình hành) nên AIJ= DCJ (so le trong)


Vì AJ // BC nên AJI= BCJ (đồng vị)


Mà CJ là phân giác góc BCD nên DCJ= BCJ=> AIJ= AJI ⇒ ∆ AIJ cân ở A


Do O là tâm đường tròn ngoại tiếp ∆ AIJ cân nên AO là trung trực IJ đồng thời là phân
giácgóc IAJ.


2) Vì JD // BC nên DJC= JCB= JCD ⇒ ∆ JDC cân tại D


Suy ra JD = DC = AB (do ABCD là hình bình hành)
Ta có OA = OJ ( bằng bán kính (O))


Xét ∆ OAJ với góc ngồi OJD có:


OJD= AOJ +OAJ =2AIJ+ OAJ= 2DCJ +OAJ= DCB +OAJ=DAB+OAJ=OAB
Xét ∆ OAB và ∆ OJD có:



( )


( ) ( . . )


( )
<i>OA</i> <i>OJ cmt</i>


<i>OAB</i> <i>OJD cmt</i> <i>OAB</i> <i>OJD c g c</i>
<i>AB</i> <i>JD cmt</i>


<i>OBA</i> <i>ODJ</i>
=




 <sub>=</sub> <sub>=> ∆</sub> <sub>= ∆</sub>




 <sub>=</sub>




=> =


⇒AODB là tứ giác nội tiếp


</div>
<span class='text_page_counter'>(68)</span><div class='page_container' data-page=68>

3) Vì ∆ OAB = ∆ OJD nên OB = OD. Mà O’B = O’D (bằng bán kính (O’)) nên OO’
làtrung trực của BD.



Gọi K là giao BD và AC ⇒K là trung điểm BD và AC.


⇒ K ∈ OO’


Vì OA = OM, O’A = O’M nên OO’ là trung trực của AM
Mà K ∈ OO’ ⇒ KA = KM = KC


⇒ M thuộc đường tròn tâm K bán kính KA, hay đường trịn đường kính AC.
Vậy khi B, D thay đổi, M luôn nằm trên đường trịn đường kính AC.


<b>Bài 5.</b>


Xét 20 sốđầu tiên. Trong 20 số này có 2 số chia hết cho 10, chúng có chữ sốhàng đơn vị là
0.


Mặt khác, trong 2 sốđó có một số có chữ số hàng chục khác 9.
Gọi sốđó là N. Xét dãy 11 số thuộc 39 sốđã cho:


N, N + 1, ... , N + 9, N + 19
Tổng các chữ số của các sốnày tương ứng là.


s, s + 1, s + 2, ..., s + 9, s + 10


Thật vậy, nếu N có tổng chữ số là s thì mỗi số N + i với 1 ≤ i ≤ 9 có tất cả các chữ số (trừ
hangđơn vị) giống số N và chữ sốhàng đơn vị của N + i là i, do đó tổng chữ số của N + i là
s + i.


Số N + 19 có chữ sốhàng đơn vị là 9, chữ số hàng chục hơn chữ số hàng chục của số N là 1,
cònlại tất cả các chữ sốở hàng khác của hai số bằng nhau, do đó tổng chữ số của N + 19 là
s + 10.



Trong 11 số liên tiếp s, s + 1, s + 2, ..., s + 9, s + 10 có một số chia hết cho 11.
Bài toán được chứng minh.


<b>Đề</b>

<b> s</b>

<b>ố</b>

<b> 12 </b>



<b>Bài 1</b>


<b>1)</b> <b>1.5 điểm </b>




1 1


( 4)
2 2 ( 2)( 2)


( 2) 2 1


( 4)
4


2 3


( 4)
4


3
<i>x</i>



<i>P</i> <i>x</i>


<i>x</i> <i>x</i> <i>x</i> <i>x</i>


<i>x</i> <i>x</i> <i>x</i>


<i>x</i>
<i>x</i>


<i>x</i> <i>x</i> <i>x</i>


<i>x</i>
<i>x</i>


<i>x</i> <i>x</i>


 


=<sub></sub><sub></sub> + + <sub></sub><sub></sub> −


+ − + −


 


 − + + + 


=<sub></sub> <sub></sub> −





 


− + +


= −




= − +


<b>2)</b> <b>0.5 điểm </b>


</div>
<span class='text_page_counter'>(69)</span><div class='page_container' data-page=69>



2


min
1 11 11


3


2 4 4


11 1 1 11


0 ( / ).


4 2 4 4


<i>P</i> <i>x</i> <i>x</i> <i>x</i>



<i>P</i> <i>x</i> <i>x</i> <i>t m</i> <i>P</i>


 


= − + =<sub></sub> − <sub></sub> + ≥


 


= ⇔ − = ⇔ = ⇒ =




<b>Bài 2: </b>


<b>1)</b> <b>1,0 điểm:</b>


+) T hay m=1 vào hệphương trình ta được: 1 (1)
7 (2)
<i>x</i> <i>y</i>


<i>x</i> <i>y</i>
− =


 + =


+) Cộng vế với vế của (1) với (2) ta được 2<i>x</i>= ⇔ =8 <i>x</i> 4
+) thay <i>x</i>=4vào (1) ta được 4− = ⇔ =<i>y</i> 1 <i>y</i> 3



+) Vậy với m=1 thì hệ có nghiệm (x;y)=(4;3)
<b>2)</b> <b>1,0 điểm:</b>


+)Từpt đầu ta có: y = mx - 1


+)Thay y=mx-1 vào pt thứhai ta được:


2


2


2 2


2 2


2 6


( 1) 6 ( 1) 2 6


1


2 6 6 1


1


1 1


<i>m</i>



<i>x</i> <i>m mx</i> <i>m</i> <i>m</i> <i>x</i> <i>m</i> <i>x</i>


<i>m</i>


<i>m</i> <i>m</i> <i>m</i> <i>m</i>


<i>y</i>


<i>m</i> <i>m</i>


+


+ − = − ⇔ + = + ⇔ =


+


+ + −


⇒ = − =


+ +


+) theo giả thiết: 3x-y=1




2


2 2



2 2


2
2


2 6 6 1


3. 1


1 1


6 18 6 1 1


0
1


2 18 0 3


<i>m</i> <i>m</i> <i>m</i>


<i>m</i> <i>m</i>


<i>m</i> <i>m</i> <i>m</i> <i>m</i>


<i>m</i>


<i>m</i> <i>m</i>


+ + −



⇔ − =


+ +


+ − − + − −


⇔ =


+


⇔ − + = ⇔ = ±


<b>Bài 3: </b>
<b>1)</b> <b>1 điểm </b>


2 2


) (2<i>m</i> 1) 4(<i>m</i> <i>m</i> 6) 25 0 <i>m</i> <i>R</i>


+ ∆ = − − − − = > ∀ ∈


Vậy ……


</div>
<span class='text_page_counter'>(70)</span><div class='page_container' data-page=70>

1 2


2 1 2 1


3; 2


5 0



<i>x</i> <i>m</i> <i>x</i> <i>m</i>


<i>x</i> <i>x</i> <i>x</i> <i>x</i>


= − = +


− = > ⇒ >


+) theo giả thiết :


1 2


3 5 2


5 5 2 3


2 5 3


: ...


<i>m</i> <i>m</i>


<i>x</i> <i>x</i> <i>m</i>


<i>m</i> <i>m</i>


<i>KL</i>


− > − > −



 


− < < < ⇔<sub></sub> ⇔<sub></sub> ⇔ − < <
+ < <


 


<b>2)</b> <b>1 điểm: </b>


Ta có pt tương đương với:


[

][

]

2


2 2 2


0
( 2)( 6) ( 3)( 4) 16


( 12 8 )( 12 7 ) 16


12 12


( 8)( 7) 16( x=0 ko la ng )


<i>x</i> <i>x</i> <i>x</i> <i>x</i> <i>x</i>


<i>x</i> <i>x x</i> <i>x</i> <i>x</i>


<i>x</i> <i>x</i> <i>do</i>



<i>x</i> <i>x</i>


+ + − − =


⇔ + + + − =


⇔ + + + − =


Đặt <i>t</i> <i>x</i> 12
<i>x</i>


= + ……


<b>Bài 4:</b>


a) <i>Do</i> <i>MP</i> <i>AB</i>


<i>MQ</i> <i>AC</i>





 <sub>⊥</sub>


 nên…….


b) ∆<i>BHA</i>∆<i>BPM</i> vì 0
90
<i>Bchung</i>



<i>BHA</i> <i>BPM</i>





= =







  <sub>………</sub>


c) Góc AHM vng nên H thuộc (O)


Tam giác ABC đều với AH là đường cao nên AH cũng là phân giác trong của góc BAC nên cung
HP bằng cung HQ . Do đó H là điểm chính giữa của cung PQ


Mặt khác PQ khơng là đường kính nên OH vng góc PQ
d) ta có:


. . .


( AB=BC=AC)


<i>ABC</i> <i>MAB</i> <i>MAC</i>


<i>S</i> <i>S</i> <i>S</i> <i>BC AH</i> <i>AB MP</i> <i>AC MQ</i>



<i>AH</i> <i>MP</i> <i>MQ do</i>


∆ = ∆ − ∆ ⇔ = −


⇔ = −


</div>
<span class='text_page_counter'>(71)</span><div class='page_container' data-page=71>

<b>Bài 5:</b>


+) đặt


2


2013 1


2
<i>x</i>
<i>y</i>


<i>x</i>



=


− . Thay vào pt ta được:


<sub>(</sub>

<sub>)</sub>

<sub>(</sub>

<sub>)(</sub>

<sub>)</sub>

<sub>(</sub>

<sub>)</sub>



(

)



(

)

(

)(

)

(

)




3
3


3 3


3
3


2 2


3 3 3


2 2


3 3 3


2 2014 2013 1


2 1 2013 2014 (*)


2 2013 2014 1


2013 2013


2 2013 <sub>2014</sub> <sub>2014</sub> <sub>1</sub> <sub>1</sub>


1 1


2013



2 2013 <sub>2014</sub> <sub>2014</sub> <sub>1</sub> <sub>1</sub>


2013


<i>x</i> <i>y</i> <i>y</i> <i>x</i> <i>x</i>


<i>x</i> <i>y</i> <i>x</i> <i>x</i> <i>y</i>


<i>x</i> <i>y</i> <i>x</i> <i>y</i> <i>x</i>


<i>x</i> <i>y</i> <i>x</i> <i>y</i>


<i>x</i> <i>y</i> <i>x</i> <i><sub>y</sub></i> <i><sub>y</sub></i> <i><sub>x</sub></i> <i><sub>x</sub></i>


<i>x</i> <i>y</i>


<i>x</i> <i>y</i> <i>x</i> <i><sub>y</sub></i> <i><sub>y</sub></i> <i><sub>x</sub></i> <i><sub>x</sub></i>


<i>x</i> <i>y</i>


+ − − = + − +


⇔ + + + = + + −


⇔ + − + = − − +


+ − − −


⇔ =



+ − + <sub>−</sub> <sub>+</sub> <sub>−</sub> <sub>+ +</sub> <sub>+</sub>


 


 


⇔ + − +


 <sub>+ −</sub> <sub>+</sub> <sub>−</sub> <sub>+</sub> <sub>−</sub> <sub>+ +</sub> <sub>+</sub> 


 


</div>
<span class='text_page_counter'>(72)</span><div class='page_container' data-page=72></div>

<!--links-->

×